Anda di halaman 1dari 172

PROGRAMA NACIONAL

OLIMPADAS
EXAMES E RESULTADOS
XVIIOlimpada
Norte/NordestedeQumica
16.Olimpada
Ibero-americanadeQumica
Olimpada
BrasileiradeQumica
43
rd
International
ChemistryOlympiad
Prof. Jesualdo Pereira Farias
Reitor da Universidade Federal do Cear
Prof. Luiz de Sousa Santos Jnior
Reitor da Universidade Federal do Piau
Prof. Tarcsio Haroldo Cavalcante Pequeno
Presidente da FUNCAP
Prof. Antnio Salvador da Rocha
Pro-Reitor de Extenso da UFC
Profa. Maria da Glria Carvalho Moura
Pr-Reitora de Extenso da UFPI
Newton Mario Battastini
Presidente da Associao Brasileira de Qumica

Prof. Srgio Maia Melo
Coordenador do Programa Nacional Olimpadas de Qumica
Prof. Jos Arimatia Dantas Lopes
Vice-coordenador do Programa Nacional Olimpadas de Qumica e
Coordenador da Equipe Pedaggica (docentes do Estado do Piau)
Prof. Antnio Carlos Pavo
Prof. Cristiano Marcelino Jnior
Prof. Carlos Eduardo G. da Silva
Preparao dos exames experimentais e edio de ftas de vdeo


ISSN: 1809-2012
Imprensa Universitria
Universidade Federal do Cear
Organizao de originais:
Prof. Srgio Melo

Capa:
Maherle
Editorao e Projeto Grfco:
Maherle/Srgio Melo
2011 Programa Nacional Olimpadas de Qumica
Lanamento em 25.11.2011 por ocasio da solenidade de encerramento dos eventos: XVII Olimpada Norte/
Nordeste de Qumica, IV Olimpada Brasileira de Qumica Jnior e Olimpada Brasileira de Qumica - 2011
Tiragem: 15.000 exemplares.
Distribuio gratuita.
ndice
1 Olimpadas Nacionais de Qumica
Calendrio das Atividades realizadas em 2011 . . . . . . . . . . . . . . 4
Mensagem do Diretor do Instituto de Qumica da Universidade de So Paulo aos
participantes das Olimpadas de Qumica . . . . . . . . . . . . . . . . . . . 5
OPINIO: O Ano Internacional da Qumica Carlos Vogt . . . . . . . . . . . 7
XVII Olimpada Norte /Nordeste de Qumica - Exames 2011 . . . . . . . . . . 9
XVII Olimpada Norte/Nordeste de Qumica - Relao dos aprovados . . . . . . 23
IV Olimpada Brasileira de Qumica Jnior Exames 2011- Fase I . . . . . . . . 31
IV Olimpada Brasileira de Qumica Jnior Exames 2011 - Fase II. . . . . . . . 38
IV Olimpada Brasileira de Qumica Jnior - Relao dos aprovados . . . . . . . 46
Olimpada Brasileira de Qumica - 2011 Fase III modalidade A . . . . . . . 49
Olimpada Brasileira de Qumica - 2011 Fase III modalidade B . . . . . . . 55
OBQ - 2011 Fase III. Soluo dos problemas das Modalidades A e B . . . . . . 60
Olimpada Brasileira de Qumica - 2011 - Resultado e classifcao . . . . . . 69
2 Processo seletivo para compor equipe nas olimpadas internacionais em 2011
Exame sobre conhecimentos de tcnicas de Laboratrio (OBQ-2010 Fase IV) . . 74
Curso de Aprofundamento e Excelncia em Qumica (OBQ-2010 Fase V) . . . . 76
Exame fnal para selecionar a equipe brasileira na IChO (OBQ-2010 Fase VI) . . 76
16 Olimpada Ibero-americana de Qumica - Exame Prtico . . . . . . . . . . 82
16 Olimpada Ibero-americana de Qumica - Exame Terico . . . . . . . . . . 90
43
rd
International Chemistry Olympiad/2011 - Exame Prtico . . . . . . . . . 98
43
rd
International Chemistry Olympiad - Exame Terico . . . . . . . . . . . 112
Destaques olmpicos - 2011 . . . . . . . . . . . . . . . . . . . . . . 120
Depoimentos . . . . . . . . . . . . . . . . . . . . . . 123
Consideraes fnais - Prof. Srgio Melo . . . . . . . . . . . . . . . . 125
Endereos dos Coordenadores nos Estados participantes
do Programa Nacional Olimpadas de Qumica . . . . . . . . . . . . . . . 128
| 5 4 |
Programa Nacional
Olimpadas de Qumica
Ano Internacional de Qumica
Calendrio 2011
Data Hora Atividade
01/02/2011 14:00h Fase IV da OBQ-2010 (Exame sobre tcnicas laboratoriais com o objetivo
de selecionar a equipe que representou o Brasil na Turquia e na OIAQ).
21/03 a 01/04/2011 Curso de Aprofundamento e Excelncia (OBQ -2010 Fase V) para os 15 estudantes se-
lecionados no exame de conhecimentos de laboratrio. Foi ministrado pela UNICAMP.
Estudantes se deslocaram at Campinas.
16/04/2011 9:00h Exames da Olimpada Brasileira de Qumica - 2010. Fase VI - Questes de
baseadas na lista sugerida pelos organizadores da 43
rd
IChO.
30/04/2011 Divulgao dos nomes dos quatro estudantes que representam o Brasil nas com-
peties internacionais em 2011.
03 a 21/05/2011 Inscries para a XVII Olimpada Norte/Nordeste de Qumica - XVII ONNeQ. Quarenta
estudantes por estado. Inscries restritas aos coordenadores-estaduais.
04/06/2011 Exames da Olimpada Norte/Nordeste de Qumica - XVII ONNeQ.
03/07/2011 Divulgao de resultados da Olimpada Norte/Nordeste de Qumica - XVII ONNeQ.
06/06 a 26/08/2011 Inscries para a IV Olimpada Brasileira de Qumica Jnior. Escolas inscrevem seus alu-
nos de 8
o
e 9
o
anos do ensino fundamental.
09 a 18/07/2011 43
a
Olimpada Internacional de Qumica, Ankara - Turquia .
01 a 15/08/2011 Inscries para a Olimpada Brasileira de Qumica - 2011. Vinte estudantes por estado
na modalidade A (estudantes da penltima srie do ensino mdio ou srie anterior),
vinte estudantes por estado na modalidade B (estudantes da ltima srie do ensino
mdio). Inscries reservadas aos coordenadores-estaduais.
27/08/2011 8:30h Exames da IV Olimpada Brasileira de Qumica Jnior - OBQjr, para estu-
dantes de 8
o
e 9
o
anos. (Fase I).
27/08/2011 14:00h Exames da Olimpada Brasileira de Qumica - 2011 Fase III - Modalidades
A e B. Questes analtico-expositivas.
16 a 24/09/2011 16 Olimpada Ibero-americana de Qumica, Teresina - Piau.
01/10/2011 Exames da IV Olimpada Brasileira de Qumica Jnior, OBQjr. (Fase II).
30/10/2011 Divulgao dos resultados da OBQ-2011. A partir de 30.10.2011.
19 a 25/10/2011 Semana Nacional de Cincia e Tecnologia.
30/10/2011 Divulgao dos resultados da OBQ-2011 e IV OBQjr. A partir de 30.10.2011.
24 e 25/11/2011 8h30min
19:30h
Reunio do Conselho de coordenadores, em Fortaleza.
Solenidade de encerramento e premiao, em Fortaleza.
| 5
Olimpada Brasileira
de Qumica 2011
Ano Internacional de Qumica
Mensagem
M
anchete em vrios meios de comunicao,
todos fcamos sabendo que nosso planeta
atingiu a marca recorde de sete bilhes de
pessoas vivendo sobre sua superfcie. Com
uma taxa anual de crescimento de aproximadamente
1,1%, quando a gerao que hoje participa da Olimpa-
da Brasileira de Qumica atingir a meia-idade, daqui a 30
trinta anos, no havendo grandes catstrofes mundiais,
prev-se que a populao vai estar se aproximando de
dez bilhes de pessoas. Nesse contexto, algum poderia
perguntar: o que a Qumica tem a ver com isso? Um pou-
co de exerccio mental vai nos dizer que a Qumica tem
muito a ver com esse aumento populacional. Como alimentar essa massa crescente
de pessoas, principalmente aquelas vivendo em regies pobres e inspitas? Como
tratar de sua sade e se precaver de grandes epidemias? Como abrig-las em am-
bientes apropriadamente climatizados e com um mnimo de infraestrutura habita-
cional? Como permitir sua locomoo de um lugar para outro? Como evitar que fon-
tes energticas naturais se degradem e substncias essenciais ao desenvolvimento
no mundo moderno no venham a se exaurir? Como educar a populao para que
no venhamos a ter grandes catstrofes num futuro no muito distante? Poderamos
estender essa lista de perguntas com outras mais especfcas, mas essas poucas bas-
tam para podermos situar a Qumica como cincia central nesse contexto. Alimentar
grandes populaes implica desenvolver fertilizantes a baixo custo para que com
planejamento possamos obter do solo vrias colheitas anuais e alimentos mais sau-
dveis e nutritivos; implica sabermos evitar pragas nas plantaes e nas criaes de
animais para corte atravs do uso ambientalmente correto de produtos qumicos. O
tratamento da sade implica aprimorar nosso conhecimento dos mecanismos bio-
qumicos no corpo humano identifcando aqueles fatores desencadeadores de do-
enas e, consequentemente, a busca de medicamentos que possam contribuir para
sua cura. A educao bsica a respeito de cuidados higinicos pessoais e de ambien-
tes comuns essencial, principalmente em regies de baixo ndice de alfabetizao
para se evitar contaminaes de rios e do ar e o surgimento de focos de doenas.
Habitao, transporte e o crescimento industrial implicam em grande consumo de
energia. Pensar novas fontes de energia renovveis e de baixo custo e em aparelhos
e veculos de menor consumo atividade de pesquisa intensa no mundo moderno.
Mensagem do Diretor do Instituto de Qumica da Universidade
de So Paulo aos participantes das Olimpadas de Qumica
| 7 6 |
Programa Nacional
Olimpadas de Qumica
Ano Internacional de Qumica
Mensagem
No Brasil, fomos pioneiros no desenvolvimento de motores a etanol. Agora, temos o
grande desafo de no s mantermos a produo de cana de acar num patamar
compatvel com o desenvolvimento do pas como tambm de usarmos esse mesmo
etanol na busca de rotas sintticas industriais alternativas quelas oriundas de deri-
vados do petrleo. Nesse exerccio mental, restringindo o olhar ao nosso redor mais
prximo, vemos a qumica se manifestar nos alimentos e bebidas que ingerimos,
na vestimenta das pessoas, nos produtos de cuidados pessoais, nos livros e obras
de arte, nos equipamentos eletrnicos, no colorido da natureza e mesmo no nosso
humor. O jovem participante da Olimpada de Qumica, independentemente de sua
escolha de carreira profssional num futuro prximo, vai ter a oportunidade de cons-
tatar esse papel da Qumica como cincia central e de participar tambm como um
ator responsvel pela transformao sustentvel do planeta Terra. A todos, que essa
participao na Olimpada de Qumica possa contribuir no s para aferir conheci-
mento, e capacidade de raciocnio, mas que tambm oferea uma oportunidade de
autoconhecimento lembrando que disciplina, perseverana e humildade so ingre-
dientes essenciais para o sucesso na vida.
Prof. Dr. Fernando Rei Ornelas (*)
Diretor do Instituto de Qumica da
Universidade de So Paulo
* Bacharel, Licenciado e Mestre em Fsico-Qumica pelo IQ-USP e Ph.D. em Che-
mical Physics pela Universidade de Indiana, E.U.A.
Segundo site da Aracruz Celulose, 1 rvore com idade de corte (acima de sete
anos), produz 20695 folhas de papel A4 (tais como estas usadas neste exemplar).
Logo, 38 rvores foram cortadas para satisfazer a edio destes Anais do Programa
Nacional Olimpadas de Qumica. Faa bom proveito, recicle o material aps uso
exaustivo. Conforme o Portal SOS Mata Atlntica, 38 rvores resgatam da atmosfera
trs toneladas de CO
2
ao longo de cinco anos. Defenda o meio ambiente, engage-se
em campanhas ecolgicas de preservao ambiental.
| 7
Olimpada Brasileira
de Qumica 2011
Ano Internacional de Qumica
Opinio
O ano internacional da qumica
CarlosVogt
Todosconcordamos,ouaomenostendemosaconcordar,queacinciacontribui,deumaforma
oudeoutra,paraamelhoriadaqualidadedevidanoplaneta,emborasejatambmverdadequea
desconfana das populaes no tenha deixado de acompanhar o desenvolvimento cientfco e as
aplicaes do conhecimento na gerao das novas tecnologias e das inovaes que se incorporam
comfrequnciacadavezmaioraocotidianodenossasvidas.
Almdosaspectosligadosaobem-estarsocialqueacinciapodeacarretar,naformadasfacili-
dadesquepodeofereceratravsdesuasaplicaestecnolgicaseinovativas,houtraespciedecon-
fortoquedizrespeitosrelaesdasociedadecomastecnocincias,queenvolvevaloreseatitudes,
hbitoseinformaes,comopressupostodeumaparticipaoativamentecrticadessasociedadeno
conjuntodessasrelaes.Aessetipodeconfortochamobem-estarcultural,comojtrateiemoutro
artigo(Cinciaebem-estarcultural,publicadononmero119darevistaeletrnicaComCincia).
Aqumica,disciplinafundadoradacinciamoderna,segueospreceitosacimamencionados,h
muitogerandoconhecimentosquepromovemodesenvolvimentocientfcoeamelhoriadaqualidade
devidadapopulao,aomesmotempoemquevista,emalgumaspocas,eemtodasaspocas,
poralguns,comcertadesconfana,comose,porsis,aqumicapudesseofereceralgumriscoouse
confgurassecomoumaameaa.Noobstanteosusosquesoouforamfeitosdaqumicaedeseus
produtoscientfcosetecnolgicos,aOrganizaodasNaesUnidas(ONU)proclamou2011como
oAnoInternacionaldaQumica,pelosinmerosbenefciosdaqumicaparaahumanidade,ecomo
propsitodecelebr-lanomundotodo.
NolivroDezteoriasquecomoveramomundo,deLeonardoMoledoeEstebanMagnani,publica-
donoBrasilpelaEditoradaUnicamp,em2009,emtraduodooriginalargentino,de2006,aqumica
reverenciadapormeiodeumadescobertaquenosomentemudouosrumosdaprpriaqumica,
masquerepresentatambmumdosmarcosdacinciamoderna.Entreasteoriasescolhidaspelos
autores, como o heliocentrismo, a gravitao universal, o evolucionismo, a teoria atmica, a teoria
dainfecomicrobiana,arelatividade,ateoriadaderivacontinental,agenticaeoBigBang,est
aindaelencadaateoriadacombusto.Lavoisier,aoanunciarquenanaturezanadasecria,nadase
perde,tudosetransforma,criandooenunciadodoprincpiodeconservaodamatria,resolveuum
importanteproblemaparaospesquisadoresdarea:anaturezadacombusto.E,dequebra,afetou
osrumos,porquenodizer,daprpriahumanidade,nosentidodetransformaodoconhecimento
acumuladoatento.
Comasuadescoberta,oqumicofrancsentranoparadoxodacomoventehistriadoconhe-
cimentoque,ameuver,seriasimplesetransitriacomodefnitivaecomplexaaprovisoriedadeda
vida.Conhecerumatodecoragemquenosleva,deperguntaemperguntaaoconfrontodealterna-
tivas:ourecusamosoconhecimentocomodado,ounosaventuramosnoquenosdadoaconhecer.
Nestecaso,aindaqueabibliotecadenossosconhecimentossejaperidica,elasertambmilimi-
| 9 8 |
Programa Nacional
Olimpadas de Qumica
Ano Internacional de Qumica
tadacomoenunciouBorgessobreaBabel;nooutro,seremossomentedefnitivoselimitadospelos
murosabertosdolabirintodeareiadodesertodeinformaes.
H,assim,pelomenosdoismodosdeconhecer:aquelequenosabandonaenosperdenapla-
nitude da informao acumulada (termo elaborado no texto A inveno da planitude, publicado
nonmero120darevistaComCincia),tornando-nossbios-sabidos;aqueleque,mantendo-nosem
estadodeignornciacrticaoquechameiemoutroartigo(Cinciaebem-estarcultural)deig-
nornciacultural,noslevaadesconfardamiragembenfazejadoconhecimentodadoenospe
emconstanteestadodealertaparaoquevempronto,planoeamide,valedizer,osmonumentos
instantneosdascertezaspassageiras.
Nestecaso,muitoprovvelquetodosnosejamossbios;certo,contudo,queteremossabe-
doriaasabedoriaparadoxalquequantomaisaumenta,maisnosfazcresceremconhecimentoemais
nosdiminuioconfortopassivodassituaesobjetivasesubjetivasdecadaconquistaticaecultural.
AOrganizaodasNaesUnidas(ONU)escolheu,comfelizacerto,marcar2011comooAno
InternacionaldaQumica,dadoqueeletambmoanodocentenriodoNobelqueMarieCuriere-
cebeupeladescobertadoselementosqumicosrdioepolnio,depoisdehaverjrecebido,comseu
maridoPierreCurie,em1903,oNobeldefsicaporsuaspesquisasnocampodaradioatividade.
SomuitasaspublicaesecomemoraesnesteanodessemodosingularizadopelaONUpara
enfatizarosgrandesavanoseconquistasdacinciaatravsdeumadesuasexpressesmaissofsti-
cadasemaisimportantesnocampodoconhecimento.
OprprioLabjorjestevepresentenessashomenagenspelaspublicaesaeleligadas,direta
ouindiretamente,comocasodaedio,ano63,n.1,darevistaCincia&Cultura,dedicadoaotema
edaedion.8,fev/2011,darevistaPr-univesp,onde,alis,partedestetextofoitambmpublicado
comoeditorial.
Aimportnciadaqumicatograndeparaahistriadoconhecimentoe,portanto,paraobem-
estarculturaldahumanidade,alm,clarodesuaenormerelevnciaparaastransformaessociais
denossahistria,que,dealgummodo,possvelafrmarqueopensamentoantropolgico,caracte-
rsticodohomemmoderno,noseriapossvelsemadescobertaeasdescobertasdaqumica.
Comaqumica,frmam-setambmarevoluoindustrialetodasasconsequnciaseconmicas,
polticaseculturaisdelaadvindasnoplanodaorganizaodavidasocialnomundomoderno.
Hoje,aqumica,seguindoatendnciaepistemolgicadeagregaodereasdoconhecimento
cientfcoparaaformaodenovasreascomcaractersticasmultidisciplinarespredominantes,evolui
paraaconstituiodecamposdosabercomosquaiselaseencontra,porexemplo,afsica,abiologia,
afarmacutica,amedicina,agentica,agenmica,aprotemica,eumagrandevariedadedeestudos
epesquisasemnanocinciasenanotecnologias.
NoscemanosdoNobeldequmicadeMarieCurie,oprimeiroanodacelebraodadinmica
permanentedeumaformadeconhecimentoquesetornoudefnitivanosprocessosculturaisdeper-
mannciaetransformaodohomem,danaturezaedesuasrelaes.
CarlosVogtDiretordeRedaodaRevistaComCincia,
publicaoeletrnicadaSBPC.Artigopublicadoem10/07/2011.
Opinio
| 9
Olimpada Brasileira
de Qumica 2011
Ano Internacional de Qumica
XVII Olimpada Norte/Nordeste de Qumica
04/06/2011
Questo1
Paracadaumdosseguintescompostos:
Dissulfetodecarbono
Tricloretodefsforo
Tetrafuoretodeestanho
Monxidodedicloro

Escrevaafrmulamolecular
DesenheaestruturadeLewis
Dahibridaodotomocentral
Prevejaageometriamolecular
Questo2
Um tcnico dispe de um frasco de cido ntrico, em cujo rtulo est
escrito:
Concentrao=60%emmassa
Densidade=1,48g.mL
-1
a.Escrevaafrmuladocidontrico
b. Escreva a equao qumica correspondente neutralizao do cido
ntricopelohidrxidodeclcio
c.Determineaconcentraoemmol.L
-1
docidontricocontidonofrasco
d.Quevolumedessecidontricoserianecessrioparapreparar500mL
deumanovasoluodecidontricodeconcentrao2mol.L
-1
.
e.Quevolumedeumasoluodehidrxidodesdiodeconcentrao
XVII ONNeQ
| 11 10 |
Programa Nacional
Olimpadas de Qumica
Ano Internacional de Qumica
20 g.L
-1
seria necessrio para neutralizar 20 mL da soluo de cido
ntricopreparadanoitemanterior(itemd)?
Questo3

Um processo industrial usado para remover cido sulfdrico do gs na-


turalconsisteemreagi-locomdixidodeenxofre,conformeaequao
qumica(nobalanceada)abaixo:
H
2
S
(g)
+SO
2(g)
S(s)+H
2
O
(g)

a)Reescrevaaequaoqumicaacimabalanceada
b) Que volume de SO
2(g)
, medido a 1 atm e 25 C, necessrio para
produzir1kgdeenxofre?
c)Emumareaoiniciadacom100gdecadaumdosreagentesacima,
que reagente sobrar, considerando que um deles consumido to-
talmente?
d)Quemassadoreagenteemexcessosobraraofnaldareao?
Questo4
EmummisturadosgasesN
2
O
4
eNO
2
emequilbrio,representadopela
equaoqumicaabaixo,temperaturade0Cepressode1atm,as
pressesparciaisdessesgasesso,respectivamente,0,8atme0,2atm.

N
2
O
4
(g) 2NO
2
(g)
a)Calculeaconstantedeequilbrioexpressaempresses,Kp.
XVII ONNeQ
| 11
Olimpada Brasileira
de Qumica 2011
Ano Internacional de Qumica
b)Calculeaconstantedeequilbrioexpressaemconcentraes,Kc.
c)CalculeoHparaoequilbrioacima,apartirdosdadosdeentalpias
padresdereaoa273,15K,dadosaseguir:
N
2
(g)+2O
2
(g) 2NO
2
(g) H=+78,31kJ/mol
N
2
(g)+2O
2
(g) N
2
O
4
(g) H=+9,67kJ/mol
d)Umaumentodatemperaturalevaraumamaioroumenordissocia-
odeN
2
O
4
?
e)Calculeaspressesparciaisdeambososgases,apsumnovoequil-
brioseratingidoaosecomprimiramisturametadedovolumeoriginal,
mantendo-seatemperaturaconstante.
Questo5
Oparacetamol,umdosanalgsicosmaisconsumidosnomundo,pode
serpreparadoatravsdaseguintesequnciadereaes:
a)EscrevaosnomesdoscompostosA,B,C,DeE
b)Escrevaumnomesistemtico(IUPAC)paraoparacetamol
XVII ONNeQ
| 13 12 |
Programa Nacional
Olimpadas de Qumica
Ano Internacional de Qumica
Conforme mostrado no esquema acima, o reagente usado na etapa 2
dessasequnciadereaesfoioKOHenaetapa4foiH
2
,Pd/C.

c)Quaisreagentesforamusadosnasetapas1,3,4e5
_______________
Dado:R=0,082Latmmol
-1
K
-1

Aprincipalmetadaeducaocriarhomensquese-
jamcapazesdefazercoisasnovas,nosimplesmente
repetiroqueoutrasgeraesjfzeram.
JeanPiaget
XVII ONNeQ
| 13
Olimpada Brasileira
de Qumica 2011
Ano Internacional de Qumica
XVII Olimpada Norte/Nordeste de Qumica
04/06/2011
Resultadofnal
Nome Escola Cidade
UF
OURO
DaviRodriguesChaves ArideSCavalcante Fortaleza CE
NathiannedeMouradeAndrade FariasBrito Fortaleza CE
RaulBrunoMachadodaSilva FariasBrito Fortaleza CE
MateusBragadeCarvalho InstitutoDomBarreto Teresina PI
ThainNobreBarrosRodrigues Master Fortaleza CE
PRATA
BrenoSaldanhaSousa FariasBrito Fortaleza CE
DavidsonAnthonyAragoFreire ArideSCavalcante Fortaleza CE
EmersonHolandaMarinho FariasBritoCentral Fortaleza CE
TaynaraCarvalhoSilva Master Fortaleza CE
AlexSilvadeCerqueira IFBA Salvador BA
NatliaAragoDias Master Fortaleza CE
JosMatheusG.deAlencarBastos InstitutoDomBarreto Teresina PI
JosMarquesNeto ArideSCavalcante Fortaleza CE
MarcosVinciusNunesdeSouza GGE Recife PE
AlynneMaraAlencarJusta ArideSCavalcante Fortaleza CE
SergioPereiradeOliveiraJnior EspaoAbertoFB Fortaleza CE
TamiresBarbosadaSilva SantaBartolomea Macap AP
FranciscoDaviBarbosadosSantos FariasBritoCentral Fortaleza CE
JanainaGomesCastro InstitutoDomBarreto Teresina PI
PauloAndrHerculanodeLima FariasBrito Fortaleza CE
Resultados
| 15 14 |
Programa Nacional
Olimpadas de Qumica
Ano Internacional de Qumica
BRONZE
AdrielGarciaMaquinSenado IF-AM Manaus AM
CarlosHenriquedaSilva Prof.AdautoCarvalho SerraTalhada PE
LetciaNunesdeOliveira InstitutoDomBarreto Teresina PI
MarianaCamylaDuartePontes FariasBritoAldeota Fortaleza CE
AlissondeSousaBarreto ArideSCavalcante Caucaia CE
ClintonHenryColaoConegundes InstitutoDomBarreto Teresina PI
FlvioLuisSchneiderJnior MilitardeManaus Manaus AM
YuriJernimoMoreira FariasBritoCentral Fortaleza CE
AndrLopesEvangelistaDias E.P.MadreMariaVillac Teresina PI
BrunoLimaverdeVillarLbo FariasBrito Fortaleza CE
EduardoFernandesBaima ArideSCavalcante Fortaleza CE
FranciscoRodriguesdaCruzJnior InstitutoDomBarreto Teresina PI
NicolasKemerichdeMoura Objetivo Palmas TO
BrunodeOliveiraLima Col.AplicaodaUFPE Recife PE
GabrielMathewsVianaPinheiro Master Fortaleza CE
GuilhermeSerraBaima MaristaAraagy SoLuis MA
JayaneCarvalhoBorges Lettera Teresina PI
LaioLadislauLopesLima FariasBrito Fortaleza CE
LaraMulatoLima ArideSCavalcante Fortaleza CE
MatheusBragaFurstemberger NossaSra.dasNeves Parnamirim RN
RafaelRibeiroAlves Motiva CampinaGrande PB
MENOHONROSA
MateusJucaPinheiro 7deSetembro Fortaleza CE
ViniciusLopesBraga InstitutoDomBarreto Teresina PI
AlineTavoradaSilva Col.AplicaodaUFPE Recife PE
GuilhermePatriotaSampaio SantaMaria Recife PE
TiagoVianaeSousa MadreMariaVillac Teresina PI
LeticiaLauraNobreNunesSantos Col.Marista SoLus MA
MarcoAntonioCostaNascimento EscolaLatoSensu Manaus AM
RenanLucasdaSilvaCustdio ArideSCavalcante Fortaleza CE
Resultados
| 15
Olimpada Brasileira
de Qumica 2011
Ano Internacional de Qumica
TadeuMenesesdeCarvalho InstitutoDomBarreto Teresina PI
AdinaildoGomesPaesJunior SistemaElitedeEnsino Barcarena PA
ClarianoPiresdeOliveiraNeto Educator SoLus MA
IagoAlmeidaNeves Anchieta Salvador BA
JosRibamarPereiraNeto InstitutoDomBarreto Teresina PI
PedroVictorBarbosaNolto InstitutoDomBarreto Teresina PI
VitriaNunesMedeiros FariasBrito Fortaleza CE
VitorJucPolicarpo 7deSetembro Fortaleza CE
AlissonBezerraGomes IFBA LaurodeFreitas BA
AnaRaquelFerreiradeAzevedo FariasBrito Sobral CE
FilipeHersonCarneiroRios FariasBrito Sobral Ce
LvioMoreiraRios Crescimento SoLuis MA
JooPedroCavalcantePereira SantaMaria Recife PE
AntenorTeixeiraNeto ColegioAnchieta Salvador BA
RaissaNiutaFreitasdeOliveira 7deSetembro Fortaleza CE
ErikaRodriguesVieiradeMacdo IFPE Paulista PE
RmulloRandellMacedoCarvalho Lavoisier Teresina PI
SlvioFurtadoXimenes DAULIABringel Fortaleza CE
AnaFlviaGalvoLopes InstitutoDomBarreto Teresina PI
MatheusFernandoCarvalhoLopes InstitutoDomBarreto Teresina PI
BiancaRohsnerBezerra FariasBrito Fortaleza CE
CarolinaCarvalhoTavares EscolaLatoSensu Manaus AM
RayssaLimadosSantos SantoAntniodeJesus S
to
Ant.deJesus BA
JosVictorMachadoNascimento ArideS Fortaleza CE
LuisFelipeFonsecaDias GeoTamb JooPessoa PB
WladimirJosLopesMartins Motivo Recife PE
JulianaMoysesPoletti Motiva JooPessoa PB
WeiTzonChangColares LatoSensu Manaus AM
MariaEduardaKounarisFuziki SartreCoc Salvador BA
FernandoAntnioSaraivaMaia DuliaBringel Fortaleza CE
PatriciaTravassosCutrim ReinoInfantil SoLus MA
LydiaPearcePessoadeAguiar InstitutoDomBarreto Teresina PI
Resultados
| 17 16 |
Programa Nacional
Olimpadas de Qumica
Ano Internacional de Qumica
AndrezaSaboiaDantas CinciasAplicadas Natal RN
WilsonVieiradaSilvaJnior DomBarreto Teresina PI
DouglasMartinsCarneiro A.E.ProfaNoronha DomPedro MA
HaroldoNogueiraVictorianoNeto DuliaBringel Fortaleza CE
JosMarcelinodeSouzaNetto SantaMaria Timbaba PE
AntonioGuilhermeC.SilvaFeitosa AntoineLavoisier Timon PI
VictorotvioAndradedasNeves Olimpo Palmas TO
OslioCndidoArajoLimeiraLima FariasBrito Fortaleza CE
YanPontajaMedeirosdaSilva SantaRosa Belm PA
JooPedroSantosWanderley Motivo Recife PE
VictorSantosdeAndrade InstitutoDomBarreto Teresina PI
RodrigoMedeirosGuercio Motiva JooPessoa PB
KsiaPriscillaOhenaCardoso IFAL Maceio AL
JooPedroAlexandreSilvaMota DuliaBringel Fortaleza CE
MarianaFeitosaCustdio CEPRON SoLus MA
DiogoFarkattKabbaz GenesedeEnsino Recife PE
LucasBritoMaynart Salesiano Aracaju SE
JosLucasdeAlencarSaraiva Motivo Timbaba PE
GabrielF.P.Arajo Lavoisier Teresina PI
RennanMartinsViana E.P.MadreMariaVillac Teresina PI
MatheusSalmitoRodriguesPontes FariasBrito Fortaleza CE
TherezaHelenaAzevedoSilva Salesiano Aracaju SE
ArturLeiteRSaldanha Contato Contato AL
JanilsondaCostaBarros Fund.NokiadeEnsino Manaus AM
NathrciaCastroMota Master Fortaleza CE
JandersonSousaFerreira A.E.ProfaNoronha DomPedro MA
YgorRodrigoMeloFontesSantos COESI Aracaju SE
YanMouraQuintino IFAL Maceio AL
ThalineAlmeidaMatosViana Educator SoLus MA
RamonSantosGdaSilva Ideal Belm PA
MariaGabrielaVianadeSa Motiva CampinaGrande PB
JooPedrodeCarvalhoMagalhes GGE Recife PE
Resultados
| 17
Olimpada Brasileira
de Qumica 2011
Ano Internacional de Qumica
DanielAbrantesFormiga Motiva JooPessoa PB
SaymondoLivramentoSantos Elite Belm PA
FilipeMouroLeite InstitutoDomBarreto Teresina PI
PedroHenriqueA.Fraiman CinciasAplicadas Natal RN
VirnaMendonaSampaioLima Anchieta Salvador BA
LeoMeiraVital Motiva CampinaGrande PB
CarlosEduardoGrivoljnior Anchieta Salvador BA
ricaReginaRodriguesdaSilva CEPRON SoLus MA
JosIvanF.deO.Neto CentrodeEnsinoPleno Belm PA
ElisAragoMagalhaes Contato Caruaru PE
carodeAlmeidaVaro InstitutoDomBarreto Teresina PI
NellyCarmenRamirezCanelo IF-AM Manaus AM
NaiaradeOliveiraBaptista MasterBezerra Fortaleza CE
IsabelleRodriguesdeM.Cmara GGE Recife PE
AnaMrciaAzevedodeSousa CEI Natal RN
JorgeJuniorPedrosoJordo LatoSensu Manaus AM
CaioEddiedeMeloAlves LatoSensu Manaus AM
JulianyPiresFigueredo LatoSenso Manaus AM
GabrielWagnerSalesCavalcante Motiva CampinaGrande PB
JosSantanadaSilvaJnior EREMLuzAlvesSilva S
ta
CruzCapibaribe PE
JairadeSousaArajo AntoineLavoisier Teresina PI
MarcoAntonioS.CavalcanteFilho InstitutoDomBarreto Teresina PI
MatheusAugustoArajoCastro SalesianoSoJos Natal RN
WilsonCoelhoMendes InstitutoDomBarreto Teresina PI
NicolasEduardodaFonsecaFarias MotivaJooPessoa JooPessoa PB
AmandadeAlmeidaeSilva Ideal Aracaju SE
JssicaSilvaLopes FariasBrito Fortaleza CE
PedroFerreiraTorres Col.GrupoEduc.Ideal Belm PA
AlissonRibeiroLucena Motiva CampinaGrande PB
AluisioPereiradaSilvaFilho GGE Recife PE
MessiasBezerradaSilvaNeto GeoTamb JooPessoa PB
DanielleOliveiradeSousa Ideal Belm PA
Resultados
| 19 18 |
Programa Nacional
Olimpadas de Qumica
Ano Internacional de Qumica
BrendaDiasMarques CEPRON SoLus MA
EduardaKarynneSouza EREMLuzAlvesSilva S
ta
CruzCapibaribe PE
LeandroGomesSantanadeSouza Anchieta Salvador BA
AddaMorganaAlvesAlves SantaRosa Belm PA
MarinaMeloSousaMendesLeal SagradoCoraodeJesus Teresina PI
MatheusDantasdeLucena Motiva CampinaGrande PB
CaiqueCastroSouza ClasseA PortoVelho RO
MarcelHenriqueSilvaMoraes SARTRECOC Salvador BA
JulianaSilvaBrasil Master Fortaleza CE
DiogoSilvaSantana COCImperatriz Imperatriz MA
KalilLimaJardimFerraz SantaEmlia Olinda PE
RafaelMouraAndrade AngloLavoisier Teresina PI
HeitorCaetanodosSantos InstitutoBrasil Parnamirim RN
PauloRobertoOliveiraMartins EscolaLatoSensu Manaus AM
VictorRipardoSiqueira LatoSensu Manaus AM
AlexMendesLeonelFreire Contato Macei AL
GeorgysonDiasGondimNeo MilitardeManaus Manaus AM
MatheusLealdeSouza AntnioVieira Salvador BA
MarcosFelyppeOliveiraCastro MAC Castanhal PA
DiegoBarrosAlbuquerque LatoSenso Manaus AM
IrmaCsasznik LatoSensu Manaus AM
FelipeSouzadeAndrade GGE Camaragibe PE
JosValnirTeixeiraCruz SagradoCoraodeJesus Teresina PI
RaphaelSouzadeAlmeida Alternativo FreiPaulo SE
BrbaraBeatrizdeAlmeidaGama IFBA Salvador BA
DanielSantosRochaSobralFilho IdealMilitar Belm PA
ThiagoAugustoDantasVilaNova Inst.MariaAuxiliadora Natal RN
CaioCsarMeloDelgado GeoTamb JooPessoa PB
AnaBrbaradeJesusChaves Arquidiocesano Aracaju SE
LucasMedeirosSobrinhodeSousa IFBA Salvador BA
JooVictorBulamarquiCoelho Crescimento SoLuis MA
VictorGasparSilvaeSilva Col.Marista SoLus MA
Resultados
| 19
Olimpada Brasileira
de Qumica 2011
Ano Internacional de Qumica
DborahNbregadeFarias Motiva JooPessoa PB
ErikLeitedeAlmeida Santaursula Maceio AL
MatiasDanielF.Batista SantaUrsula Maceio AL
EzauSilvaRibeiro SistemaElitedeEnsino Barcareno PA
KlausAntonTyrrasch Contato Maceio AL
MarcusDiFabianniF.LopesFilho MilitardeManaus Manaus AM
GuilhermeHenriqueM.deAraujo Crescimento SoLus MA
ElizaEdneideO.SouzadeAlmeida ImaculadaConceio CampinaGrande PB
RuthEllenFdeCastroDantas CEI Natal RN
LuizFernandodaSilvaCostaSantos Saint-Louis Aracaju SE
DaniloPequeno Motiva Queimadas PB
EmillyRennaleFreitasdeMelo Motiva CampinaGrande PB
EleodrioSalesBonfmNeto Motiva Areia PB
GabrielCsarPereira Nsa.SradaConceio Craibas AL
GustavoHenriqueM.FerreiraFilho Contato Maceio AL
HenriqueSantosdeAlmeida MilitardeManaus Manaus AM
RafaelaGesMachado Anchieta Salvador BA
BrbaraOliveiradeAndrade InstitutoDomBarreto Teresina PI
AnaliceCardosodeBrito TeotonioFerreiraBrando CocaldosAlves PI
SarahPereiraMartins Esc.Crescimento SoLus MA
PatriciadosSantosLimaDutra Motiva JooPessoa PB
VictorPeresdeMeloGoulart MilitardeManaus Manaus AM
CarlaLorenaSilvaCardoso SistemaEliteVila Barcarena PA
CralosEduardoAdrianoFilho Mar Castanhal PA
PaulaCarollineCostadeSantAna Salvador Aracaju SE
JosMatheusSantosPereira IFBA Salvador BA
AlandeAndradeMonteiroNeto Estadual Recife PE
Resultados
| 21 20 |
Programa Nacional
Olimpadas de Qumica
Ano Internacional de Qumica
IVOlimpadaBrasileiradeQumicaJnior
FaseI
8e9anosdoEnsinoFundamental
Exameaplicadoem27.08.1011
I NS T R U E S
1.Aprovaconstade20questesobjetivas,cadaumacontendoquatroalterna-
tivas,dasquaisapenasumadeveserassinalada.
2.Aprovatemduraode3horas
3.Vocreceberogabaritoaps1horadoinciodaprova,pararegistrarassuas
opesderespostas.
01Umapginadainternetcontmaseguinteafrmao:Opetrleouml-
quidoescuro,deaspectooleoso,menosdensoqueagua.Deacordocomas
caractersticasdesseproduto,essaafrmativa:
A)correta,poiselemaislevedevidoserumasubstnciasimples.
B)incorreta,porqueopetrleoumamisturademuitosconstituintes,portanto
maisdensoqueagua.
C)correta,poistemperaturaambienteopetrleopermanecesempresobrea
superfciedagua.
D)incorreta,poisopetrleoumasubstnciamaisdensadoqueagua,doceousalgada.
02 Um tipo de petrleo bruto extrado do subsolo misturado com gua sal-
gada, areia e argila. Por isso, utilizam-se dois processos distintos (I e II) para a
separaodessesmateriais.Inicialmente,atravsdoprocessoI,separa-seagua
salgada.Depois,utilizando-seoprocessoII,retiram-seaareiaeaargila.Seguin-
doessasequncia,qualdasalternativasabaixotrazumaindicaotecnicamente
maisadequadaparaosprocessosIeII,respectivamente?
A)Decantaoefltrao. B)Destilaoefuso.
C)Filtraoeevaporao. D)Fusoedestilao.
03 Em um vdeo disponibilizado no YouTube, um recipiente de vidro fechado
quecontmalgunscristaisdeiodo(I
2
)foiaquecido.Comisso,devidotransio
doestadoslidoparaogasoso,verifcou-seaformaodeumanuvemdecolo-
raovioletaemseuinterior.
Adaptadodehttp://www.youtube.com/user/VideosdeQuimicaUFFAcessoem31/07/2011
IV OBQ Jnior
| 21
Olimpada Brasileira
de Qumica 2011
Ano Internacional de Qumica
Nessademonstrao,ocorrea
A)destilaodeumasubstnciacovalente.B)vaporizaodeumasubstnciainica.
C)sublimaodeumasubstnciacovalente.D)recristalizaodeumasubstnciainica.
04Umadasetapasparaprocessamentodomeldeformahiginicaeseguraen-
volveousodeumequipamentoque,atravsdomovimentoderotaoemtorno
deseuprprioeixo,retiraessealimentodosalvolosexistentesnacolmeia.No
comrcio, esse tipo de equipamento encontrado com vrias capacidades de
extrao, podendo ser com sistema de rotao acionado manualmente ou por
eletricidade,commotoredispositivosdecontroledevelocidadederotao.
Adaptadodehttp://sistemasdeproducao.cnptia.embrapa.br/FontesHTML/Mel/SPMel/extracaomel.htm
Acessoem31/07/2011
Quetipodeprocessodeseparaorealizadonesseequipamento?
A)Centrifugao.B)Cristalizao.C)Flotao.D)Peneirao.
05Osorofsiolgicovendidoemfarmciasumprodutoquecontm0,9%,em
massa,deNaClemguadestilada.Esseprodutoconsideradouma
A)misturaheterognea. B)soluocontendoons.
C)soluosaturada. D)substnciacomposta.
06Umachaleiracontmapenasguamineralefoiaquecidaemumfogo.Aps
umcurtoperodo,observou-seumaforteproduodevapor,comoumacon-
sequnciada(o)
A)diminuiodasinteraesentreasmolculasdagua.
B)desejodostomospresentesnaguaemdoarourecebereltrons.
C)rompimentodasligaescovalentesduranteamudanadeestadofsicoda
gua.
D)decomposiodasmolculasdeguaparaproduodehidrognioeoxig-
niogasosos.
07Emumaatividadedelaboratriotransferiu-seumpoucodeacardemesa
(sacarose) para um bquer. Em seguida, agitou-se com um basto de vidro e,
rapidamente,observou-sequeolquidofcoutransparenteelmpido.Sobreesse
processo,foramfeitastrsafrmativas,conformemostradoabaixo.
I.Oacardissolvidopodeserrecuperadoatravsdeummtododeseparao.
II.Oacarreagiucomaguaeformouumasoluo,masasuadouraconti-
nuouaexistir.
III.Oacarreagiucomaguaedeixoudeexistir,ouseja,transformou-seem
outrasubstncia.
Exames - Fase I
| 23 22 |
Programa Nacional
Olimpadas de Qumica
Ano Internacional de Qumica
Qual(is)dessa(s)afrmativa(s)est(o)CORRETA(S)?
A)I,apenas.B)II,apenas.C)IIeIII,apenas.D)IeIII,apenas.
08Omendelvio(Z=101;confguraoeletrnica:[Rn]5f
13
7s
2
)umelemento
qumicoidentifcadoem1955,mesmoapstersidoproduzidoemquantidade
insufcienteparaserobservadoaolhonu.Oseunomefoiumahomenagemao
qumicorussoDmitriMendeleev.OMdpossui
A)101prtonse101eltrons.
B)menoseltronsdoqueoradnio(Rn).
C)confguraoeletrnicaidnticaaosdemaiselementosqumicosartifciais.
D)menosprtonsdoqueoelementoqumiconaturalcommaiornmeroatmico.
09Diferentespropostasdidticastrazemverseseletrnicasparaatabelaperi-
dica,conformeofragmentomostradoaolado,quefoiretiradodeumadelas.
Nessefragmentohdiferentesinformaessobreoalumnio,taiscomo:nmero
atmico, massa atmica, confgurao eletrnica, fonte natural e utilizao na
formaelementaroucomoliga.ApartirdessasinformaesCORRETOafrmar
que
A)amassaatmicadoalumnioiguala13.
B)panelasdescartadassoasprincipaisfontesparaa
reciclagemdoalumnio.
C) o alumnio na forma de uma substncia simples,
elementar,chamadodebauxitaeusadoemfogos
deartifcio.
D)Alocalizaodoalumniono3perododatabela
peridicatemrelaocomasuaconfguraoele-
trnica.
http://www.abiquim.org.br/tabelaperiodica/tabela_est.asp
Acessoem31/07/2011
10Nadcadade1920,independentementeErnestRutherford(1871-1937),na
Inglaterra,WilliamDraperHarkins(1873-1951),nosEUA,eOrmeMasson(1858
1937),naAustrlia,propuseramapossvelexistnciadeumapartculaatmica
semcarga.Porm,apenasem1932,naInglaterra,JamesChadwick(1891-1974)
comprovou a existncia do ___(I)___. Esse processo exemplifca que a cincia
umaatividade______(II)________.
IV OBQ Jnior
| 23
Olimpada Brasileira
de Qumica 2011
Ano Internacional de Qumica
Aslacunas(I)e(II)notextoacimapodemsercompletadasdeformaCORRETAe
namesmasequnciapelaopo
A)Ieltron;II-prpriadosexomasculino.
B)Inutron;II-individual.
C)Iprton;II-extensivaapesquisadoresdevriospases.
D)Inutron;II-humana.
11OfsforotemZ=15etodofsforoencontradonanatureza
31
P.Apesardis-
so,oscompostosdofsforo-32(P-32)somuitoempregadoscomomarcadores
radioativos,paracompreenderosciclosvitaisdeplantaseanimais,ondehajaa
participaodecompostosquecontmfsforodemassa31.
AdaptadodePEIXOTO,E.A.M.Fsforo.QumicaNovanaEscola,15,51,2002.
Diantedessasconsideraes, CORRETOafrmarqueofsforoencontradona
naturezaeofsforoutilizadocomomarcadorradioativoso
A)elementosqumicosdiferentes. B)isbaros.
C)istopos. D)substnciasnaturais.
12 Em uma atividade experimental realizada na sala de aula, uma professora
transferiuumapequenaquantidadedeumlcool,slidotemperaturaambien-
te,paraumbquerquecontinhaguadestilada.Aps1hora,verifcou-sequeo
lcoolnosedissolveu.Emrelaoaoquefoiobservadoduranteesseperodo,
CORRETOafrmarque
A)esselcooloetanol.
B)olcoolsedissolveriacasoaguadestiladaestivessemuitogelada.
C)adiferenadedensidadeentreasduassubstnciasimpediuadissoluodolcool.
D) o sistema formado entre esse lcool e a gua um exemplo uma mistura
heterognea
13Oquadroabaixotrazumarelaoentrecomponentes,caractersticaseusos
dealgumasligasmetlicas.
Ligametlica Componentes Caracterstica Exemplodeuso
I Ferroecarbono Resistnciacorroso Utensliosdomsticos
Amlgama Mercrio,prataeestanho Resistnciamecnica II
Ouro18quilates Ouroecobre III Jias
Exames - Fase I
| 25 24 |
Programa Nacional
Olimpadas de Qumica
Ano Internacional de Qumica
Paraatenderaosdadosfornecidosnoquadroanterior,aslacunasI,IIeIIIpodem
sercompletadasdeformaCORRETAenamesmasequnciapelaopo
A)soldaindustrial;ferramentasemoedas;ebrilhointenso.
B)ao;restauraodedentes;ealtaductibilidadeemaleabilidade.
C)bronze;soldasdeprodutoseletrnicos;ebaixacondutividadeeltrica.
D)lato;plsticosbiodegradveisparasacolas;altacondutividadetrmica.
14Aaobenfcadooznioaoserhumanobemconhecida.Masseuefeito
nocivo ou favorvel vida na Terra depende da altitude em que ele se situa. A
maiorpartedessegsestnaestratosfera(entre13e40km),ondefuncionacomo
escudoradiaoultravioleta.Maisprximodasuperfcie,porm,oO
3
umpo-
luentequecausadanosaotecidopulmonardosanimaiseprejudicaavegetao.
Natroposfera,ooznio(O
3
)formadoquandomolculasdeoxignio(O
2
)so
divididaspelaluzsolareostomoslivresdesseelementoseligamaoutrasmol-
culasdeoxignio.PrximodaTerra,ooznioformadopelareaodooxignio
compoluentesurbanos.
http://cienciahoje.uol.com.br/noticias/meteorologia/acao-de-relampagos-modifca-quimica-da-
atmosferaAcessoem31/07/2011
Aformaodooznionaatmosferaenvolveuma
A)divisodetomosdeoxigniopelaluzsolar.
B)quebradaligaoinicadogsoxignionatroposfera.
C)reaoentreoO
2
,poluenteetomoslivredoOembaixasaltitudes.
D)reaonaqualhaparticipaodasuaformaalotrpicaemaltitudesentre
13e40km.
15Osdiferentesaspectosaseremconsideradosnaconcepodeumprocesso
paracumprirosprincpioseobterumasnteseverdeenvolvemdiferentespar-
metroscomo,porexemplo,
A)apotencializaodaincorporaodosreagentesnoproduto
B)ousodematrias-primasdefontesrenovveis,comoopetrleo.
C)ageraodecompostosdegradveisqueliberemmetaispesados.
D)ainclusodecompostostxicoscomoreagentes,paraestimularasuaretirada
docomrcio.
IV OBQ Jnior
| 25
Olimpada Brasileira
de Qumica 2011
Ano Internacional de Qumica
16Observeachargemostradaabaixo.
http://interagindoquimica.blogspot.com/2011/02/charge-quimica.html
Acessoem03/08/2011
Corrigindogramticaequimicamenteostextosdessacharge,trstiposdeme-
taisseriam
A)carbono,potssioezinco.B)prata,titnioecobre.
C)sdio,cloroemercrio.D)urnio,hlioecromo
17Nospalitosdefsforoqueconhecemos,nohpresenadoelementofs-
foro;elessoencontradosnapartesperadacaixa.Napontadopalito(aparte
vermelha)nstemoscloratodepotssio(KClO
3
),responsvelporliberaroxig-
nioparamanterachamaacesa,eaoutrapartedopalitorevestidaporuma
camada de parafna (mistura de hidrocarbonetos). Na caixa, temos sulfeto de
antimnio, Sb
2
S
3
, e trixido de ferro, Fe
2
O
3
, para gerar atrito, e o fsforo, para
produzircalorintenso.Quandoriscamosopalitonacaixaproduzimosumafasca
queemcontatocomocloratodepotssioliberamuitooxignio(O
2
)quereage
comaparafnagerandoumachamaqueconsomeopalitodemadeira.
http://www.infoescola.com/elementos-quimicos/fosforo/(Modifcado)Acessoem31/07/2011
Oacendimentodeumpalitodefsforopeloatritonacaixinhadefsforoenvolve
A)umaformaalotrpicadofsforoexistentenapontadopalito.
B)aliberaodeumasubstnciainicaquereagecomaparafna.
C)reaesenvolvendoaliberaoeconsumodeumasubstnciasimples.
D)umatransformaoqumicacausadapelosxidos,substnciassimples,pre-
sentesnascaixas.
18 O conceito de ecomaterial passou a ser disseminado, a partir do incio da
dcadade1990,comoumarespostafrentenecessidadedenovastecnologias
para a produo de materiais ambientalmente adequados. Considerando essa
Exames - Fase I
| 27 26 |
Programa Nacional
Olimpadas de Qumica
Ano Internacional de Qumica
caractersticaeosprincpiosdaqumicaverde,qualdasalternativasabaixotraz
umexemplodeecomaterialparaaproteoambiental?
A)Chumbodebateriadecarrosparaadescoloraodeefuentes.
B)Clulassolaresproduzidasapartirdebateriasdetelefonesrecicladas.
C)Cermicascondutorasdeenergiaproduzidasapartirdecarvovegetal.
D) Carbono obtido a partir do p de casca de coco para uso na remoo de
corantes.
19Acaractersticaintrinsecamenteinterdisciplinardetemascomoamicroeletr-
nica,vidrosespeciais,plsticoseaosdealtodesempenhoedasatuaispinturas
automobilsticas,contriburamparaaformaodeumanovareanaQumica.
Ela envolve qumicos orgnicos, inorgnicos, fsico-qumicos e tambm fsicos,
engenheiroseoutrosprofssionaisligadosaosdepartamentosdasuniversidades,
dos institutos de pesquisas e das indstrias. De acordo com as caractersticas
apontadas,essareaconhecidacomo
A)Biotecnologia.B)Eletroqumica.C)QumicadeMateriais.D)Radioqumica.
20 Tambm originada da Qumica clssica, a Qumica Ambiental atualmente
umacinciainterdisciplinareumdosseusobjetivos
A)desenvolverpesquisasquecomprovemainfunciahumananosrecentesde-
sastresnaturais.
B)aumentarasemissesgasosasindustriaisparacontrolarosfenmenosnatu-
raisatmosfricos.
C)melhorarosprocessosparaqueahumanidaderetorneaummododevida
maisrsticoeartesanal.
D)estudarprocessosqumicosqueocorremnanatureza,tantonaturaisquanto
oscausadospelohomem.
GABARITO-QUESTESDEMLTIPLAESCOLHA
1 2 3 4 5 6 7 8 9 10
C A C A B A A A D D

11 12 13 14 15 16 17 18 19 20
C D B C A B C D C D
IV OBQ Jnior
| 27
Olimpada Brasileira
de Qumica 2011
Ano Internacional de Qumica
Exames - Fase II
IVOlimpadaBrasileiradeQumicaJnior-FaseII
Exameaplicadoem01.10.1011
QUESTESDEMLTIPLAESCOLHA
01Osmoradoresdeumbairroorganizaramumacampanhasobrecoletase-
letivadelixoereciclagemdemateriais.Dentreasaesrealizadas,elescon-
seguiram e distriburam conjuntos de lixeiras de cores diferentes. Em cada
umadelasdeveriamserdepositadosprodutoseobjetosdeacordocomos
componentesdosseusrespectivosmateriais:
Amarela-Metal;Azul-Papel;VermelhaPlstico;Verde-Vidro.
Caso seja seguida essa orientao, a maior probabilidade de se encontrar
objetos que ao serem reciclados forneam altas porcentagens de alguma
substnciaelementarestnalixeiradecor
A)amarela. B)azul. C)verde. D)vermelha.
02 Observe a charge mostrada ao
lado.Elafazumacrtica
A)snteseverdedeumasubstncia
gasosaapartirdaguapoluda.
B)etapapoluentedoprincipalpro-
cessodaindustrializaodeO
2
a
partirdefontesnaturais.
C)despoluiodegrandesreserva-
trios naturais com a utilizao
deoxigniocomercial.
D)formadepoluioquecompro-
meteadissoluodeumaimpor-
tantesubstnciaapolarnagua.
Acessoem31/07/2011
03Umexperimentointerativomontadoemummuseudecinciastraziaum
circuitoeltricocontendoumasireneconectada,atravsdefosdecobre,a
umabateriaeadoiseletrodos.Nomomentoemqueoseletrodoseramco-
locadosdentrodealgumassolues,ocircuitoerafechado,haviapassagem
decorrente,easireneeraacionada.
| 29 28 |
Programa Nacional
Olimpadas de Qumica
Ano Internacional de Qumica
Qualdassoluesabaixopossuimaiorprobabilidadedeacionaressasirene
demodomaisforteaoseremcolocadasemcontatocomoseletrodosdesse
experimento?
A)Soluodesacarose.B)Soluodeetanola50%.
C)Soluoconcentradadecloretodesdio.D)Soluodecidoactico
a4%(vinagre).
04 A regra do octeto uma das formas mais utilizadas para explicar as li-
gaes qumicas entre tomos de um mesmo elemento ou de diferentes
elementosqumicos.Porm,halgunscompostosquesoexceesaessa
regra,mostrandoassuaslimitaesenquantoummodelonicoparaprever
as valncias e as frmulas de todos os compostos. Abaixo, associe as duas
colunas, relacionando alguns tipos de exceo regra do octeto aos seus
exemploscorrespondentes.
(I)onsdemetaisdetransio ()PCl
5
,I
3
-

(II)Compostosdegasesnobres ()Fe
3+
,Cu
2+

(III) Molculas e ons contendo tomos com mais de oito el-


trons
()XeO
3
,KrCl
2

AsequnciaCORRETAdessaassociao
A)I,II,IIIB)II,III,IC)II,I,IIID)III,I,II
05Tungstnioeargniosoutilizados,respectivamente,comoconstituintesdo
A)flamentoedogsinertedelmpadasincandescentes.
B)recipienteplsticoedocomprimidoefervescenteusadocontraazia.
C)materialmetlicoedogscombustveldecilindrosusadosnasoldagem
depeasdeferro.
D)vidroedolquidodetermmetrosusadosemresidncias,paraverifcara
temperaturaambiente.
IV OBQ Jnior
| 29
Olimpada Brasileira
de Qumica 2011
Ano Internacional de Qumica
Exames - Fase II
06
Acessoemhttp://www.profpc.com.br/Tirinhas%20de%20Qu%C3%ADmica.htm
Acessoem31/07/2011
Emboraacenaretratadanatirinhaacimanosejacitadanosrelatoshist-
ricos sobre Dmitri Ivanovich Mendeleev (1834-1907), com um humor, ela
envolveumadasestratgiasutilizadasnaproposiodatabelaperidicapor
essequmicorusso.Elecriouumacartaparacadaumdos63elementosco-
nhecidosataquelemomentoeasorganizouporordem
A)cronolgicadaidentifcaodecadaelementoqumicoeagrupando-as
aoacaso.
B)crescentedemassasatmicaseagrupando-asemelementosdeproprie-
dadessemelhantes.
C)decrescentedenmerosatmicoseagrupando-asdeacordocomasle-
trasdosseussmbolos.
D)alfabticadonomedoelementoqumicoeagrupando-aspelasemelhan-
adosseuspontosdefuso.
07Onitrogniopodeestarpresentenaguasobvriasformas:molecular,
amnia(NH
3
),nitrito(NO
2
-
),nitrato(NO
3
-
);umelementoindispensvelao
crescimento de algas, mas, em excesso, pode ocasionar um exagerado de-
senvolvimentodessesorganismos,fenmenochamadodeeutrofzao.So
causasdoaumentodonitrognionagua:esgotosdomsticoseindustriais,
fertilizantes,excrementosdeanimais.
Adaptadodehttp://www.ufv.br/dea/lqa/qualidade.htmAcessoem31/07/2011
| 31 30 |
Programa Nacional
Olimpadas de Qumica
Ano Internacional de Qumica
Segundoasinformaescontidasnotextoacima:
A)apresenadeN
2
naguadecorrentedoprocessodeeutrofzao.
B)substnciasmoleculareseinicasdenitrogniosoelementosqumicos
indispensveisaocrescimentodealgas.
C)olanamentodeexcrementosdeanimaisnaguapodelevaraumexces-
sodaproduodaformamoleculardonitrognio,NO
2
.
D)esgotosdomsticoseindustriaisefertilizantespodemaumentaraquan-
tidadedeNH
3
,NO
2
-
eNO
3
-
naguaecontribuirparaumexageradode-
senvolvimentodasalgas.
08Observeasinformaessobreoselementosqumicosindicadosabaixo.

Adaptadodehttp://www.abiquim.org.br/tabelaperiodica/tabela_est.aspAcessoem31/07/2011
Deacordocomdadosapresentados,CORRETOafrmarque
A)aoganharumeltronumtomodeforsetornaumtomodenenio.
B)aoganharumeltronumtomodesdiosetornaumtomodenenio.
C)aformaodocompostoNaFenergeticamentemaisfavorveldoqueadoNeF.
D)os trs elementos qumicos indicados pertencem ao mesmo perodo da
tabelaperidica.
09Duranteumavisitaorganizadaporumaescolaaumlaboratriodeuma
universidade, os alunos realizaram uma anlise de um reagente qumico.
Eles pulverizaram uma pequena quantidade do reagente em um almofa-
rizetransferiramumaamostraparadentrodeumpequenoeestreitotubo
de vidro, fechado em uma das extremidades. O reagente foi compactado
atserobtidaumacamadadeaproximadamente1cmdealtura.Depois,o
IV OBQ Jnior
| 31
Olimpada Brasileira
de Qumica 2011
Ano Internacional de Qumica
Exames - Fase II
tubofoipresoaumtermmetro,deixando-seapartecontendoaamostra
junto ao bulbo. Em seguida, mergulhou-se esse sistema em um recipiente
contendoleomineral,quefoisendoaquecido.Passadosalgunsinstantes,
percebeu-seoaparecimentodealgumasgotculasnointeriordotubodevi-
droeobservou-seatemperaturaemelevao.Somentedepoisde30(trinta)
minutosdessaprimeiraobservaotodoreagentefcoulquido.Esseexperi-
mentopermitiuverifcara
A)ebuliodoreagente.B)ebuliodeumasubstnciapura
C)fusodeumamistura.D)fusodeumasubstnciapura.
10Emumaatividadeexperimentalforamtransferidosparadoisbqueresdis-
tintos,AeBummesmovolumedegua(d=1,0g/mL;p.e.=100
o
C),500mL.
Depois,adicionou-se500mLdeetanol,CH
3
CH
2
OH(d=0,789g/mL;p.e.=78,4
o
C)aobquerAe500mLdeclorofrmio,CHCl
3
(d=1,48g/mL;p.e.=61,2
o
C)
aobquerB.Apsagitao,umdossistemasfcoumonofsicoenquantoque
ooutrofcoubifsico.Sobreesseprocesso,CORRETOafrmarque
A)nobquerB,afaseinferioreragua.
B)nobquerA,afasesuperioreraoetanol.
C)possvelsepararcadacomponentedosistemadobquerAatravsde
evaporaodessasoluo.
D)possvelsepararasfasescomponentesdosistemadobquerButilizan-
do-seumfunildeseparao.
QUESTESANALTICOEXPOSITIVAS
11 O grafeno um material composto por
uma folha de grafte. Ele contm uma nica
camada de tomos de carbono (Z=6; Conf-
guraoeletrnica:1s
2
2s
2
2p
2
)organizadosde
forma hexagonal, como favos em uma col-
mia,conformeindicadonafguraaolado.O
grafenopromissorparaodesenvolvimento
de telas sensveis ao toque (touchscreens) e
clulas fotovoltaicas porque fexvel, trans-
parenteecondutordeeletricidade.
Adaptadodehttp://www1.folha.uol.com.br/ciencia/
Acessoem31/07/2011
Deacordocomascaractersticasepropriedadesapresentadaspelografeno,
CORRETOafrmarqueeleumaformaalotrpicadeumelementometli-
co?Justifqueasuaresposta.
| 33 32 |
Programa Nacional
Olimpadas de Qumica
Ano Internacional de Qumica
12Realizou-seumexperimentodesimulaoatmicaemumasaladeaula
para uma abordagem sobre a natureza microscpica da matria. Utilizan-
do-seumarguaplsticaeoutrosobjetosdebaixocusto,montou-seuma
balana. Aps serem colocadas 12
(doze) bolas de isopor em um dos
pratose1(uma)bolanooutroprato
da balana, o sistema permaneceu
emumasituaodeequilbrio,con-
formemostradoaolado.
Explique essa situao de equilbrio
baseando-seemumtipodemodelo
atmico.

Fonte:Gambo,J.A.;Corso,H.L.;Severno,M.H.
Qumicaatractivaenuningresoalauniversidad.Rev.EurekaEnse.Divul.Cien.,6,3,423-439,2009.
13Duranteumdeterminadoprocessoforamproduzidos88(oitentaeoito)
gramas de dixido de carbono. Sabendo que esse processo est represen-
tadopelaequaodareaoqumicaC+O
2

CO
2
indiquequantos
molsdooutrogsseronecessriosparaproduziressaquantidadedeCO
2
.
Justifqueasuaresposta.
Dadosdemassasatmicasdoselementosqumicos:C=12u;O=16u.
GABARITO
1 2 3 4 5 6 7 8 9 10
A D C D A B D C C D
Se um dia voc tiver que escolher entre o mundo e o amor, lembre-se: Se escolher o
mundofcarsemamor,massevocescolheroamor,comeleconquistaromundo.
ALBERTEINSTEIN
IV OBQ Jnior
| 33
Olimpada Brasileira
de Qumica 2011
Ano Internacional de Qumica
Questes resolvidas I
OBQJr-2011-RESPOSTASESCOLHIDAS
QUESTO11
ResoluodeTeresaVirgniaNevesFloriano-ColgioAmadeus,Aracaju-SE
No.Poisografenocompostoporapenasumanicacamadadetomosde
carbonoe,sendocompostoporcarbono,nopodeserconsideradoumele-
mentometlicojqueocarbononoummetal.Poressemotivo,nocor-
retoafrmarqueografenoumaformaalotrpicadeumelementometlico.
QUESTO12
ResoluodePedroHenriqueRochadeFreitas-ColgioMilitar,Braslia-DF

UtilizandoomodeloatmicodeDalton,podemosdizerqueasbolasdeisopor
representamoelementoqumicoX,commassaA.Ostomosdesseelemento
soesferasmaciaseindivisveis,assimcomoacontececomoutroselementos
qumicos,ecadaelementoapresentasuamassaprpria(segundoomodelo
atmicodeDalton).Jabolaqueestdooutroladodabalanarepresentao
elementoqumicoY,commassa12A.Comodeumladotemos12tomosde
X,edooutro1tomodeY,chegamosnaseguinteequaodasmassasdos
doiselementos:
12A
x
=1.12A
y
Comoessaequaoverdade,chegamosconclusodequeamassade12
tomosdeXigualmassade1tomodeY.
Portanto,pelomodeloatmicodeDalton,explicamosqueosdoispratosda
balanaestoemequilbrio,poisamassade1tomodeY12vezesmaior
queamassade1tomodeX.
| 35 34 |
Programa Nacional
Olimpadas de Qumica
Ano Internacional de Qumica
QUESTO13
ResoluodeVictriaFreitasVieiradaCunha,ColgioParaso-Juazeirodo
NorteCE

Sonecessrios32gdeO
2
paraproduzir44gdeCO
2
.Logo,seronecessrios
XgdeO
2
paraproduzir88gdeCO
2
talque:
O
2
CO
2
32g-44g1moldeO
2
-32g
xg-44gxmoldeO2-64g
x=64gx=2mol
Sonecessrios64gdeO
2
paraobter88gdeCO
2
,portantoseronecessrios
2molsdeO
2
.
Nuncaconsideresoestudocomoumaobrigao,mascomouma
oportunidadeparapenetrarnobeloemaravilhosomundodosaber.
AlbertEinstein
IV OBQ Jnior
| 35
Olimpada Brasileira
de Qumica 2011
Ano Internacional de Qumica
NOME ESCOLA CIDADE UF
O U R O
PedroHenriqueRochadeFreitas MilitardeBraslia Braslia DF
DboraLetciaNogueiradeOliveira Esc.ModelodeIguatu Iguatu CE
IsabelledeSousaPereira FariasBrito Fortaleza CE
MarcusVinciusdosSantosLima UNISUZ Suzano SP
MateusVasconcelosAlbuquerque SantaCeclia Fortaleza CE
VittriaNobreJacinto FariasBrito Fortaleza CE
P R AT A
ThallesFerreiradaPonte FariasBrito Sobral CE
GabrielDemetriusBertoldodaSilva Objetivo-Cantareira SoPaulo SP
CarlosGabrielOliveiraFreitas 7deSetembro Fortaleza CE
GeorgeHenriqueNunesdaM.Jnior FariasBrito Fortaleza CE
IagoDantasFigueirdo Anchieta Salvador BA
JuliaPracianoLopes FariasBrito Fortaleza CE
DanielPinheiroMotadaS.Ferreira AntnioVieira Salvador BA
ElcioKoodiroYoshida Etapa SoPaulo SP
ImanMusaIsmilAbdelR.Jadallah MilitardeBraslia Braslia DF
EduardoSerpa Master Fortaleza CE
FelipeDAmorimBarreto Anchieta Salvador BA
FernandodeMoraesRodrigues FariasBrito Fortaleza CE
GesaCostaOliveiradeM.Santana FariasBrito Fortaleza CE
LucianoPinheiroBatista FariasBrito Fortaleza CE
PriscilaTavaresVitoriano FariasBrito Fortaleza CE
RapahelFernandesLigrio MaristaDomSilvrio BeloHorizonte MG
B R O N Z E
AnaKarolineBorgesCarneiro FariasBrito Fortaleza CE
EricIanNoronhaJunqueira ArideSCavalcante Fortaleza CE
MariaPaulaTellezFrias Anchieta Salvador BA
MateusAragoEsmeraldo ArideSCavalcante Fortaleza CE
IdliaMariaBarbosadoNascimento Ass.Educ.Profa.Noronha DomPedro MA
VictorSousaSilva ArideSCavalcante Fortaleza CE
DanielleMayumiShiba PolciaMilitardoParan Curitiba PR
PedroSalazarCosta CentroEduc.AdalbertoValle Manaus AM
RobertoRebouasPratesFilho Anchieta Salvador BA
ThasSilvaSouza Fund.Itab.Dif.doEnsino-FIDE Itabira MG
Resultados
| 37 36 |
Programa Nacional
Olimpadas de Qumica
Ano Internacional de Qumica
GabrielPereiraPennaAndrade Fund.Itab.Dif.doEnsino-FIDE Itabira MG
GustavoOliveiraMartins DuliaBringel Fortaleza CE
LiaSantosPoncedeLeon DuliaBringel Fortaleza CE
LucasBastosOliveira DuliaBringel Fortaleza CE
AlexsanderFelipeAlencarJusta ArideSCavalcante Fortaleza CE
BrunaLuizaBragaPantoja 7deSetembro Fortaleza CE
DaviVieiraFerreira Paraso JuazeirodoNorte CE
EloSantiagodaSilvaPereira ObjetivoUnidadeAquarius S.J.dosCampos SP
JooPedroGrangeiroCarioca FariasBrito Fortaleza CE
JoelFerreiraMadureira FariasBrito Fortaleza CE
LaraPotiNobre FariasBrito Fortaleza CE
LiadeOliveiraDomingues FariasBrito Fortaleza CE
MatheusBernini PedroIIUEENII RiodeJaneiro RJ
MayaraMelodosSantos ArideSCavalcante Fortaleza CE
NarjaraSmyaRodriguesPereira Ass.Educ.Profa.Noronha DomPedro MA
TicianaAlencarNoronha FariasBrito Fortaleza CE
ME N S O H O N R O S A
JuanFreireDantasGalvo FariasBrito Fortaleza CE
PedroHenriqueS.deOliveira EtapaValinhos Valinhos SP
GabrieldeSousaCorreia FariasBrito Fortaleza CE
RhayssaIngridPereiraRangel ArideSCavalcante Fortaleza CE
VinciusRodriguesMascarenha CEV- Teresina PI
LuevertonGonalvesdosSantos GEOGarciaeBrito Mossor RN
MatheusCarisCastro FariasBrito Fortaleza CE
RodrigoCeccatodeFreitas GabrielChalupe Barueri SP
GabrielCortizoFerraz BoaViagem Recife PE
PedroMacedoFlores MilitardeJuizdeFora J.deFora-MG MG
GabrielGuedesFerreiradeSouza DuliaBringel Fortaleza CE
JoanaLailaVitalCarneiro FariasBrito Sobral CE
RicardoNbregaMachado PolciaMilitardoParan Curitiba PR
BrunoRobertVasconcellosOliveira Anchieta Salvador BA
CarlosArthurGrangeiroSampaio Paraso JuazeirodoNorte CE
DaniloCinsSantAnadeLima Anchieta Salvador BA
EullerReisFarias Ass.Educ.Profa.Noronha DomPedro MA
FelipeMartinsGomes Master Fortaleza CE
GabrielMoraesRamosStudart ArideSCavalcante Fortaleza CE
IV OBQ Jnior
| 37
Olimpada Brasileira
de Qumica 2011
Ano Internacional de Qumica
JulianaLeiteCampelo SantaCeclia Fortaleza CE
MauroclioRochaPontesFilho FariasBrito Sobral CE
SamuelPalciodeOliveira Paraso JuazeirodoNorte CE
GuilhermedeFreitasRodrigues ArideSCavalcante Fortaleza CE
JandersonB.daFonsecaJnior NotreDamedeLourdes Cuiab MT
LeonardoGregriodeAndrade MilitardeJuizdeFora J.deFora-MG MG
BrunoDaviRocha SantAna Sobral CE
LorayneLinoSousa Esc.JardimCrescimento SoLus MA
LucasFelipeAlbuquerqueLins ArideSCavalcante Fortaleza CE
MariaMarianaBarrosdeVasconcelos ArideSCavalcante Fortaleza CE
EliabeBastosDias 7deSetembro Fortaleza CE
LuanLincolnCunhaRibeiro MaterChristi Mossor RN
AnaLetciaGondimdeCarvalho FariasBrito Fortaleza CE
AnaLuizaVianaPequeno 7deSetembro Fortaleza CE
AndressadaSilvaFigueiredo FariasBrito Fortaleza CE
AntnioBruno.deHolandaLima CentroEduc.AdalbertoValle Manaus AM
ArturSoutoMartins ArideSCavalcante Fortaleza CE
FelcioHolandaMoreira FariasBrito Fortaleza CE
IsabellaMayumideSouzaMatsura Bandeirantes SoPaulo SP
LarissaFonsecaChaves Ass.Educ.Profa.Noronha DomPedro MA
PedroJorgeL.AlvesCronemberger SagradoCoraodeJesus Teresina PI
PedroSantosBarbosa COESI Aracaju SE
PedroSouzaFaria FariasBrito Fortaleza CE
RodrigoSilvadeAndrade SantoAntniodeJesus S.Ant.deJesus BA
ThiagoAssisBorgesMorais DomBarreto Teresina PI
VictorAlmeidaCosta FariasBrito Fortaleza CE
PauloVitorBarreiroGidi SartreCOC Salvador BA
ReneBezerraMoreira FariasBrito Fortaleza CE
FranciscoGrigoredeAmorimJnior MilitardeBraslia Braslia DF
LuisCludioMagalhesdeHolanda DuliaBringel Fortaleza CE
AlrioAlbertoFurtadoCorts Prevest Goiania GO
GiovanniElsonRafaeldeSouza LuzaTvora Fortaleza CE
GustavoHenriquedosSantos COESI Aracaju SE
IzaelFranciscodeBritoArajo U.E.TeotnioF.Brando CocaldoAlves PI
AndreyJhenShanChen Etapa Campinas SP
EmanuelGabrielTeodoro PolciaMilitardoParan Curitiba PR
Resultados
| 39 38 |
Programa Nacional
Olimpadas de Qumica
Ano Internacional de Qumica
HenriqueMartinezRocamora Etapa SoPaulo SP
LuizDavidBoteroAlessi BomJesusN.SdeLourdes Curitiba PR
ArturMeloCatunda BatistaSantosDumont Fortaleza CE
BrunoAlmeidaCosta FariasBrito Fortaleza CE
GuilhermeAniteleSilva AngloPrudentino Junqueirpolis SP
JulianaRamosTeixeiraBonfm SantoAntniodeJesus S.Ant.deJesus BA
MarianaLimaVieira EducandrioChristus Piripiri PI
MateusAlmeidaFariasdosSantos DomBarreto Teresina PI
MateusFreireBezerra MaterChristi Mossor RN
NathliaMariaFonsecaFres SantoAntniodeJesus S.Ant.deJesus BA
PauloRobertoPereiradeF.Filho FariasBrito Fortaleza CE
ThamiresStoppelliRibeiro FariasBrito Fortaleza CE
CarlosAugustoJardimChiarelli E.M.E.F.Profa.ArlindaR.Negri Dumont SP
IgorVianaFerreira MagnumCidadeNova BeloHorizonte MG
JooPedroVerasMunizFarias ArideSCavalcante Fortaleza CE
MatheusBittencourtBraga MilitardePortoAlegre PortoAlegre RS
BrunoGomesRibeiro DecisoAnglo-ICM Icm SP
RafaeldaCostaAraripe Petroplis S.B.doCampo SP
ArthurCarneiroMoura FariasBrito Fortaleza CE
CaioFelipeSiqueiraGomes 7deSetembro Fortaleza CE
DeborahGiovannaSantanaRabelo FariasBrito Fortaleza CE
EnricoPascucciLoffel Petroplis S.B.doCampo SP
FernandoAntnioS.deAragoFilho 7deSetembro Fortaleza CE
FloraMorganaC.doBonfmGorender Anchieta Salvador BA
GliciaRodriguesFerreira Master Fortaleza CE
GustavoGentilLeitedeArajo SantaCeclia Fortaleza CE
IlzaneMariadeOliveiraMorais SantaCeclia Fortaleza CE
IsabellaFreitasFigueiredo MagnumAgostiniano BeloHorizonte MG
JooMaurcioAlmeidaNos Petroplis S.B.doCampo SP
JooPauloMotaTelles Anchieta Salvador BA
JosuSilvaColhodeOliveira DomBarreto Teresina PI
JulianaSampaioSaraivadeOliveira Master Fortaleza CE
LucasAlmeidaLinhares FariasBrito Fortaleza CE
MrlonNeresdeMoura CursoPensi-PontodeEnsino RiodeJaneiro RJ
MatheusCariocaSampaio FariasBrito Fortaleza CE
MatheusdeAlmeidaCosta 7deSetembro Fortaleza CE
IV OBQ Jnior
| 39
Olimpada Brasileira
de Qumica 2011
Ano Internacional de Qumica
NairCammilaBenciodeL.Lopes Teleyos Fortaleza CE
PauloDaviBorgesEsteves FariasBrito Fortaleza CE
RicardoPatrcioHonoratoAlmeida FariasBrito Fortaleza CE
SarahBarretoOrnellas Anchieta Salvador BA
TinoMiroAurlioMarques ArideSCavalcante Fortaleza CE
GustavoHenriqueA.B.Melo BoaViagem Recife PE
LucasCavalcantedoNascimento FariasBrito Fortaleza CE
LucasEneasGomesPinheiro FariasBrito Fortaleza CE
PedroAngeloVazdeCarvalho Bernolli BeloHorizonte MG
ThayanaTaynaraAndradedosSantos SantoAntniodeJesus S.Ant.deJesus BA
ItaloLesionedePaivaRocha Master Fortaleza CE
PhabloWemesonFigueiredodeSouza Paraso J.doNorte CE
AnaLuizaNogueiraMoror Anchieta Salvador BA
AnaValriaGonalvesTorresIncio Paraso J.doNorte CE
GiovannaCarvalhoF.Figueirdo MonteiroLobato J.doNorte CE
GiselleFurtadoSilva SantAna Sobral CE
JooPedroIzidrioLima Lavoisier-Anglo Teresina PI
JooLucasMirandaFrancelino ArideSCavalcante Fortaleza CE
JoilsondeJesusBarretoJnior SantoAntniodeJesus S.Ant.deJesus BA
LucasResende Esc.MunicipalPaulaAssis ResendedeCosta MG
AndrMirandaOnofre Etapa SoPaulo SP
AlciaMouroVieira ArideSCavalcante Fortaleza CE
BeatrizPrazeresCmara BoaViagem Recife PE
CaioMatheusTeixeiraBrito SistemaElitedeEnsino Belem PA
CarlosAugustoF.F.deCarvalho Anchieta Salvador BA
DanielCrisstomoWainstock A.Liessin RiodeJaneiro RJ
FelipePinheiroMendes SantaCeclia Fortaleza CE
GabrielMaiaBezerra BatistaSantosDumont Fortaleza CE
GabriellaBrandoPita SartreCOC Salvador BA
GiselleSilvestredeJesus Paraso J.doNorte CE
HeitorAugustoSerafm Petroplis S.B.doCampo SP
IagoHenriqueGomesSilvadeJesus CentrodeExcelen.Master Aracaju SE
IgorLobatodoNascimento BrigadeiroNewtonBraga RiodeJaneiro RJ
karoVinhasFernandes FariasBrito Fortaleza CE
JooFelipeSouzaRibeiro Anchieta Salvador BA
LucasCunhaAgustini Petroplis S.B.doCampo SP
LucasdeFreitasMartins Anchieta Salvador BA
Resultados
| 41 40 |
Programa Nacional
Olimpadas de Qumica
Ano Internacional de Qumica
LucasLevyAlvesMoraes SagradoCoraodeJesus Teresina PI
LucasPinheirodeMarchi ArideSCavalcante Fortaleza CE
LucianoCostaBrito U.E.B.AlfredoSilva PaodoLumiar MA
LuizaFerreiraLopes Bandeirantes SoPaulo SP
MiguelTorrresdeAzevedoMaia Prevest Goiania GO
PaulaFernandaP.TeixeiradeOliveira ArideSCavalcante Fortaleza CE
PedroLucasdeSouzaSilveira MaterChristi Mossor RN
RenanNogueiraPinto FariasBrito Fortaleza CE
RodrigoResendedeVasconcelos Master Aracaju SE
ThaissaLannesPaulaSouza NotreDamedeLourdes Cuiab MT
VanessaPereiradeSouza MilitardeJuizdeFora JuizdeFora MG
VitorDiasGomesBarriosMarin AngloPrudentino Junqueirpolis SP
AndressaPaulaSilvaCoelho DomBarreto Teresina PI
BrunoCiconedeAlmeida ObjetivoUnidadeAquarius S.J.dosCampos SP
RaquelSouzaCaminhaBret FariasBrito Fortaleza CE
DboraLusaBez DuliaBringel Fortaleza CE
JlioCsarFerreiraFilho FariasBrito Fortaleza CE
PedroColetoBeserradosAnjos IntegraldeAtibaia S.Ant.deJesus BA
PedroTeotniodeSousa ArideSCavalcante Fortaleza CE
AbnerPintoSoares SalesianoS.Jos Natal RN
DanieladeSousaGomes FariasBrito Fortaleza CE
GiseleChristineAraujoNascimento EducandrioChristus Piripiri PI
MiguelWoodallSilvadeAssis Coleguium BeloHorizonte MG
JooVitorRochaAlmeida Lavoisier-Anglo Salvador BA
lvaroPessoaMenezes COESI Aracaju SE
AmandaRibeiroFonsecadeSouza BatistaGetsmani BeloHorizonte MG
FilipePadilhadeMiranda BoaViagem Recife PE
JulianeMariaMoreiraAguiar SantAna Sobral CE
LeonardodaSilvaOliva Deciso-Anglo NovaGranada SP
LucasRodriguesArajo Lavoisier-Anglo Teresina PI
RhanyellePereiradeJesus U.E.B.AlfredoSilva PaodoLumiar MA
SarahRoqueSanz PrevestSul Goiania GO
AmandaGomesdeVasconcelos FariasBrito Fortaleza CE
GiovannaCoimbraPorto Prevest Goiania GO
IzadoraMariaLealGomes Ass.Educ.Profa.Noronha DomPedro MA
JooVictorBezerraFirmiano FariasBrito Fortaleza CE
IV OBQ Jnior
| 41
Olimpada Brasileira
de Qumica 2011
Ano Internacional de Qumica
StfaniadeFigueiredoCoutinho Inst.Educ.SoJosdaEsccia PoosdeCaldas MG
RaquelPaesPatrcioT.Ferreira BrigadeiroNewtonBraga RiodeJaneiro RJ
DboraRomoPintoSantos Paraso J.doNorte CE
OtvioCunhadaSilva Fund.Itab.Dif.doEns.-FIDE Itabira MG
RobsonCamilottiSlompo PolciaMilitardoParan Curitiba PR
AlexandreMessiasMendesFilho Petroplis S.B.doCampo SP
AlexyaNunesFonsca U.E.B.AlfredoSilva PaodoLumiar MA
AmandadeSouzaMota DecisoAnglo-ICM Icm SP
AnaCeciliaVenncioVenceslau 7deSetembro Fortaleza CE
AnaLusaLemosBessadeOLiveira FariasBrito Fortaleza CE
AndrLusMoretiFrana FariasBrito Fortaleza CE
ArthurFelipeLustosa DomBarreto Teresina PI
BernardoBarcellosdeCastroCunha FariasBrito Fortaleza CE
CamillaRochadeO.Fontoura BrigadeiroNewtonBraga RiodeJaneiro RJ
CarlosHenriqueJardimChiarelli E.M.E.F.Profa.ArlindaR.Negri Dumont SP
CntiaAndradeCostaFonsca MaterChristi Mossor RN
DaniloVianaFigueiredo SetedeSetembro-COC PoosdeCaldas MG
FranciscoCludioAlmeidadaS.Jnior ArideSCavalcante Fortaleza CE
GabrielPereiraSouzadaSilva CursoOpo Camaragibe PE
GabriellaFernandesRampinelli GabrielChalupe Barueri SP
JooMarcosLimaGarcia Ass.Educ.Profa.Noronha DomPedro MA
JooPedroSilvaSoares SagradoCoraodeJesus Teresina PI
JooVictorFerreiradaCosta E.M.E.F.MonteiroLobato SoPaulo SP
JlioCezarCastroLinsBarroso CentroEduc.AdalbertoValle Manaus AM
KlebersonJohnSantosdeMaria EEEFMSolondeLucena Camp.Grande PB
LeonardoSantamariaFerrazSouto JardimSoPaulo SoPaulo SP
LuanaSadSoutoMaior SartreCOC Salvador BA
LucasMouraAndrade Lavoisier-Anglo Teresina PI
LusMilani Bandeirantes SoPaulo SP
MarcosAurliodeOliveiraJesus Bandeirantes SoPaulo SP
MatheusNunesGonsalves DecisoAnglo-ICM Icm SP
PedroKasprzykowskiS.Magalhes SantaCeclia Fortaleza CE
ThadeuHenriqueC.V.A.S.Costa PedroIIUEENII RiodeJaneiro RJ
BethaniaTavaresBarretodosReis MilitardeJuizdeFora JuizdeFora MG
GabrielGriep MariaAuxiliadora Canoas RS
BrunoCamposSantos MagnumAgostiniano BeloHorizonte MG
Resultados
| 43 42 |
Programa Nacional
Olimpadas de Qumica
Ano Internacional de Qumica
ErickAlvesMoitinho E.M.E.B-Gov.AndrF.Montoro Valinhos SP
NaraLviaCarlosdeCastroPinheiro MaterChristi Mossor RN
TawanaCntiaHenriquedeLima DuliaBringel Fortaleza CE
AnaMariaJucNovaesRamalho ArideSCavalcante Fortaleza CE
ArthurGuyFurtadoColfort DuliaBringel Fortaleza CE
LuisFernandoFreireFigueiredo Bernolli BeloHorizonte MG
CamilledeMouraAndrade ArideSCavalcante Fortaleza CE
JooGabrielMatosSerra Ass.Educ.Profa.Noronha DomPedro MA
LetciaGonalvesCaldeira MagnumAgostiniano BeloHorizonte MG
LucasAndradedeToledo Anchieta Salvador BA
MarianaMoraesPacheco MagnumAgostiniano BeloHorizonte MG
NdiaMariaPiresSilva GluciaCosta Timon MA
PauloDdimoCamuraV.Filho SantaCeclia Fortaleza CE
RafaelHiplitodeFarias FariasBrito Fortaleza CE
RenataOliveiraBatista U.E.TeotnioF.Brando CocaldoAlves PI
RodrigoSobraldeAthaydeRocha Anchieta Salvador BA
VitorMeloRebelo DomBarreto Teresina PI
VivianeBarbosaAiresLeal DuliaBringel Fortaleza CE
LeonardoGomesGonalves MilitardeBraslia Braslia DF
GabrielAlbuquerque Organ.Educ.LopesPereira Fortaleza CE
GabrielOliveiraMartins DuliaBringel Fortaleza CE
LusaLuzardoStarling Galois Braslia DF
RenatadeOliveiraMelo Coleguium BeloHorizonte MG
SusanaSilvaLima EducandrioChristus Piripiri PI
PatrickCasteloB.RamadaCampos ArideSCavalcante Fortaleza CE
ThayaraAlineNadalAfonso CentroEduc.AdalbertoValle Manaus AM
VictorHugoPinheiroSantos Lavoisier-Anglo Teresina PI
AntnioRomilsonPiresRodrigues E.F.PadreJosRoberto Martinpole CE
BrendaBezerraVasconcelos FariasBrito Sobral CE
BrunoVilasBoasFahel Anchieta Salvador BA
CaioLuisPinheiroArenas Anchieta Salvador BA
CristianPhilippeB.DiasEichstaedt JardimSoPaulo SoPaulo SP
EduardoGeraldoAssis Esc.MunicipalPaulaAssis R.deCosta MG
FelipeKutait SoJos Bauru SP
FranciscoArthurBomfmAzevedo ArideSCavalcante Fortaleza CE
GabrielPriscoDiogodeHolanda FariasBrito Fortaleza CE
IV OBQ Jnior
| 43
Olimpada Brasileira
de Qumica 2011
Ano Internacional de Qumica
JooVitorBuenoPeixoto Bandeirantes SoPaulo SP
LorenaOliveiradeAndrade DomBarreto Teresina PI
LucasHenriqueCamposMendes PauloFreire Jundia SP
LusFelipeVieiraSoaresBarradas DomBarreto Teresina PI
NewtonBezerradeOliveira ArideSCavalcante Fortaleza CE
RodrigoZanettedeMagalhes Etapa SoPaulo SP
RuanMachadoCoelhoRossato Etapa SoPaulo SP
VictorH.Mirkhan AugustoLaranja SoPaulo SP
VictorPauloNeryRibeiro CentroEduc.AdalbertoValle Manaus AM
VictriaFreitasVieiradaCunha Paraso J.doNorte CE
VinciusTakiutiMiura UNISUZ Suzano SP
LucasT.Kabayashi SoJos Bauru SP
VictorBarbosadoMonte CentroEduc.AdalbertoValle Manaus AM
AnaBeatrizdeQueirozLeite NEC-NcleodeEduc.Criativo Natal RN
IanZaerorosinQuadrosVieira Coleguium BeloHorizonte MG
IsabelaMarques Anchieta Salvador BA
PauloVictorCordeiroAlves FariasBrito Fortaleza CE
PedroB.Belisario Bernolli BeloHorizonte MG
ThalysXavierBatista FariasBrito Fortaleza CE
WalterUlbrichNeto E.M.Prof.EdgarM.Castanheira Joinvile SC
YanCruzFernandes xitodoCariri Fortaleza CE
YurideSouzaDutra Coleguium BeloHorizonte MG
AlcsiaMadeiroMartins InstN.S.Auxiliadora Baturit CE
CaioJosPrado FariasBrito Fortaleza CE
GabrielGomesdosSantos Certus SoPaulo SP
AmandaFerreiraBrasil EducareRedePitgoras Betim MG
AnaCarolineSaleseSouza FariasBrito Fortaleza CE
ArthurOliveiraVale MilitardoRecife Recife PE
BeatrizVazSampaiodeCarvalho Anchieta Salvador BA
BiancaAragoSilva FariasBrito Fortaleza CE
CaioTeixeiradeQueiroz Master Fortaleza CE
CamilaMachadoFigueiredoMenezes MilitardeBraslia Braslia DF
CarlosAlbertoGomesdaCostaJnior MotivaAmbiental JooPessoa PB
CarlosHenriqueM.SaboiaPompeu SantAna Sobral CE
DaniloMacielArajo Teleyos Fortaleza CE
DavidAzevedo Anchieta Salvador BA
Resultados
| 45 44 |
Programa Nacional
Olimpadas de Qumica
Ano Internacional de Qumica
FernandadeFigueiredoGomes MagnumAgostiniano BeloHorizonte MG
GabrielCicaleseBevilaqua Col.AplicaodaUFPE Recife PE
carodeAzevdoAlexandre ArideSCavalcante Fortaleza CE
JooVictorArajoDande MagnumAgostiniano BeloHorizonte MG
LeonardoVictorCostaLima Educ.ParasodaCultura Crato CE
LuizVictriodeCastroCruzMartins Bandeirantes SoPaulo SP
MarcoEnriquedosSantosAbensur Col.Integ.Objetivo MogidasCruzes SP
MarliadeMoraesBarros Ass.Educ.Profa.Noronha DomPedro MA
MatheusHenriqueMartinsCosta BoaViagem Recife PE
MyqueiasdoNascimentoSilva UEJosNarcisodaR.Filho Piripiri PI
OdilioFerreiraLimaNeto Master Fortaleza CE
PriscillaDiasLimaVicentini Anchieta Salvador BA
RaphaelZImmermannTorres MilitardeJuizdeFora JuizdeFora MG
RenanLeiteVieira FariasBrito Fortaleza CE
ValterAugustoDiProfoFelix Bandeirantes SoPaulo SP
VanessaFerreiraBaldoino DomBarreto Teresina PI
ViniciusMendesBarros deAplicao-UGB VoltaRedonda RJ
YuriSchenkelToledo PauloFreire Jundia SP
DenielDesconziMoraes MilitardeBraslia Braslia DF
GiancarloTomazzonideOliveira SalesianoDomBosco PortoAlegre RS
RodrigoEstevamMendesDantas BatistaSantosDumont Fortaleza CE
EmanuelaMartinsBezerraSoares MonteiroLobato J.doNorte CE
GabrielToledoGuerra DecisoAnglo-ICM Icm SP
GabrielaLucenadeA.Romeiro BoaViagem Recife PE
GleicianydaConceioNunes U.E.B.AlfredoSilva PaodoLumiar MA
JulianaGuimaresdeM.Furtado SagradoCoraodeJesus Teresina PI
LucasdaSilvaFrana CursoPensi-PontodeEnsino RiodeJaneiro RJ
LucasMelodeCosta SalesianoS.Jos Natal RN
LucasSilvaBraga FariasBrito Fortaleza CE
LuisAugustoIrineuAguiarRamos MotivaJardimAmbiental JooPessoa PB
MateusNunesdeOliveira FariasBrito Fortaleza CE
NairtonTavaresLima Plos Iguatu CE
TerezaCristhinadeMatosPereira Ass.Educ.Profa.Noronha DomPedro MA
ThasCristinaLeiteMoraes U.E.B.AlfredoSilva PaodoLumiar MA
BrenoDalBianco BomJesus Curitiba PR
IsabelaYang Etapa SoPaulo SP
IV OBQ Jnior
| 45
Olimpada Brasileira
de Qumica 2011
Ano Internacional de Qumica
AndersonSantosdeFreitas Esc.RodriguesAlves Sta.M.doSul RS
AndrLuizNunesAmaru PedroIIUEENII RiodeJaneiro RJ
ngeloMacielOliveira SantaCeclia Fortaleza CE
AntnioEmanuelMarquesdosSantos Teleyos Fortaleza CE
BeatrizMelodeAlbuquerque Master Fortaleza CE
BrunoMartinsAboud AugustoLaranja SoPaulo SP
BrunoVasconcelosSilva FariasBrito Fortaleza CE
EenanePedroMatosBarros DomBarreto Teresina PI
EmanuelAnselmoN.Segundo SagradoCoraodeJesus Teresina PI
GabrielQueirozMoura DomBarreto Teresina PI
IracemaCapistranoCostaFook Esc.JardimCrescimento SoLus MA
JooCzarOliveiraC.deSousa FariasBrito Fortaleza CE
JulianaNovaesBuenodeCamargo AugustoLaranja SoPaulo SP
LetciaLaraRamosLima Ass.Educ.Profa.Noronha DomPedro MA
LetciaTutihashi Col.Integ.Objetivo MogidasCruzes SP
MariaClaraSilveiraGontijo MagnumAgostiniano BeloHorizonte MG
PedroVictorAlvesBarbosa SagradoCoraodeJesus Teresina PI
RenatoCorreiaBorgesLopes SistemaElitedeEnsino Belem PA
SadiKneippNeto FariasBrito Fortaleza CE
SophiaGasparC.daS.V.Trvia ArideSCavalcante Fortaleza CE
TiemiBeatrizFernandesNagazawa NotreDamedeLourdes Cuiab MT
MariaAlinedoNascimentodoCarmo Anchieta Salvador BA
NoliaYeseniaRojasCruz BatistaGetsmani BeloHorizonte MG
RassaMariaBastosVasconcelos FariasBrito Fortaleza CE
CaioSouzaPimentel FariasBrito Fortaleza CE
JonasMartinsdeAbreu MaterChristi Mossor RN
AlexandreSatyrodeMedeiros Galois Braslia DF
ArthurMedeirosGrigoli ArideSCavalcante Fortaleza CE
FelipeAndersonNunesLima Integral Teresina PI
GustavoFreitasdeAbreu MagnumAgostiniano BeloHorizonte MG
IgorTasukeCorradeA.Morimoto Esc.InternacionaldeAldeia Camaragibe PE
JooPedroMarquesAzevedo MaristaDomSilvrio BeloHorizonte MG
MarianaDiasCarvalho MilitardeJuizdeFora JuizdeFora MG
RebeccaPeixotoHolanda JuvenaldeCarvalho Fortaleza CE
SandroLuisVieiraColho U.E.B.PodaVida PaodoLumiar MA
SauloGonaloBrasileiro Col.AplicaodaUFPE Recife PE
Resultados
| 47 46 |
Programa Nacional
Olimpadas de Qumica
Ano Internacional de Qumica
ViniciusLeonardiPinheiro Atibaia Atibaia SP
EduardoSoareseSilvaBritto Anchieta Salvador BA
EduardoAndreattiTakashe PolciaMilitardoParan Curitiba PR
VinciusFranciscoVieiraFerreira Master Fortaleza CE
AndersonCarlosFelix ArideSCavalcante Fortaleza CE
BernardodeSTelesPassos SartreCOC Salvador BA
EdkioSantosPassos DomBarreto Teresina PI
GabrielaFiorRibeiro MagnumCidadeNova BeloHorizonte MG
GleybsonRobertoArajodosSantos DuliaBringel Fortaleza CE
GustavoAlencarSilvaA.Dantas GabrielChalupe Barueri SP
HenriqueMatosCampos Col.Pr-Universitrio IlhaSolteira SP
JonathanIvensSilvadosSantos MilitardeJuizdeFora JuizdeFora MG
LiadeFreitasArajoAlves FariasBrito Fortaleza CE
LucasTadeuRochaSantos FariasBrito Sobral CE
MrcioNormandoBorgesC.Filho MilitardeFortaleza Fortaleza CE
PaulaAbikoNavarroCarrion CharlesDarwin Linhares ES
RafaelaMayumiYamasaki SalesianoSoGonalo Cuiab MT
SamanthaAlvesdeOliveira MilitardeBraslia Braslia DF
VanessaPelasiniGuimaresMello Petroplis S.B.doCampo SP
YasminPissolateMattosBretz MagnumCidadeNova BeloHorizonte MG
MathewsCorreiadoNascimento ArideSCavalcante Fortaleza CE
GiovanaLandaldeAlmeidaLobo PositivoJnior Curitiba PR
JooPauloNunesSoares FundaoBradesco Braslia DF
AlexandreRibeiroCarneiro Acesso FeiradeSantana BA
AlyssonFaanhaMoreira Inst.deEducaoCastro Aquiraz CE
AnaBeatrizAnchietaSeixas AdventistadoGuar Braslia DF
BrenoDiGregorio SanrtaMnica MogidasCruzes SP
CamilaMachadodeArajo Esc.VirgemdeLourdes Camp.Grande PB
IsabellaRossanaAlmeidaS.Soares BoaViagem Recife PE
JooVictorSoaresCarvalho ArideSCavalcante Fortaleza CE
JuanFelipeSiqueiraSilva SESIHanletoMagnavacca BeloHorizonte MG
LarissaRodriguesEsmeraldoCarneiro ArideSCavalcante Fortaleza CE
MarinaCatundaPinheiroJuc ArideSCavalcante Fortaleza CE
MilennaMaclinksdeMacedo JuvenaldeCarvalho Fortaleza CE
PedroPaixoBorges PedroIIUEENII RiodeJaneiro RJ
VitriaLaraMartinsMarques FariasBrito Fortaleza CE
IV OBQ Jnior
| 47
Olimpada Brasileira
de Qumica 2011
Ano Internacional de Qumica
UgorTomazFernadez SalesianoS.Jos Natal RN
AlexandreMendonaCardoso MilitardeSalvador Salvador BA
AntonioVictorGouveiaA.dosSantos ArideSCavalcante Fortaleza CE
CarolineArequeUchaMaciel FundaoBradesco Braslia DF
GiovannaMariaPereiradeSampaio PositivoJnior Curitiba PR
IgorMoreiraEduardo FariasBrito Fortaleza CE
LetciaEduardadeArrudaPrado PolciaMilitardoParan Curitiba PR
RafaelSantanaBrito MilitardeSalvador Salvador BA
AlineAlvesRodrigues Col.Integ.Objetivo MogidasCruzes SP
AndersonDigenesSouzaRicardo Master Fortaleza CE
AnneTeresaDamascenoCosta Profa.AlicedoC.deMelo SoBenedito CE
GeovaneTomazBatista Esc.13deJunhoObjetivo Junqueirpolis SP
HiagoVerasArajoSoares DomBarreto Teresina PI
IagoSantosRodrigues DomBarreto Teresina PI
IgorCoelhoBarbosa ArideSCavalcante Fortaleza CE
IngraBezerradeMeloGonalves 7deSetembro Fortaleza CE
JssicaTenfenWazilerski MaristadeCascavel Cascavel PR
JooFelipedosSantosSales LucianoFeijo Sobral CE
JooHenriqueAlvesTaveira Esc.ModelodeIguatu Iguatu CE
JooMarcosMesquitaViana UEJosNarcisodaR.Filho Piripiri PI
JulianaFerreiradeOliveira SantoAntniodeJesus Sto.Ant.deJesus BA
KevinSaymonA.SantosCerqueira JosdeAnchieta FeiradeSantana BA
KeylaMariaRodriguesGomes Esc.SenadorNiloCoelho Picos PI
LeonardoCairoCarvalhoNeto Anchieta Salvador BA
LucasFernandoBarbosaMendes MaristaDomSilvrio BeloHorizonte MG
LucasTavaresCostaMeira ImaculadaConceio Camp.Grande PB
LciaOliveiraLindso FarinadoBrasil SoLus MA
LuisArthurCoutinhodeBrito Ass.Educ.Profa.Noronha DomPedro MA
LuizMatheusFerreiraFernandes DomBarreto Teresina PI
MarcosVinciusPitombeiraNoronha Esc.NormalR.deLimoeiro Lim.doNorte CE
MatheusHenriquedeArajoDutra Prevest Goiania GO
MatheusPradodePaula MagnumAgostiniano BeloHorizonte MG
MatheusRibeiroFelippin MilitardeBraslia Braslia DF
MatheusRochaBarbosa GluciaCosta Timon MA
MayraReginaTeixeiradeMiranda SagradoCoraodeJesus Teresina PI
MilenaScurWagner SebastiodeAbreu Maranguape CE
Resultados
| 49 48 |
Programa Nacional
Olimpadas de Qumica
Ano Internacional de Qumica
TarikH.Ashmawi Bandeirantes SoPaulo SP
ThaysGomesVasconcelosBrito FariasBrito Fortaleza CE
VictorMachado Integral Teresina PI
VinciusXavierGarcia PedroIIUEENII RiodeJaneiro RJ
HeitorMartinsOliviera Coleguium BeloHorizonte MG
DouglasRafaelSilvaSousa SoLucas Picos PI
FelipeJosdeCastroSantos Master Fortaleza CE
GabrielaRebeccaHack PositivoJnior Curitiba PR
AlexandraKazumiKameoka Exponente Recife PE
GabrielaFerrodaCunha PedroIIUEENII RiodeJaneiro RJ
IsabelliAlvesFernandes Deciso-Anglo NovaGranada SP
JooVictorMarquesGomesRodrigues MaristadeGoinia Goiania GO
KarenLumiIi PilardoSul PilardoSul SP
LucasEduardoPortodeToledo ObjetivoUnidadeAquarius S.J.dosCampos SP
MarliadeOliveiraCunha Esc.ModelodeIguatu Iguatu CE
MirnaMariaAlvesPorto FreiJooPedroVI Fortaleza CE
NathliaUchadeCastroBessa ArideSCavalcante Fortaleza CE
AlanAndrGoldenberg Bandeirantes SoPaulo SP
AnaCarolinaLobatoSaldanha MaristaDomSilvrio BeloHorizonte MG
AnaCecliaFernandesCavalcante FariasBrito Fortaleza CE
AnaLetciaCarvalhoLobo BoaViagem Recife PE
AnnaHeloisadeVasconcelos FariasBrito Fortaleza CE
AntonioJaquesPedreiraNeto Anchieta Salvador BA
ArmindoAugustodeN.Albuquerque N.S.dasNeves Natal RN
ureaSoaresZica MagnumAgostiniano BeloHorizonte MG
BrunaQueirozAllenPalcio FariasBrito Fortaleza CE
CaioMachadoVieiraBorges ArideSCavalcante Fortaleza CE
CaioVitorSaundersBarreto SantaCeclia Fortaleza CE
CamilaRodriguesFerreira Sag.CoraodeMaria Vitria ES
EduardoAntnioDiasCristino FariasBrito Sobral CE
FranciscoAlanHolandaAlmeida FariasBrito Fortaleza CE
GuilhermeFaroCorraReis LiceudeEstudosIntegrados Aracaju SE
GustavoCrubellatiNunes CentroEduc.AdalbertoValle Manaus AM
IghorBelmMoreiradeOliveira FariasBrito Fortaleza CE
IsabellaFranaMoreira MagnumAgostiniano BeloHorizonte MG
ItaloBezerra Esc.NormalR.deLimoeiro Lim.doNorte CE
IV OBQ Jnior
| 49
Olimpada Brasileira
de Qumica 2011
Ano Internacional de Qumica
JairAdhonaiCorreiadosSantos GeoSul JooPessoa PB
JosWellingtonSaraivaSousaJnior ArideSCavalcante Fortaleza CE
JoycedosSantosMonteiro Contato Macei AL
LaraMariaAlexandredeArajo Esc.VirgemdeLourdes Camp.Grande PB
LuccaSeveroFurtado N.S.dasNeves Natal RN
NicolasMartinsAguiar FariasBrito Fortaleza CE
RicardoAmaralMendesdaConceio OLIMPO-DF Braslia DF
VivianBarbosadaSilva DuliaBringel Fortaleza CE
GabrielFerreiraLopes E.M..E.F.Profa.Iracema.. Lindia SP
MarcosGabrielLimadaCosta Acesso FeiradeSantana BA
FranciscoMauroFalcoM.Filho SantAna Sobral CE
JosOtviodeOliveiraVidal MilitardeManaus Manaus AM
PauloBrunodeSousaCardoso LuzaTvora Fortaleza CE
RafaelMaestPrado ObjetivoUnidadeAquarius S.J.dosCampos SP
RafaeladeAlbuquerqueCunha FariasBrito Sobral CE
SamanthaSouzaNapoleo ArideSCavalcante Fortaleza CE
VitorBorgesCataldo CursoPensi-PontodeEnsino RiodeJaneiro RJ
BeatrizPauladaRocha Alfa DuquedeCaxias RJ
CarinaLobarinhasCorreia MaristaSoJos RiodeJaneiro RJ
RicardoRibeirodaCruz PilardoSul PilardoSul SP
AbnerNbregaMaiaAires FariasBrito Fortaleza CE
BrunaGreenhalghdeM.Ebrahim DuliaBringel Fortaleza CE
CaioSantosCabral DuliaBringel Fortaleza CE
CarlosFelipeL.deMoraes Etapa SoPaulo SP
IsabellaCoelhoFerraz Etapa SoPaulo SP
LucasRenatoAlbertiFranco PolciaMilitardoParan Curitiba PR
OrlandoWozmiokdeL.Nogueira PositivoJnior Curitiba PR
PedroRodriguesMachadodosSantos U.E.TeotnioF.Brando CocaldoAlves PI
danBarbosaRibeiro DomBarreto Teresina PI
AlexandreSeungYungKwon Bandeirantes SoPaulo SP
AllanBatistaMarton EducareRedePitgoras Betim MG
AmandaLimadeJesus Esc.TcnicaWalterBelian SoPaulo SP
AnaMartaV.R.FurtadoEstanislau Anchieta Salvador BA
AndrezaFerreiraMalta MagnumAgostiniano BeloHorizonte MG
BeatrizRafaelaRodriguesLeito Ass.Educ.Profa.Noronha DomPedro MA
BrunaDrummonddeMouraMiguez MagnumAgostiniano BeloHorizonte MG
Resultados
| 51 50 |
Programa Nacional
Olimpadas de Qumica
Ano Internacional de Qumica
CharlesAllanDuarteLimaFilho ArideSCavalcante Fortaleza CE
DanielCordeirodeAlencarFilho Paraso J.doNorte CE
DanielPonteFrota FariasBrito Sobral CE
DvilaLorraineMarqueseSilva E.M.PadreCcerodeCastro Teresina PI
DiegoCostadeFreitas ReinodoSaber PiquetCordeiro CE
EduardoMatosLinhares SagradoCoraodeJesus Teresina PI
EduardoRgisBarreiradeFigueirdo ArideSCavalcante Fortaleza CE
FlvioMeirelesMouraPessoa SantaCeclia Fortaleza CE
FlvioTvoraDix-huitRosadoVentura MaristadeNatal Natal RN
GabrielCaetanoVisconti Bandeirantes SoPaulo SP
GabrielVianaFigueiredo SetedeSetembro-COC PoosdeCaldas MG
GladysDantasBorges Profa.AlicedoC.deMelo SoBenedito CE
GuilhermeRobertoNavesMiranda Galileu Anpolis GO
GustavoFerreiradaSilvaMontes deAplicao-UGB VoltaRedonda RJ
GustavoGomesArruda CentroEduc.C.ElShadai SantoAndr SP
HlioMendesdeCarvalhoC.Lima SagradoCoraodeJesus Teresina PI
HrcullesTeixeiraSilva EducandrioChristus Piripiri PI
JadedaCostaWang MartaFalco Manaus AM
JooPedroAparecidodaSilva Esc.ProfssionalDomBosco PoosdeCaldas MG
JooRicardodoEgitoS.Gonalves Parthenon Guarulhos SP
JulioCsarLopesdaSilva Esc.TcnicaWalterBelian SoPaulo SP
KarolinaKotsuho Lettera Teresina PI
LaraRochaSobral FariasBrito Fortaleza CE
LuanaCarvalhoMendes Certo Teresina PI
LuanadeLimaMatoso MaristaDomSilvrio BeloHorizonte MG
LucasFilipeSousaRamos E.E.F.ProfaEstefnicaMatos Itapaj CE
LuizJosBezerraPinheiro FariasBrito Fortaleza CE
MariaTeresadosSantosSilva MagnumAgostiniano BeloHorizonte MG
MarianedeJesusArajo SantAna Sobral CE
MarinaRajoSantiagoMaciel MagnumAgostiniano BeloHorizonte MG
MayaraCristinaSiqueiraLima Integral Teresina PI
OlavoCarvalhoHaase Coleguium BeloHorizonte MG
PedroHenriqueSantanaSousa SagradoCoraodeJesus Teresina PI
RafaelFontesdeCarvalhoAmorim DomBarreto Teresina PI
RaquelGurgelLinsRamalho FariasBrito Fortaleza CE
RicardoKaoruKomeso Objetivo-Cantareira SoPaulo SP
IV OBQ Jnior
| 51
Olimpada Brasileira
de Qumica 2011
Ano Internacional de Qumica
RodolfoEduardoSantosCarvalho Salvador Aracaju SE
TeresaVirgniaNevesFloriano Amadeus Aracaju SE
ThiagoLucasOliveiraHonorato ArideSCavalcante Fortaleza CE
VanessaGabriellyPedrosaDantas Integral Teresina PI
WendelMatosdaS.Pedreira Gnesis FeiradeSantana BA
GabrielSantosGonalves JosdeAnchieta FeiradeSantana BA
PaulaLadeiaBarros Anchieta Salvador BA
PedroRibeirodaCosta Anchieta Salvador BA
ArthurdeMeloBarbosaBittencourt MagnumAgostiniano BeloHorizonte MG
CamilaFariaGarcia Inst.Educ.S.JosdaEsccia PoosdeCaldas MG
CarlosAntnioCanabravaJnior MaristadeGoinia Goiania GO
CarlosHenriqueArajoBezerra ArideSCavalcante Fortaleza CE
DhouglasLucenaArajo ArideSCavalcante Fortaleza CE
EmillyAmorimDantas Esc.JardimCrescimento SoLus MA
FelipePriscoDiogodeHolanda FariasBrito Fortaleza CE
GiancarloM.Massoni Atibaia Atibaia SP
IgorTelesFortuna SantaCeclia Fortaleza CE
IsadoradeLimaSantos MagnumCidadeNova BeloHorizonte MG
JooVictorMoraisdoCouto MagnumAgostiniano BeloHorizonte MG
JliaNogueiraGuedesMonteiro DamasdaInstruoCrist Recife PE
KtiadaSilvaSouza Plos Iguatu CE
LasMartinsQueiroz Prevest Goiania GO
MatheusMachadoDiniz Anchieta Salvador BA
MelquesedequeFerreiraMorais BatistaSantosDumont Fortaleza CE
ThalysHeriquiAndradedaSilva ArideSCavalcante Fortaleza CE
TomSeichiDaNbregaGuenka Galois Braslia DF
UryMatheusSerpadeF.Ramalho MilitardoRecife Recife PE
GabriellaMariaRadkeChaves E.M.AnnaMariaHarger Joinvile SC
EvaAlicedaSilvaPinheiro SESIHanletoMagnavacca BeloHorizonte MG
EbenzerPintoBandeiraNeto ArideSCavalcante Fortaleza CE
lvaroBrcioA.AlmeidaeSilva FariasBrito Fortaleza CE
AnaBeatrizValdivinoCalixtodeAguiar FariasBrito Fortaleza CE
AugustoCsarO.SerraPintoFilho MotivaAmbiental JooPessoa PB
BiancaDesidrioMatosJacana FariasBrito Fortaleza CE
CamilladaRochaBezerril Integral Teresina PI
CatarinaCarvalhoHaase Coleguium BeloHorizonte MG
Resultados
| 53 52 |
Programa Nacional
Olimpadas de Qumica
Ano Internacional de Qumica
DimasMacedodeAlbuquerque FariasBrito Fortaleza CE
EduardodosSantosOliveira CEMP-C.deEd.MpaldePacajus Pacajus CE
EullioSoteroGalvoJnior Integral Teresina PI
GabrielaGuimaresR.dosSantos Bandeirantes SoPaulo SP
GabrielleSoaresSalomo SoJos PousoAlegre MG
GuilhermeBragadePaula FariasBrito Fortaleza CE
JhonesGonalvesdeAguiar E.E.E.F.M.GiselaS.Fayet Dom.Martins ES
JosGabrielPaesSantana Amadeus Aracaju SE
JosuFernandesdeOliveiraJnior Inst.deEducaoCastro Aquiraz CE
JliaCarolinaMadeiraBoffa Parthenon Guarulhos SP
MarcosViniciusdaSilva MaristadeCascavel Cascavel PR
MarinaSantosMoulin MagnumAgostiniano BeloHorizonte MG
MatheusdeOliveiraCamelo N.S.dasNeves Natal RN
OtvioAugustoScariotto MaristadeCascavel Cascavel PR
OtonGonalvesdeSNeto Esc.JardimCrescimento SoLus MA
PedroHenriquedaSilvaAbreu Lettera Teresina PI
RafaeldeAlmeidaBaziotti SetedeSetembro-COC PoosdeCaldas MG
RodrigoCoelhoLeite MagnumAgostiniano BeloHorizonte MG
TiagoRodrigusSeniuk Col.Est.N.S.Ftima Iva PR
VictorLanderArajoBandeira FariasBrito Fortaleza CE
VictorMatheusdeLima MilitardeManaus Manaus AM
VitorRodriguesBarbosa CharlesDarwin VilaVelha ES
IV OBQ Jnior
| 53
Olimpada Brasileira
de Qumica 2011
Ano Internacional de Qumica
OlimpadaBrasileiradeQumica-2011
MODALIDADEA(1e2anos)
PARTEA-QUESTESMLTIPLAESCOLHA
QUESTO1
comum encontrarmos objetos que brilham no escuro, particularmente,
brinquedos de crianas. Tais objetos podem apresentar o sulfeto de zinco
emsuaconstituio.Estefenmenoocorreemrazodequealgunseltrons
destestomosabsorvemenergialuminosaecomissosaltamparanveisde
energia mais externos. Esses eltrons retornam aos seus nveis de origem,
liberandoenergialuminosae,noescuro,possvelobservaroobjetobrilhar.
Essacaractersticapodeserexplicadaconsiderandoomodeloatmicopro-
postopor:
a)Thomson b)Rutherford
c)Bohr d)MarieCurie
e)Planck
QUESTO2
At 1982, a presso padro era tomada como uma atmosfera (1 atm ou
101325Pa)eatemperaturacomo0C(273,15K)e,portanto,ovolumemo-
lardeumgsnasCNTPera22,4L/mol.Apartirde1982,aUnioInternacio-
naldeQumicaPuraeAplicada(IUPAC)alterouovalordapressopadro,de
formaqueasnovascondiesnormaisdetemperaturaepressopassarama
ser:t=0CouT=273,15Kep=100.000Paou1bar.Assim,ovalorreco-
mendadohojepelaIUPAC,paraovolumemolar:
a)V
m
=0,021631m
3
mol
-1
b)V
m
=0,035845m
3
mol
-1
c)V
m
=0,022711m
3
mol
-1
d)V
m
=0,035745m
3
mol
-1
e)V
m
=0,027211m
3
mol
-1
A
Exames Mod. A
| 55 54 |
Programa Nacional
Olimpadas de Qumica
Ano Internacional de Qumica
QUESTO3
Dentreosprincipaisfatoresqueinfuenciamnaproduoagropecuria,po-
demoscitar:oclima,omaterialgentico,omanejodepragas,asdoenase
plantasdaninhaseomanejodosolo,comespecialnfasenomanejoqumi-
cocomobaseparaanutriodasplantas.Emrazodaproduodealimen-
tosemescalacadavezmaior,osnutrientesdosoloquedovidasplantas
voseesgotando.Parasupri-los,produtosqumicosconhecidoscomofertili-
zantessoincorporadosterraemquantidadescrescentes.Paracorreoda
acidezdosolo,oprocedimentoderotinaacalagematravsdaincorpora-
odeumxidobsico.corretoafrmarqueessexidobsicopodeser:
a)MgO
2
b)CaO
c)SO
2
d)NaO
e)CO
QUESTO4
AporcentagemdelcooladicionadogasolinaregulamentadaporLei,e
recentementefoiestabelecidoumnovopadroquede18a24%(volume/
volume).Quando50mLdeguaforemmisturadosa50mLdegasolinaco-
mercializadanospostosdeserviocomomximoteorpermitidodelcool,
serobservadaaformaode:
a)100mLdeumamisturahomognea
b)Duasfasesde50mLcada
c)Duasfases,sendoamaisdensade38mL
d)Duasfases,sendoamaisdensade62mL
e)Duasfasessendoamaisdensade74mL
QUESTO5
Supondoqueaseguintereaoocorrapormeiodeummecanismodeuma
nicaetapaelementarnasduasdirees:
A+2B AB
2
Sendok
1
ek
2
asconstantes,aumadeterminadatemperatura,paraasreaes
diretaeinversa,respectivamente,assinaleaalternativacorreta.
OBQ - 2011 Fase III
| 55
Olimpada Brasileira
de Qumica 2011
Ano Internacional de Qumica
a)k
1
/k
2
=k=[AB
2
]/[A][B]
2
b)k
1
/k
2
=k=[A][B]
2
/[AB
2
]
c)k
2
/k
1
=k=[AB
2
]
2
/[A][B]
d)k
2
/k
1
=k=[A][B]
2
/[AB
2
]
2
e)k
2
+k
1
=k=[AB
2
]/[A][B]
2
QUESTO6
Ovolumedecidontrico0,1mol.L
-1
necessrioparaneutralizarumamistura
0,40gdehidrxidodesdioe1,71gdehidrxidodebrio
a)20mL b)30mL c)50mL d)200mL e)300mL
QUESTO7
Oetanolanidro,ouseja,etanolisentodegua,podeserobtidoapartirdo
etanol96GLportratamentocomcalvirgemCaO.Acalvirgemreagecom
aguaconformeaequaoabaixo,desidratandooetanol.
CaO(s)+H
2
O(dissolvidoemlcool)Ca(OH)
2
(s).
SobreesseprocessoCORRETOafrmarque:
a)ohidrxidodeclcioformadoreagecomoetanol
b)oxidodeclcioreagecometanolpararetiraragua
c)ohidrxidodeclcioformadopodeserseparadoporfltrao
d)amisturaobtidaapsareaoumamisturahomognea
e)oxidodeclcioatuacomoumagenteredutor
QUESTO8
Acombustocompletadopropanoocorresegundoareao,representada
pelaequaonobalanceada,abaixo:
C
3
H
8
(g)+O
2
(g) CO
2
(g)+H
2
O(g)
ArelaoentreosvolumesdeO
2
(g)consumidoedeCO
2
(g)produzidode:
a)1:1 b)2:3 c)3:1 d)3:2 e)5:3
Exames Mod. A
| 57 56 |
Programa Nacional
Olimpadas de Qumica
Ano Internacional de Qumica
QUESTO9
A quantidade de gs carbnico produzida pela respirao de seres vivos
podeseracompanhadaatravsdareaodessegscomumasoluosatu-
radadehidrxidodebrio,medindo-seoaumentodemassadessasoluo.
Esseaumentodemassasedeve:
a)formaodecarbonatodebrio
b)adsorodoCO
2
adsorvidopelagua
c)formaodecidocarbnico
d)precipitaodohidrxidodebrio
e)formaodexidodebrio
QUESTO10
Umaamostradeumcidodiprticopesando12,25gfoidissolvidaemgua
eovolumedasoluocompletadopara500mL.Se25,0mLdestasoluo
soneutralizadoscom12,5mLdeumasoluodeKOH1,00mol.L
-1
,amassa
molardessecido,considerandoqueosdoisprtonsforamneutralizados,
iguala:
a)2,25 b)24,5 c)49,0 c)98,0 e)122,5
Otalentonoumapropriedadeprivada,umaproprie-
dadepblicaeningumtemodireitodedesperdi-lo.
WladimirEfroimson,geneticistarusso.
OBQ - 2011 Fase III
| 57
Olimpada Brasileira
de Qumica 2011
Ano Internacional de Qumica
ParteB-QUESTESANALTICO-EXPOSITIVAS
QUESTO11
A B
C
Considerandoos3elementos(A,BeC)indicadosnatabelaperidicaacima,
determine:
a)otomoqueapresenta
a.1)omaiorraioatmico.
a.2)amaiorafnidadeeletrnica
a.3)amaiorenergiadeionizao.
b)afrmuladocompostoformadoentre:
b.1)otomoAeotomoB.
b.2)otomoBeofor
c.3)otomoCeooxignio
QUESTO12
Paracadaumadasespciesqumicasabaixo:
I)NH
4
+
II)PCl
3
III)CO
3
2-
IV)NO
3
-
V)BF
4
-

a)EscrevaaestruturadeLewis
b)Indique:
b.1)ahibridaodotomocentral
b.2)onmerodeparesdeeltronsnoligantessobreotomocentral
b.3)onmerodeligaessigmaedeligaespienvolvendootomocentral
b.4)ageometriaemtornodotomocentral
Exames Mod. A
| 59 58 |
Programa Nacional
Olimpadas de Qumica
Ano Internacional de Qumica
QUESTO13
Uma macromolcula biolgica foi isolada de uma fonte natural em quan-
tidademuitopequenaesuamassamolarfoideterminadacomosendo4,0
x10
5
g.mol
-1
.Paraumasoluopreparadapeladissoluode0,8mgdessa
macromolculaem10,0gdegua.
a)Calcule
a.1)opontodecongelamento
a.2)apressoosmtica,a25C
b) Suponha que a massa molar dessa macromolcula no fosse conhecida
e que se pretendesse calcul-la a partir da determinao da presso os-
mticadasoluocitadaacimaequefossecometidoumerrode0,1torr
namedidadessapressoosmtica,qualseriaovalorencontradoparaa
massamolardamacromolcula?
QUESTO14
Ocarbetodesilcio(SiC),tambmconhecidocomocarborundum,umasubs-
tnciaduraempregadacomoabrasivo,podeserobtidoapartirdareaode
SiO
2
comcarbono,aaltastemperaturas,conformeaequaoqumica(no
balanceada)abaixo:
SiO
2
(s)+C(s)SiC(s)+CO(g)
a)Reescrevaaequaoqumicaacima,devidamentebalanceada
Emumexperimentocolocou-separareagir6,01gdeSiO
2
e7,20gdecarbono.
b)Qualseroreagentelimitante?
c)Quemassadecarborundumpoderserobtida,considerandooconsumo
completodoreagente(rendimentode100%)?
d)Quemassarestardoreagenteemexcesso?
e)Se,noexperimentoacima,obtm-se2,56gdeSiC,qualorendimentodareao?
QUESTO15
Paraareao:2NO(g)+Br
2
(g)2BrNO(g),tem-seumKc=0,21Lmol
-1

a350C.
Sobreestareao,respondaasquestesabaixo.
a)Se2,0x10
-3
molsdeNO,4,0x10
-3
molsdeBr
2
e4,0x10
-3
molsdeBrNO
sointroduzidosemumrecipientedevolumeiguala50,0mL,350C,em
quesentidoocorrerareao?Justifquesuaresposta
Exames Mod. A
| 59
Olimpada Brasileira
de Qumica 2011
Ano Internacional de Qumica
b)QualovalorKpparaessareaoa350C?
c)Seomesmorecipientecontm,noequilbrio,1,4x10
-3
molsdeNOe1,4x
10
-4
molsdeBrNOa350C,quequantidadedeBr
2
estarpresente?
d)Seaosistemadescritoem(c)seadicionaumgsinerte,demodoquea
pressototaldentrodorecipientepasseaserde3atm,a350C:
d.1)Ocorrermudanasnasconcentraesdoscomponentesdamistura?
d.2)Oequilbrioserdeslocado?
d.3)Seocorredeslocamento,emquesentidoser?
Justifquesuasrespostas.
QUESTO16
Um tcnico de laboratrio dispe de uma soluo de NaOH, que no era
utilizadahmuitotempo,eemcujortuloestescrito:NaOH0,25mol.L
-1
.
Comoestsoluoinstvel,antesdeus-laotcnicodecidiutitular25,0
mL dessa soluo com uma soluo de HCl 0,25 mol.L
-1
e gastou 22,5 mL
destaltimasoluo.Responda:
a)PorqueasoluodeNaOHinstvel?
b)Aconcentraoindicadanortuloestcorreta?
c) Como voc poderia preparar 250 mL de uma soluo exatamente 0,25
mol.L
-1
apartirdasoluaoanterior?(considerequevocdispedegua
destiladaedeNaOHslidoequeaadiodeNaOHslidonoalterao
volumedasoluo).
Dados:R=0,082dm
3
.atm.K
-1
.mol
-1

Constantecrioscpicadagua(KC)=1,86Kkg.mol
-1
ou
1,86Ckg.mol
Massasatomicasaproximadas:
H=1,0;C=12,0;O=16,0;Na=23,0;Si=28,1;Ba=137,3
Nmerosatmicos:
H=1;B=5;C=6;N=7;O=8;F=9;P=15;Cl=17
Gabarito:
1 2 3 4 5 6 7 8 9 10
C C B D A E C E A D
Exames Mod. A
| 61 60 |
Programa Nacional
Olimpadas de Qumica
Ano Internacional de Qumica
OlimpadaBrasileiradeQumica-2011

MODALIDADEB(3ano)
PARTEB-QUESTESMLTIPLAESCOLHA
QUESTES1a7
VerasmesmasquestespropostasparaaModalidadeA.
QUESTO8
Os compostos aromticos, por serem bastante estveis do preferencial-
mente reaes de substituio, e, dentre essas, as reaes de substituio
eletroflicas so as mais favorveis. Considerando essas reaes, indique a
alternativaqueapresentaoscompostosemordemcrescentedereatividade,
numareaodenitrao
a)Clorobenzeno,Anilina,BenzenoeTolueno
b)Tolueno,Benzeno,AnilinaeClorobenzeno
c)Benzeno,Anilina,ToluenoeClorobenzeno
d)Benzeno,Tolueno,ClorobenzenoeAnilina
e)Clorobenzeno,Benzeno,ToluenoeAnilina
QUESTO9
Onomesistemtico(IUPAC)doparacetamol,umdosanalgsicosmaiscon-
sumidosemtodoomundo,N-(4-hidroxifenil)etanamida.Oparacetamol
podeserobtidoapartirdareaode
a)Umcidoeumanidrido b)Umaaminaeumanidrido
c)Umcidoeumster d)Umaaminaeumlcool
e)Umcidoeumlcool
QUESTO10
Umcompostoorgnicofoisubmetidoaanliseporespectrometriademas-
sas(EM)porinfravermelho(IV)eporressonnciamagnticadecarbono-13
(RMN-
13
C)easseguintesinformaesforamobtidas:
I)PorEM,suamassafoidefnidacomosendo10
2
g.mol
-1
.
II)PorIVconstatou-seapresenadeumacarbonila
III)PorRMN-
13
C,deduziu-sequeamolculacontm2carbonosmetlicos
(CH
3
),2carbonosmetilnicos(CH
2
)eumcarbononohidrogenado
B
Exames Mod. B
| 61
Olimpada Brasileira
de Qumica 2011
Ano Internacional de Qumica
Essecompostopodeser:
a)Obutanoatodemetila b)Apentanona
c)O2-metilpropanoatodemetila d)Ocidopentanico
e)O2-hidroxi-3-metil-butanal
Gabarito:
1 2 3 4 5 6 7 8 9 10
C C B D A E C E B A
PARTEB-QUESTESANALTICO-EXPOSITIVAS
QUESTES11a14
VerasmesmasquestespropostasparaaModalidadeA.
QUESTO15
Omentol,tambmdenominadodecnforadehortel-pimenta,umasubs-
tncia que apresenta um sabor de menta e que encontrada em alguns
leos essenciais, como por exemplo: o leo de hortel-pimenta. O nome
sistemtico(IUPAC)domentol:2-isopropil-5-metilciclohexanol.
a)Desenheaestruturadomentolsemlevaremcontaaestereoqumica
b)Escrevaafrmulamoleculardomentol
c)Indiquequaissooscarbonosassimtricospresentesnamolcula
d)Indiqueonmeropossveldeestereoismeros
e)SabendoqueocarbonoligadohidroxilatemconfguraoRequeos
gruposmetilaeisopropilaesto,respectivamente,emposioCISeposio
TRANSemrelaohidroxila,faaumnovodesenhodaestruturadomentol,
agoracomaestereoqumicacorrreta.
QUESTO16
Umhidrocarbonetoinsaturado(compostoA)produz,porozonlise,propa-
nonae2-metilpropanal.
a)EscrevaaestruturaeonomedocompostoA
b)Escrevaasestruturaseosnomesdoscompostosorgnicosproduzidosnas
reaesdocompostoAcom:
b.1)B
2
H
6
,seguidodeZn,H
+
. b.2)H
2
/Pd
b.3)KMnO
4
,OH-,afrio b.4)H
2
O,H
+
b.5)Br
2
,H
2
O
Exames Mod. B
| 63 62 |
Programa Nacional
Olimpadas de Qumica
Ano Internacional de Qumica
Questo 11 Modalidade A - Resoluo de Daniel Arjona
de Andrade Hara, Objetivo - Cotia.
TemosA=Na,B=SeC=Ca
A1)OmaiorraioatmicodoCa(elementoC)jqueestlocalizadonoquar-
toperodo,enquantoosoutrosestolocalizadosnoterceiroperodo.
A2)MaiorcarganuclearefetivadoS(elementoB),poisestmaisadireita
natabelaperidica,oseltronsmaisinternostemmaioratraocomo
ncleo,logotermaiorcapacidadedeacomodarcargasnegativas.
A3) Maior carga nuclear efetiva do S faz com que seus eltrons estejam
maisligadoscomoncleologonecessriomaisenergiaparaarrancar
eltronsdoS,secompararmoscomNaeCa.
B1)OsdioformactionsNa
+
eoenxfreformanionsS
2-
logoafrmula
docompostoNa
2
S.
B2)OenxfreformadiversoscompostoscovalentescomoforcomoSF
2
,
SF
4
eSF
6
.SegundoaregradooctetootomodeSfariaduasligaes
covalentescomofor,resultandonaseguinteestrutura,F-S-F.
B3)OclcioformaCa
2+
ooxignioformanionsO
2-
,logoocompostoCaO.
Questo 12 Modalidade B - Resoluo de Bruno Lima-
verde Vilar Lbo, Farias Brito - Fortaleza.
A)
Questo 12 modalidade B
Cdigo 133

A)
I - NH
4
+

N
H
H
H
H
+

B.1 - sp
3
B.2 - nenhum par de eltrons no ligantes.
B.3 - 4 ligaes sigma ( ) e nenhuma pi ( ).
B.4 - Tetradrica

II - PCl
3
P Cl Cl
Cl

B.1 - sp
3
B.2 - 1 par de eltrons no ligantes.
B.3 - 3 ligaes sigma ( ) e nenhuma pi ( ).
B.4 - Piramidal

III -CO
3
2-
C
O
O O
2-

B.1 - sp
2
B.2 - nenhum par de eltrons no ligantes.
Questes resolvidas I
| 63
Olimpada Brasileira
de Qumica 2011
Ano Internacional de Qumica
Questes resolvidas I
B.1-sp
3
B.2-nenhumpardeeltronsnoligantes.
B.3-4ligaessigma()enenhumapi().
B.4-Tetradrica
II-PCl
3
Questo 12 modalidade B
Cdigo 133

A)
I - NH
4
+

N
H
H
H
H
+

B.1 - sp
3
B.2 - nenhum par de eltrons no ligantes.
B.3 - 4 ligaes sigma ( ) e nenhuma pi ( ).
B.4 - Tetradrica

II - PCl
3
P Cl Cl
Cl

B.1 - sp
3
B.2 - 1 par de eltrons no ligantes.
B.3 - 3 ligaes sigma ( ) e nenhuma pi ( ).
B.4 - Piramidal

III -CO
3
2-
C
O
O O
2-

B.1 - sp
2
B.2 - nenhum par de eltrons no ligantes.

B.1-sp
3
B.2-1pardeeltronsnoligantes.
B.3-3ligaessigma()enenhumapi().
B.4-Piramidal
III-CO
3
2-

Questo 12 modalidade B
Cdigo 133

A)
I - NH
4
+

N
H
H
H
H
+

B.1 - sp
3
B.2 - nenhum par de eltrons no ligantes.
B.3 - 4 ligaes sigma ( ) e nenhuma pi ( ).
B.4 - Tetradrica

II - PCl
3
P Cl Cl
Cl

B.1 - sp
3
B.2 - 1 par de eltrons no ligantes.
B.3 - 3 ligaes sigma ( ) e nenhuma pi ( ).
B.4 - Piramidal

III -CO
3
2-
C
O
O O
2-

B.1 - sp
2
B.2 - nenhum par de eltrons no ligantes.
B.1-sp
2
B.2-nenhumpardeeltronsnoligantes.
B.3-3ligaessigma()e1pi().
B.4-TrigonalPlanar
V-BF
4
-
B.3 - 3 ligaes sigma ( ) e 1 pi ( ).
B.4 - Trigonal Planar
V - BF
4
-
B F
F
F
F
-

B.1- sp
3
B.2 - nenhum par de eltrons no ligantes.
B.3 - 4 ligaes sigma ( ) e nenhuma pi ( ).
B.4 - Tetradrica.


















B.1-sp
3
| 65 64 |
Programa Nacional
Olimpadas de Qumica
Ano Internacional de Qumica
B.2-nenhumpardeeltronsnoligantes.
B.3-4ligaessigma()enenhumapi().
B.4-Tetradrica.
Questo 13 Modalidade B - Resoluo de Lara Mulato
Lima, Ari de S - Fortaleza
Questo 13 Modalidade B
Inscrio 505

a)
a.1) t
c
= K
c
W
K
c
= 1,86 Kg mol X = macromolcula
M
x
= 4 10
5
g mol m
x
= 8 10
7
Kg
mH
2
O = 0,01 Kg n
x
= 2 10
7
molal
I) W =
n
x
mH
2
O
=
210
9
mol
0,01 Kg
= 2 10
7
molal
II) t
c
= 1,86 2 10
7
= 3,72 10
7

III) t
c
= 273,15 3,72 10
7
= 273,1499996 K

a.2) = M. R. T T = 298 K dH
2
O = 1 g mL vH
2
O
=
2 10
9
0,01
0,082 298
= 4,887 10
6
atm

b) Nova presso osmtica = 1,364 10
4
atm
= M. R. T
M =

RT
= 5,5846 10
6
mol L
n
x
= 5,5846 10
6
10
6
0,01 = 5,5846 10
8
mol
M
x
=
m
x
n
x
M
x
=
8 10
4
g
5,5846 10
8
mol
= 1,4325 10
4
g mol









Questo 13 Modalidade B
Questes resolvidas I
| 65
Olimpada Brasileira
de Qumica 2011
Ano Internacional de Qumica
Questes resolvidas I
Questo 14 Modalidade B - Resoluo de Carlos Eduar-
do Grivol Jnior, Anchieta - Salvador
a) 1SiO
2(s)
+3C
(s)
1SiC
(s)
+2C
(g)

b) M
1
= 28,1 +2 16 = 60,1 g mol de SiO
2

M
2
= 12 g mol de C
(s)

m
1
= 6,01 g
m
2
= 7,2 g
60,1 =
6,01
n
1
n
1
= 0,1 mol de SiO
2

12 =
7,2
n
2
n
2
= 0,6 mol de C
(s)

Como a proporo do reagente SiO


2(s)
para o reagente C
(S)
de 1:3,
obeserva-se que 0,1 mol de SiO
2(s)
ir reagir com 0,3 mol de C(s)
apenas. Desse modo, C
(S)
est em excesso e o reagente limitante
SiO
2(s)
.

c) Deacordocomaequaobalanceada,sehouverconsumocompleto
de0,1moldeSiO
2(s)
,serproduzido0,1moldeC
(S)
M
X
= 28,1 +12 = 40,1 g mol de carborudum
40,1 =
m
x
0,1
m
x
= 4,01 de carborudum

d) Como o reagente limitante determinou que apenas 0,3 mols de C(s)


reagiu,podemoscalcularseuexcesso:
n = 0,6 0,3 n = 0,3 mol de C
(s)

12 =
m
r
0,3
m
r
= 3,6 g de C
(s)

Restarumamassade3,6gdeC
(S)
emexcesso.

e) Jfoicalculadoacimaquecomumrendimentode100%ser
produzidaumamassade4,01gdeSiO
2(s),
portanto:
100
4,01
=
p
2,56
p = 63,84%
Orendimentodareaoserde63,84%.

| 67 66 |
Programa Nacional
Olimpadas de Qumica
Ano Internacional de Qumica
a) 1SiO
2(s)
+3C
(s)
1SiC
(s)
+2C
(g)

b) M
1
= 28,1 +2 16 = 60,1 g mol de SiO
2

M
2
= 12 g mol de C
(s)

m
1
= 6,01 g
m
2
= 7,2 g
60,1 =
6,01
n
1
n
1
= 0,1 mol de SiO
2

12 =
7,2
n
2
n
2
= 0,6 mol de C
(s)

Como a proporo do reagente SiO


2(s)
para o reagente C
(S)
de 1:3,
obeserva-se que 0,1 mol de SiO
2(s)
ir reagir com 0,3 mol de C(s)
apenas. Desse modo, C
(S)
est em excesso e o reagente limitante
SiO
2(s)
.

c) Deacordocomaequaobalanceada,sehouverconsumocompleto
de0,1moldeSiO
2(s)
,serproduzido0,1moldeC
(S)
M
X
= 28,1 +12 = 40,1 g mol de carborudum
40,1 =
m
x
0,1
m
x
= 4,01 de carborudum

d) Como o reagente limitante determinou que apenas 0,3 mols de C(s)


reagiu,podemoscalcularseuexcesso:
n = 0,6 0,3 n = 0,3 mol de C
(s)

12 =
m
r
0,3
m
r
= 3,6 g de C
(s)

Restarumamassade3,6gdeC
(S)
emexcesso.

e) Jfoicalculadoacimaquecomumrendimentode100%ser
produzidaumamassade4,01gdeSiO
2(s),
portanto:
100
4,01
=
p
2,56
p = 63,84%
Orendimentodareaoserde63,84%.

Questes resolvidas I
| 67
Olimpada Brasileira
de Qumica 2011
Ano Internacional de Qumica
Questo 15
Mod. B - Resoluo de Bruno Limaverde Vilar
Lbo, Farias Brito - Fortaleza
a)
Clculodasconcentraesdosgases:
[NO]=2,0x10
-3
mol/0,05L=0,04molL
-1

[Br
2
]=4,0x10
-3
mol/0,05L=0,08molL
-1

[BrNO]=4,0x10
-3
mol/0,05L=0,08molL
-1

ClculodoQc,quocientedereao:
Qc=[BrNO]
2
/[NO]
2
.[Br
2
]
Qc=(0,08molL
-1
)
2
/(0,04molL
-1
)
2
.(0,08molL
-1
)
Qc = 50 mol L
-1
(Como Qc > Kc, a reao ocorrer no sentido dos
reagentes,demodoqueQc=Kc)

b)Kp=Kc(RT)
n
Kp=0,21x(0,082x623)
-1
Kp=4,11x10
-3

c)
ClculodasconcentraesdeNOedeBrNO:
[NO]=1,4x10
-3
mol/0,05L=0,028molL
-1

[BrNO]=1,4x10
-4
/0,05L=0,0028molL
-1
ClculodaconcentraodeBr
2
:
Kc=[BrNO]
2
/[NO]
2
.[Br
2
]
[Br
2
]=[BrNO]
2
/Kc.[NO]
2

[Br
2
]=(0,0028molL
-1
)
2
/0,21molL
-1
.(0,028molL
-1
)
2

[Br
2
]=0,0476molL
-1

d)
d.1) Ao adicionarmos o gs inerte, no ocorrer mudanas nas
concentraesdoscomponentesdamistura,poisnemasquantidadesdos
gaseseosvolumesdosrecipientessoalterados.
d.2) O equilbrio no ser deslocado, pois como as concentraes dos
gases participantes do equilbrio permanecem constantes, Qc permanece
igualaKc.
Questes resolvidas I
| 69 68 |
Programa Nacional
Olimpadas de Qumica
Ano Internacional de Qumica
a)
Clculodasconcentraesdosgases:
[NO]=2,0x10
-3
mol/0,05L=0,04molL
-1

[Br
2
]=4,0x10
-3
mol/0,05L=0,08molL
-1

[BrNO]=4,0x10
-3
mol/0,05L=0,08molL
-1

ClculodoQc,quocientedereao:
Qc=[BrNO]
2
/[NO]
2
.[Br
2
]
Qc=(0,08molL
-1
)
2
/(0,04molL
-1
)
2
.(0,08molL
-1
)
Qc = 50 mol L
-1
(Como Qc > Kc, a reao ocorrer no sentido dos
reagentes,demodoqueQc=Kc)

b)Kp=Kc(RT)
n
Kp=0,21x(0,082x623)
-1
Kp=4,11x10
-3

c)
ClculodasconcentraesdeNOedeBrNO:
[NO]=1,4x10
-3
mol/0,05L=0,028molL
-1

[BrNO]=1,4x10
-4
/0,05L=0,0028molL
-1
ClculodaconcentraodeBr
2
:
Kc=[BrNO]
2
/[NO]
2
.[Br
2
]
[Br
2
]=[BrNO]
2
/Kc.[NO]
2

[Br
2
]=(0,0028molL
-1
)
2
/0,21molL
-1
.(0,028molL
-1
)
2

[Br
2
]=0,0476molL
-1

d)
d.1) Ao adicionarmos o gs inerte, no ocorrer mudanas nas
concentraesdoscomponentesdamistura,poisnemasquantidadesdos
gaseseosvolumesdosrecipientessoalterados.
d.2) O equilbrio no ser deslocado, pois como as concentraes dos
gases participantes do equilbrio permanecem constantes, Qc permanece
igualaKc.
Questes resolvidas I
| 69
Olimpada Brasileira
de Qumica 2011
Ano Internacional de Qumica
Questo 15
Mod.B - Resoluo de Francisco Rodrigues da
Cruz Jnior, Dom Barreto - Teresina

a)

b)MentolC
10
H
22
O

c)
O2carbonodacadeiaprincipal
d)2
n
=2
1
=2estereoismeros

Questo15ModalidadeB
Cdigo:441

OH

OH
2
Questes resolvidas I
| 71 70 |
Programa Nacional
Olimpadas de Qumica
Ano Internacional de Qumica
Questo 16 Modalidade A - Resoluo de Nathrcia Cas-
tro Mota, Master - Fortaleza
Porquequandoumasoluoaquosadissocia-seemNa
+
eOH
-
,oNa
+
ser
hidratado,jquevriashidroxilas(devidoosemeioaquoso)seagruparam
a) Porquequandoumasoluoaquosadissocia-seemNa
+
eOH
-
,oNa
+
serhidratado,jque
vrias hidroxilas (devido o se meio aquoso) se agruparam ao redor do Na
+
hidratado
dificultandosualocomoo.

b) NaOH+HClNaCl+H
2
O
25mLdeNaOH(aq)
HCl0,25mol/L22,5mL 0,25mol1000mL
x22,5mL
x=5,625x10
-3
mol

1moldeNaOH1moldeHCl
y5,625x10
-3
moldeHCl
y=0,225moldeNaOH
M =
m
MM
=
0,225
40
= 5,625 10
3
mol de NaOH
M =
n
V
=
5,62510
3
2510
3
= 0,225 M
Resposta:No,jqueaconcentraocorreta0,225mol/L.

c) [ ] =
n
v
0,25 mol L =
n
0,250
n = 0,0625 mol
n =
m
MM
0,0625 =
m
40
= 2,5 g de NaOH
Ento,acrescentando2,5gdeNaOHemumbalocontendo250mLdeguadestilada.

aoredordoNa
+
hidratadodifcultandosualocomoo.
Resposta:No,jqueaconcentraocorreta0,225mol/L.
Ento,acrescentando2,5gdeNaOHemumbalocontendo250mLdegua
destilada.
a) Porquequandoumasoluoaquosadissocia-seemNa
+
eOH
-
,oNa
+
serhidratado,jque
vrias hidroxilas (devido o se meio aquoso) se agruparam ao redor do Na
+
hidratado
dificultandosualocomoo.

b) NaOH+HClNaCl+H
2
O
25mLdeNaOH(aq)
HCl0,25mol/L22,5mL 0,25mol1000mL
x22,5mL
x=5,625x10
-3
mol

1moldeNaOH1moldeHCl
y5,625x10
-3
moldeHCl
y=0,225moldeNaOH
M =
m
MM
=
0,225
40
= 5,625 10
3
mol de NaOH
M =
n
V
=
5,62510
3
2510
3
= 0,225 M
Resposta:No,jqueaconcentraocorreta0,225mol/L.

c) [ ] =
n
v
0,25 mol L =
n
0,250
n = 0,0625 mol
n =
m
MM
0,0625 =
m
40
= 2,5 g de NaOH
Ento,acrescentando2,5gdeNaOHemumbalocontendo250mLdeguadestilada.

a) Porquequandoumasoluoaquosadissocia-seemNa
+
eOH
-
,oNa
+
serhidratado,jque
vrias hidroxilas (devido o se meio aquoso) se agruparam ao redor do Na
+
hidratado
dificultandosualocomoo.

b) NaOH+HClNaCl+H
2
O
25mLdeNaOH(aq)
HCl0,25mol/L22,5mL 0,25mol1000mL
x22,5mL
x=5,625x10
-3
mol

1moldeNaOH1moldeHCl
y5,625x10
-3
moldeHCl
y=0,225moldeNaOH
M =
m
MM
=
0,225
40
= 5,625 10
3
mol de NaOH
M =
n
V
=
5,62510
3
2510
3
= 0,225 M
Resposta:No,jqueaconcentraocorreta0,225mol/L.

c) [ ] =
n
v
0,25 mol L =
n
0,250
n = 0,0625 mol
n =
m
MM
0,0625 =
m
40
= 2,5 g de NaOH
Ento,acrescentando2,5gdeNaOHemumbalocontendo250mLdeguadestilada.

Questes resolvidas I
| 71
Olimpada Brasileira
de Qumica 2011
Ano Internacional de Qumica
Questo 16 Modalidade A - Resoluo de Guilherme
Costa G. Fernandes, Colgio Militar - Braslia
a) Porquequandoumasoluoaquosadissocia-seemNa
+
eOH
-
,oNa
+
serhidratado,jque
vrias hidroxilas (devido o se meio aquoso) se agruparam ao redor do Na
+
hidratado
dificultandosualocomoo.

b) NaOH+HClNaCl+H
2
O
25mLdeNaOH(aq)
HCl0,25mol/L22,5mL 0,25mol1000mL
x22,5mL
x=5,625x10
-3
mol

1moldeNaOH1moldeHCl
y5,625x10
-3
moldeHCl
y=0,225moldeNaOH
M =
m
MM
=
0,225
40
= 5,625 10
3
mol de NaOH
M =
n
V
=
5,62510
3
2510
3
= 0,225 M
Resposta:No,jqueaconcentraocorreta0,225mol/L.

c) [ ] =
n
v
0,25 mol L =
n
0,250
n = 0,0625 mol
n =
m
MM
0,0625 =
m
40
= 2,5 g de NaOH
Ento,acrescentando2,5gdeNaOHemumbalocontendo250mLdeguadestilada.

a) Sabendoqueaozonlisedealcenosquebraaduplaligao,formando
dois compostos carbonilados, o composto A tem a seguinte frmula
estrutural:


Nomenclatura: 2,4-dimetilpent-2-eno

b)
b.2)Reaodehidrogenaocataltica:

H
2
Pd
2,4-dimetilpentano

b.3)Oxidaobranda

KMnO
4
OH, frio
OH
OH
2,4-dimetilpentano-2,3-diol
b.4)Adiodeguaemmeiocido
A ligao se desloca preferencialmente para o carbono 3, pois
assimformadoocarboction,tercirioquerelativamenteestvel


OH
2,4-dimetil-pent-2-ol

b.5)Adiodebromoaquoso
Emguaobromosofreheterlise:


Br
2
(aq)
Br
Br

A ligao do alceno se desloca analogamente ao exemplo


anterior,paraqueacargapositivafiquemaisaliviada
a) Sabendoqueaozonlisedealcenosquebraaduplaligao,formando
dois compostos carbonilados, o composto A tem a seguinte frmula
estrutural:


Nomenclatura: 2,4-dimetilpent-2-eno

b)
b.2)Reaodehidrogenaocataltica:

H
2
Pd
2,4-dimetilpentano

b.3)Oxidaobranda

KMnO
4
OH, frio
OH
OH
2,4-dimetilpentano-2,3-diol
b.4)Adiodeguaemmeiocido
A ligao se desloca preferencialmente para o carbono 3, pois
assimformadoocarboction,tercirioquerelativamenteestvel


OH
2,4-dimetil-pent-2-ol

b.5)Adiodebromoaquoso
Emguaobromosofreheterlise:


Br
2
(aq)
Br
Br

A ligao do alceno se desloca analogamente ao exemplo


anterior,paraqueacargapositivafiquemaisaliviada
a) Sabendoqueaozonlisedealcenosquebraaduplaligao,formando
dois compostos carbonilados, o composto A tem a seguinte frmula
estrutural:


Nomenclatura: 2,4-dimetilpent-2-eno

b)
b.2)Reaodehidrogenaocataltica:

H
2
Pd
2,4-dimetilpentano

b.3)Oxidaobranda

KMnO
4
OH, frio
OH
OH
2,4-dimetilpentano-2,3-diol
b.4)Adiodeguaemmeiocido
A ligao se desloca preferencialmente para o carbono 3, pois
assimformadoocarboction,tercirioquerelativamenteestvel


OH
2,4-dimetil-pent-2-ol

b.5)Adiodebromoaquoso
Emguaobromosofreheterlise:


Br
2
(aq)
Br
Br

A ligao do alceno se desloca analogamente ao exemplo


anterior,paraqueacargapositivafiquemaisaliviada
Questes resolvidas I
| 73 72 |
Programa Nacional
Olimpadas de Qumica
Ano Internacional de Qumica
a) Sabendoqueaozonlisedealcenosquebraaduplaligao,formando
dois compostos carbonilados, o composto A tem a seguinte frmula
estrutural:


Nomenclatura: 2,4-dimetilpent-2-eno

b)
b.2)Reaodehidrogenaocataltica:

H
2
Pd
2,4-dimetilpentano

b.3)Oxidaobranda

KMnO
4
OH, frio
OH
OH
2,4-dimetilpentano-2,3-diol
b.4)Adiodeguaemmeiocido
A ligao se desloca preferencialmente para o carbono 3, pois
assimformadoocarboction,tercirioquerelativamenteestvel


OH
2,4-dimetil-pent-2-ol

b.5)Adiodebromoaquoso
Emguaobromosofreheterlise:


Br
2
(aq)
Br
Br

A ligao do alceno se desloca analogamente ao exemplo


anterior,paraqueacargapositivafiquemaisaliviada
Questes resolvidas I
| 73
Olimpada Brasileira
de Qumica 2011
Ano Internacional de Qumica
OlimpadaBrasileiradeQumica-2011
ModalidadeA
OURO
BrunoMatissekWorm Tiradentes PortoAlegre RS
VitriaNunesMedeiros FariasBrito Fortaleza CE
LeonardodeAndradeMesquita Militar PortoAlegre RS
ViniciusLopesBraga DomBarreto Teresina PI
EmersonHolandaMarinho FariasBrito Fortaleza CE
JlioCsarPradoSoares Militar Braslia DF
PRATA
VictorSantosdeAndrade DomBarreto Teresina PI
DanielArjonadeAndradeHara Objetivo Cotia SP
FranciscoDaviBarbosadosSantos FariasBrito Fortaleza CE
JoaquimIvoVasquesD.Landim Paraiso J.doNorte CE
MatheusSalmitoRodriguesPonte FariasBrito Fortaleza CE
GabrielMatheusVianaPinheiro Mster Fortaleza CE
LviaRodriguesdeArajo FariasBrito Fortaleza CE
AndrSantosFernandes AlbertSabin Osasco SP
MarianaCamylaDuartePontes FariasBrito Fortaleza CE
SergioPereiradeOliveiraJnior FariasBrito Fortaleza CE
RogrioYuukiMotisuki Etapa SoPaulo SP
BrandonWahibBogossianKhalil IF-RJ RiodeJaneiro RJ
JooPedroAlexandreSilvaMota FariasBrito Fortaleza CE
RaissaNiutaFreitasdeOliveira 7desetembro Fortaleza CE
BRONZE
GabrielBerlingieriPolho Renovao SoPaulo SP
LaioLadislauLopesLima FariasBrito Fortaleza CE
FilipeHersonCarneiroRios FariasBrito Fortaleza CE
RenatoFrankileydaSilvaLima EREMProf.AdautoCarvalho Recife PE
NicholasdeSouzaCostaLima ArideS Fortaleza CE
RamonSantosGonalvesdaSilva IdealMilitar Belm PA
A
Resultado - Fase III
| 75 74 |
Programa Nacional
Olimpadas de Qumica
Ano Internacional de Qumica
KelvinAzevedodosSantos UniclassObjetivo Goinia GO
DboraBarretoOrnellas Anchieta Salvador BA
IohannaMoreiraNunesRibeiro IF-RJ Paracambi RJ
ViniciusLimaFerreira IF-RJ RiodeJaneiro RJ
NathrciaCastroMota Master Fortaleza CE
GiuvanniMutton Etapa SoPaulo SP
IsabelTheresadeHolandaFreitas COLUNI Viosa MG
HenriqueMagriMaron Etapa Valinhos SP
RenataSantosMartins DomBarreto Teresina PI
LeonardoKazunoriTsuji Etapa SoPaulo SP
PedroHenriqueAlmeidaFraiman CinciasAplicadas Natal RN
MENOHONROSA
BrenoCassaroMouro IF-RJ Nilpolis RJ
ViniciusAlvesDuartedaSilva IF-RJ Maracan RJ
PedroSilveiraQuintana PolitcnicodaUFSM SantaMaria RS
RomulloRandellMacedoCarvalho Lavoisier Teresina PI
DrvylladeSousaLima Teleyos Fortaleza CE
EnochYang PresbiterianoM.Tambor Sant.deParaiba SP
NiltonLealArajo DomBarreto Teresina PI
JosIvanF.deO.Neto Ideal Belm PA
MarcusVinciusMartinsdeAlmeida Etapa SoPaulo SP
AlexandreAndradeC.deAlmeida Col.deAplicao-UFPE Recife PE
carodeAlmeidaVaro DomBarreto Teresina PI
GabrielAugustoGinja Etapa SoPaulo SP
HenriqueGaspariniF.Nascimento Militar Braslia DF
DrieleNevesRibeiro Col.deAplicao-UFPE Recife PE
caroSampaioViana 7deSetembro Fortaleza CE
DimitriReisdeMatos Anchieta Salvador BA
RenanFerreirinhaCarneiro Militar RiodeJaneiro RJ
RafaelRibeiroAlves Motiva CampinaGrande PB
AnaRaquelFerreiradeAzevedo FariasBrito Fortaleza CE
JssicaSilvaLopes FariasBrito Fortaleza CE
BernardoGuimaresL.deCarvalho SantoAntnio BeloHorizonte MG
RayssaOliveiradoNascimento IF-RJ RiodeJaneiro RJ
EduardoPoleze Bertoni Medianeira PR
Resultado - Fase III
| 75
Olimpada Brasileira
de Qumica 2011
Ano Internacional de Qumica
FlvioLuisSchneiderJunior Militar Manaus AM
GustavoRosaGameiro Universitario Londrina PR
LeonardoHautriveMedeiros PolitcnicodaUFSM SantaMaria RS
LauraMartinsPinadeAlmeida IF-RJ RiodeJaneiro RJ
VitorJucPolicarpo 7desetembro Fortaleza CE
DanieldeOliveiraS.VasconceloseS GGE Recife PE
LucasHenriqueCarvalhoFurquim DomBarreto Teresina PI
MatheusBrumMoraes Ateneu Londrina PR
VictorSousaSilva ArideS Fortaleza CE
RenataSegattoPigatto PolitcnicodaUFSM SantaMaria RS
BrunoAlvesCaremiro CEFET RiodeJaneiro RJ
LuizFelipehAguiardeLimaAlves Militar RiodeJaneiro RJ
LucasGabrieldeBarrosSilva Militar PortoAlegre RS
GustavoXavierdeJesus Militar RiodeJaneiro RJ
RamonSilvadeLima Objetivo SoPaulo SP
AndrezaSaboiaDantas CinciasAplicadas Natal RN
GeanAlmeidadaSilva IF-RJ RiodeJaneiro RJ
IsraelWilsonNobreBarreto Master Fortaleza CE
RogrioJosMenezesAlves CharlesDarwin VilaVelha ES
RaphaelSouzadeAlmeida Alternativo Aracaju SE
EmanoelaBitencourtVarjo Amadeus Aracaju SE
GabrielAntnioFloresChies LiberatoSalzano NovoHamburgo RS
FelipeBrandoForte FariasBrito Fortaleza CE
RodrigoCastiel Motivo Recife PE
DiegoTartgliaL.deAndrade PedroII RiodeJaneiro RJ
MateusJucPinheiro 7desetembro Fortaleza CE
JooVictorBurlamaquiCoelho Crescimento SoLus MA
HaroldoNogueiraVictorianoNeto DuliaBringel Fortaleza CE
PatrickyAlbanideSouza CEEProfssionaldeCuritiba Curitiba PR
MariaElizaMachadoRomeros CEFET BeloHorizonte MG
MarinaMeloSousaMendesLeal Sag.CoraodeJesus Teresina PI
VitriaEspindolaLeiteBorges SantoAgostinho Goinia GO
RafaelReinaldoGracezNedel MariaAuxiliadora Canoas RS
BrunodeLimaSantos IFES C.deItapemirim ES
AurimarBezerraM.deSousaFilho DomBarreto Teresina PI
AnaBeariceBonganhaZanon UnidadeJardim SantoAndr SP
Resultado - Fase III
| 77 76 |
Programa Nacional
Olimpadas de Qumica
Ano Internacional de Qumica
JosValnirTeixeiraCruz Sag.CoraodeJesus Teresina PI
PauloAugustodePaivaSilva IFET Niteri MG
VictorVenturi Etapa Campinas SP
MatheuCoelhoFerraz Etapa SoPaulo SP
ErickLeonardodeSouzaMonteiro FundaoNokia Manaus AM
DiegoRodriguesCarvalho AntonioVieira Salvador BA
EmilleRenalleFreitasdeMelo Motiva CampinaGrande PB
PedroGuimaresMartins Bernoulli BeloHorizonte MG
MatheusCompartHemerly CharlesDarwin Vitria ES
CarolinaKonrdorferRangel LiberatoSalzano NovoHamburgo RS
MatheusdeMouraAmaral CharlesDarwin Anchieta ES
HenriqueMartinsTavares MaristaRosrio PortoAlegre RS
NicolasEduardodaFonsecaFarias Motiva JooPessoa PB
MatheusRocha CharlesDarwin Vitria ES
FagnerLuciodeToledo COLUNI Viosa MG
AbdonMoutinho Exponente Recife PE
RenatoXavierdeJesus Militar RiodeJaneiro RJ
FernandoAntnioSaraivaMaia DuliaBringel Fortaleza CE
FilipeFerracioli Etapa Vinheda SP
LucasStefanMinuzziNeumann PolitcnicodaUFSM SantaMaria RS
FilipeMouroLeite DomBarreto Teresina PI
GabrielHenriqueKuupfer SENAINorteII Joinville SC
MarianaLciaBrancoZeitune IF-RJ RiodeJaneiro RJ
FranciscoMarkanNobredeSouza FariasBrito Fortaleza CE
VitorSouzaGuimares SoFranciscoXavier BeloHorizonte MG
AnaBeatrizMarques IFET Barbacena MG
HelosaMariaMachadoMassaro BrunoGiorgi Mococa SP
CamiladeLimaMagalhes Olimpo Goinia GO
ThiagoPereiraCanoco 7deSetembro Fortaleza CE
GuilhermedaSilvaArajo CEFET BeloHorizonte MG
BartiraMagalhesRodrigues Modulo Salvador BA
IvanTadeuF.AntunesFilho ObjetivoPaulista Lins SP
LeonardoYujiMiakawa Etapa SoPaulo SP
ValriaKarinedeAzevedoFerreira LatoSensu Manaus AM
ElaineCosta Militar Recife PE
DanielCostaXavierdeOliveira Olimpo Goinia GO
Resultado - Fase III
| 77
Olimpada Brasileira
de Qumica 2011
Ano Internacional de Qumica
AlexMendesLeonelFreire Contato Macei AL
SamuelIncioOliveiraArajo ColgioArnaldo BeloHorizonte MG
CamilaPiovesanWiethan PolitcnicodaUFSM SantaMaria RS
WedersonSantosSilva Motiva CampinaGrande PB
RaphaelLimaSantos Alternativo Aracaju SE
VictordeMelloAlvesNunes IF-RJ RiodeJaneiro RJ
AnaCarolinaF.GondinGuimares COLUNI Viosa MG
ArthurShinitiCato Etapa SoPaulo SP
RodrigoLinsSantAnadeLima Anchieta Salvador BA
MarcioSantosSouzaCaroso SartreCoc Salvador BA
NaiaradeOliveiraBaptista Mster Fortaleza CE
GabrielJosMoreiradaCostaSilva Contato Macei AL
JulianyPiresFigueiredo LatoSensu Manaus AM
FelipeHenriqueMouraChupel Militar Braslia DF
TalesMesquitaMuniz FariasBrito Fortaleza CE
LarissaFarnettiPinto Coleguim BeloHorizonte MG
DiegoRodriguesdaPonte DuliaBringel Fortaleza CE
MatheusWenzelSGonalves DoSalvador Aracaju SE
IgorBarbozaCunha Salesiano Aracaju SE
MatheusFreitasRochaBastos MilitarDomPedroII Braslia DF
KoodyAndrHassemiKitawara Universitario Londrina PR
JooMarcosCNamosdaSlva Prevest Goinia GO
FellipeFranciscodaSilva IF-RJ Nilpolis RJ
JoanaLourenodeCarvalho Motiva JooPessoa PB
PedroIgncioNogueiraGes ArideS Fortaleza CE
AlanBarbosaCosta IF-AL Macei AL
PedroHenriqueG.Carvalho SantaCeclia Fortaleza CE
MariaEmanuellaMouraSilva GGE Recife PE
GabrielRezendeNahas Bionatus CampoGrade MS
WagnerThieleFracassi LiberatoSalzano NovoHamburgo RS
MarinaAlbuquerqueB.deAlmeida MaterChristi Mossor RN
JooPedroO.Freitas Salesiano Aracaju SE
JooPedroSilvaKirmse LeonardodaVinci Braslia DF
MorganaGaribaldiDiefenthaeler MaristaRosrio PortoAlegre RS
MuriloFreitasYonashiroCoelho Etapa SoPaulo SP
RafaelEllerCruz Sag.CoraodeMaria Braslia DF
Resultado - Fase III
| 79 78 |
Programa Nacional
Olimpadas de Qumica
Ano Internacional de Qumica
LucasRafaelMendesAlmeida Sag.CoraodeJesus Teresina PI
LivioMoreiraRios Crescimento SoLus MA
DiegoGuedesChaves Motiva JooPessoa PB
NiloParentePessoaDias FariasBrito Fortaleza CE
HeitorAugustoC.deAlmeida Motiva JooPessoa PB
MelineGabrieledePaulaSantos PedroII Niteri RJ
LeonardoEnricoM.Mendes Olimpo Braslia DF
MaradePaulaNunes MaristaDomSilvrio BeloHorizonte MG
LucasFernandoSilvaHess CEFET BeloHorizonte MG
ArmandoM.Nader Olimpo Goinia GO
MatheusRavelTimoBarbosa Militar Braslia DF
LusaNaomiCastroSuda SoFranciscoXavier BeloHorizonte MG
AmandaDuartedeSouza Senai Catalo GO
JulianaSchenhelZotti EEEMGustavoBiazus TupancidoSul RS
EzauSilvaRibeiro EliteViladosCabanos Belm PA
PedroOtviodeCarvalhoRamos DomBosco SoLus MA
DouglasMartinsCarneiro Cepron DomPedro MA
JooPedroMaiaSadi Loyola BeloHorizonte MG
CassianoSampaioCampana Maxi Londrina PR
GuilhermeHenriqueM.deAraujo Crescimento SoLus MA
SarahSegantiniCamposdeSouza SantaMariaPampulha BeloHorizonte MG
RodrigoDiasGarcia SartreCoc Salvador BA
VictorHugoQueirozRebello Nacional Vitria ES
UriahMartelliGlaza Acesso Curitiba PR
RicardoDognoniHudsmann SENAINorteII Joinville SC
RuanLucasJucAzevedo Elite Belm PA
AndrTaytSohnFerioli PedroII Niteri RJ
LaraRamosMonteiroSilva Crescimento SoLus MA
TiagodaSilvaNunes Arquidiocesano Aracaju SE
LucasGondimBriandVieira FariasBrito Fortaleza CE
EleodorioSalesBomfmNeto Motiva CampinaGrande PB
VictordeArajoRocha Anchieta Salvador BA
MarcusViniciusPessoadeAlmeida ArideS Fortaleza CE
ErosGimenesdeQueiroz LeonardodaVinci Vitria ES
LucasHage SantaBartolomea Macap AP
JulioCesarOliveiraL.Filho Batista SoLus MA
Resultado - Fase III
| 79
Olimpada Brasileira
de Qumica 2011
Ano Internacional de Qumica
MarcusDiFabianniF.LopesFilho Militar Manaus AM
WilsonSantosCraveiroRosa Lettera Teresina PI
OslioCndidodeArajoL.Lima FariasBrito Fortaleza CE
MarianaBragaLacerda Motiva JooPessoa PB
VitorHugoFernandesBreder DomBarreto Teresina PI
DanielFerreiraMatos SartreCoc Salvador BA
AnaBeatrizCostaCavalcanti GGE Recife PE
MurilloHenriquedeM.Rodrigues Senai Catalo GO
MateusHenriqueSchneider MaristaRosrio PortoAlegre RS
ThallesEmannuelBatistaPinheiro Motiva JooPessoa PB
JosRenatoNascimentoGomes Unifev Votuparanga SP
MatheusSonegoTemp PolitcnicodaUFSM SantaMaria RS
LeonardoJorgeBessaTajraFilho JuvenaldeCarvalho Fortaleza CE
NicolasChiuOgassavara Etapa SoPaulo SP
LuanadosAnjosAssis Arquidiocesano Aracaju SE
HricaCarolineMathiel Profa.JulietaMotaSantos Coscim MS
PedroPereiradeAlmeida Anchieta Salvador BA
WendreyLustosaCardoso Militar Manaus AM
GustavoHenriquedeM.F.Filho Contato Macei AL
KsiaPriscilaOmenaCardoso IF-AL Macei AL
HenriqueTrivisanRolo Positivo Curitiba PR
LuanaPompeudosSantosRocha DomBarreto Teresina PI
MariellyRezendeSantos IF-MT Cuiab MT
RafaelCamposdoNascimento SoFranciscodeAssis Apucarana PR
ArthurLiraAraujo Motiva CampinaGrande PB
Resultado - Fase III
| 81 80 |
Programa Nacional
Olimpadas de Qumica
Ano Internacional de Qumica
OlimpadaBrasileiradeQumica-2011
ModalidadeB
Nome Escola Cidade UF
OURO
TaynaraCarvalhoSilva Mster Fortaleza CE
GuilhermeCostaG.Fernandes Militar Braslia DF
CarolinadeSMendona Militar RiodeJaneiro RJ
JosMatheusG.deA.Bastos DomBarreto Teresina PI
AlexSilvadeCerqueira CEFET Salvador BA
IgorFranzoniOkuyama Olimpo Goinia GO
JooPedroCavalcantiPereira SantaMaria Recife PE
PRATA
ArthurEduardoPastoredeLima Medianeira Medianeira PR
MateusBragadeCarvalho DomBarreto Teresina PI
RaulBrunoMachadodaSilva FariasBrito Fortaleza CE
VitorLucenaCarneiro Olimpo Goinia GO
DaviRodriguesChaves ArideS Fortaleza CE
RenanLucasdaSilvaCustdio ArideS Fortaleza CE
LaraMulatoLima ArideS Fortaleza CE
FabriciodaSilvaCosta Militar RiodeJaneiro RJ
WilliamTutihashi Objetivo MogidasCruzes SP
VitorMarquesAssad CharlesDarwin Vitria ES
DaniloMoreiraSimes ObjetivoIntegrado SoBernardo SP
JayaneCarvalhoBorges Lettera Teresina PI
RyanGimenesdeSouza Militar RiodeJaneiro RJ
B
Resultado - Fase III
| 81
Olimpada Brasileira
de Qumica 2011
Ano Internacional de Qumica
BRONZE
MarcoAntoniodaC.Nascimento LatoSensu Manaus AM
WilliamTeixeiraMiranda CEFET BeloHorizonte MG
CarlosEduardoGrivolJnior Anchieta Salvador BA
MarcosVinciusNunesdeSouza GGE Recife PE
BrunoLimaverdeVilarLbo FariasBrito Fortaleza CE
DavidsonAnthonyAragoFreire ArideS Fortaleza CE
RafaelaGesMachado Anchieta Salvador BA
GreiceCalloni LiberatoSalzano N.Hamburgo RS
LetciaNunesdeOliveira DomBarreto Teresina PI
SlvioFurtadoXimenes DuliaBringel Fortaleza CE
GermanoBrunattoF.Miranda DomBosco Curitiba PR
FernandoBarbosaNeto Nacional VilaVelha ES
AlissondeSousaBarreto ArideS Fortaleza CE
RodrigoMedeirosGuercio Motiva JooPessoa PB
LeonardodosAnjosCunha MaterAmabilis Guarulhos SP
DanielGracianoCosta Expresso Goinia GO
IgorSantosGhignotti CharlesDarwin VilaVelha ES
PedroVictorBarbosaNolto DomBarreto Teresina PI
MENOHONROSA
FabianoMalarddeArajo Bernoulli BeloHorizonte MG
LuizFelipeGuainTeixeira SantaMaria SoPaulo SP
AlineTvoradaSilva Col.deAplicao-UFPE Recife PE
GuilhermeSerraBaima Col.Marista SoLus MA
JulianaBarbosaEchenique IF-RS Pelotas RS
LuizHenriqueFreireArajo Militar RiodeJaneiro RJ
MarinaTomsTeixeiraCarvalho CEFET BeloHorizonte MG
EmanuelCassoudosSantos BomJesusLourdes Curitiba PR
ArturCarvalhoSantos MaterAmabilis Guarulhos SP
Resultado - Fase III
| 83 82 |
Programa Nacional
Olimpadas de Qumica
Ano Internacional de Qumica
ricaSaldanhaFreireSimes ArideS Fortaleza CE
GabrieldeSouzaVieiraBi Olimpo Goinia GO
NatliaAragoDias Mster Fortaleza CE
LeonardoTessaroloSilva CharlesDarwin VilaVelha ES
RafaelTakeshiSasakiOkida Olimpo Braslia DF
PedroPauloC.A.Chaves MaristaSoJos RiodeJaneiro RJ
JliaResendeKanno Agostiniano Goinia GO
GabrielChallubPires Militar PortoAlegre RS
ricaReginaRodriguesdaSilva Cepron DomPedro MA
WeiTzonHackanChangColares LatoSensu Manaus AM
WladimirJosLopesMartins Militar Recife PE
BrenoSaldanhaSousa FariasBrito Fortaleza CE
RafaelFerreiraAntonioli UnidadeJardim SoBernardo SP
GustavoHaddadF.SampaioBraga Objetivo S.JosdosCampos SP
NathiannedeMouradeAndrade FariasBrito Fortaleza CE
DouglasRamalhoQ.Pacheco EliteValedoAo Ipatinga MG
PedroLobodeC.Nascimento Olimpo Goinia GO
SauloCzarO.Parreira Olimpo Goinia GO
RodolfoPradoTorres Mackenzie Braslia DF
AlissonBezerraGomes CEFET Salvador BA
MarcelePereiraDias Militar RiodeJaneiro RJ
RodrigoGomesDemeterko Militar RiodeJaneiro RJ
PauloAndrHerculanodeLima FariasBrito Fortaleza CE
JorgeJunioPedrosaJordo LatoSensu Manaus AM
PalomaLinsBarros Motiva JooPessoa PB
ArthurBernardoFloresMoreira IF-RJ RiodeJaneiro RJ
VirnaMendonaSampaioLima Anchieta Salvador BA
JosAlexandreAragoMelo IFS Aracaju SE
MarianeHigaShinzato Bionatus CampoGrande MS
Resultado - Fase III
| 83
Olimpada Brasileira
de Qumica 2011
Ano Internacional de Qumica
JaymeMendonaRamos Nacional Serra ES
FranciscoRodriguesdaCruzJr. DomBarreto Teresina PI
GabrielMoraesSilva IF-RJ RiodeJaneiro RJ
JosRibamarPereiraNeto DomBarreto Teresina PI
ThalesGalvoCosta IF-RJ RiodeJaneiro RJ
LucasKojiMatsuzaki Universitario Londrina PR
MarianaGabrieladeOliveira CEFET BeloHorizonte MG
GustavoHenriqueR.Guimares Bernoulli BeloHorizonte MG
MuriloVieiraSantaBrbara Olimpo Goinia GO
JooVictorFragosoDias LeonardodaVinci Vitria ES
MarcosAlcinoS.SiqueiraMarquesJr. DomBarreto Teresina PI
VinciusClevesdeOliveiraCarmo Helyos FeiradaSantana BA
ArthurChangNogueira Bernoulli BeloHorizonte MG
JosMarquesNeto ArideS Fortaleza CE
MariaGabrielaVianadeSa Motiva CampinaGrande PB
DiegoDelFabroKunzler LiberatoSalzano NovoHamburgo RS
NicolasKemerichdeMoura Objetivo Palmas TO
WilliamVeberMoissdaSilva Militar PortoAlegre RS
BrunoLunardiGonalves Militar Braslia DF
VictorMatheusGalvo BoaViagem Recife PE
PalomaAlvesBezerraMorais CharlesDarwin Vitria ES
LucasPietroGalvam BomJesusLourdes Curitiba PR
TainNobreBarrosRodrigues Master Fortaleza CE
HenriqueTonAzevedoGiacomin LeonardodaVinci Vitria ES
TadeuMenesesdeCarvalho DomBarreto Teresina PI
BrunodeOliveiraLima Col.deAplicao-UFPE Recife PE
AntenorTeixeiraNeto Anchieta Salvador BA
JeanLucasRamos Universitario Curitiba PR
IagoAlmeidaNeves Anchieta Salvador BA
Resultado - Fase III
| 85 84 |
Programa Nacional
Olimpadas de Qumica
Ano Internacional de Qumica
EduardoVinciusColmandaSilva Marista Cascavel PR
AmandaSaraCavalcanteSouza Expresso Apar.deGoinia GO
JoseVictorMachadoNascimento ArideS Fortaleza CE
JanainaGomesC.E.Mascarenhas DomBarreto Teresina PI
JooPedroCoelhodeSousaLima EliteValedoAo Ipatinga MG
VictorAugustoSouzadeOliveira IF-RJ RiodeJaneiro RJ
DanielRoediger Etapa SoPaulo SP
VictorHugoV.S.Oliveira IF-RJ RiodeJaneiro RJ
GleycaLazarinodeAlmeida EducaoCriativa BeloHorizonte MG
DouglasToledoCamilo CharlesDarwin Vitria ES
LydiaPearcePessoadeAguiar DomBarreto Teresina PI
TbataCludiaAmaraldePontes Etapa SoPaulo SP
AdinaildoGomesPaesJr. EliteViladosCabanos Belm PA
CaioEddiedeMeloAlves LatoSensu Manaus AM
AndrLopesEvangelistaDias MadreVillac Teresina PI
LarissaDuresCaminhas MagnumAgostiniano BeloHorizonte MG
RaphaelNercessianCorradini Alternativo Aracaju SE
IgorPittoldeCarvalho CharlesDarwin VilaVelha ES
JooLucasMaeharaSaiddosReis IFES C.deItapemirim ES
AlyneMaradeAlencarJusta ArideS Fortaleza CE
CarolinaCarvalhoTavares LatoSensu Manaus AM
MarianaBritodoCouto IF-RJ RiodeJaneiro RJ
IsabelLopesdaSilveira BomJesusLourdes Curitiba PR
LucianadeOliveiraBotelho Bernoulli BeloHorizonte MG
BrunoAlvesMotaRocha Motiva CampinaGrande PB
MartinaVitriaFlachDietrich LiberatoSalzano NovoHamburgo RS
WilsonVieiradaSilvaJnior DomBarreto Teresina PI
MatheusLealdeSouza AntonioVieira Salvador BA
MarianaSanchezMalagutti BomJesusCentro Curitiba PR
Resultado - Fase III
| 85
Olimpada Brasileira
de Qumica 2011
Ano Internacional de Qumica
MarianaFeitosaCustdio Cepron DomPedro MA
DiogoFarkattKabbaz GGE Recife PE
RaphaelGalateBaptistaRibeiro Poliedro RibeiroPires SP
MarianaLopesFerreiraGomes MilleniumClasse Goinia GO
JosLucasdeAlencarSaraiva Motivo Recife PE
AdrielGarciaMaquinSenado IF-AM Manaus AM
AndrCaranLima AngloVinci SantanadeParaiba SP
LuisFernandoEsteves LiberatoSalzano NovoHamburgo RS
GabrielF.P.Arajo Lavoisier Teresina PI
FelipeDuarteFreire PontodeEnsino RiodeJaneiro RJ
DanielAbrantesFormiga Motiva JooPessoa PB
BiancaRohsnerBezerra FariasBrito Fortaleza CE
LeonardoBressanAnizelli MaristadeCascavel Toledo PR
TatianaNovaesTheoto LeonardodaVinci Jundia SP
JairodeSousaAraujo ANGLO Teresina PI
MatheusHenriqueD.Menezes Agostiniano Goinia GO
JooHenriquedeA.V.Sistema ELITEdeEnsino RiodeJaneiro RJ
MatheusSalgueiroCastanho CharlesDarwin VilaVelha ES
PaulaNascimentoChoinski BomJesusLourdes Curitiba PR
KalilLimaJardimFerraz SantaEmlia Olinda PE
AlineLimaAlves Bernoulli BeloHorizonte MG
MatheusBragaFurstenberger NsaSradasNeves Parnamirim RN
GustavoCervi EscolaTupySociesc Joinville SC
LuizHenriqueMinatti SagradaFamlia Blumenau SC
RadharaniGitaKanesiro Etapa SoBernardo SP
VictorPeresdeMeloGoulart Militar Manaus AM
PauloRobertoOliveiraMartins LatoSensu Manaus AM
NellyCarmenRamirezCanelo IF-AM Manaus AM
HannaTatsutaGalassi Universitario Londrina PR
Resultado - Fase III
| 87 86 |
Programa Nacional
Olimpadas de Qumica
Ano Internacional de Qumica
ThaynaraMachadodosSantos Agostiniano Goinia GO
RayanaLorenaSantosFlorncio Amadeus Aracaju SE
LetciadeArajoCarvalho Agostiniano Goinia GO
AnaFlviaGalvoLopes DomBarreto Teresina PI
VictorRipardoSiqueira LatoSensu Manaus AM
MathuesDantasdeLucena Motiva CampinaGrande PB
EduardoCintraSimes Damas Recife PE
BrunoSilvaMucciaccia LeonardodaVinci Vitria ES
KlynsmannD.C.Bagatini SoJos Apucarana PR
AugustoHenriquedosSantos FundaoBradesco Laguna SC
JulianaMoisesPoletti Motiva JooPessoa PB
IsabellaGomesSantos CharlesDarwin Serra ES
LucasBitoMaynart Salesiano Aracaju SE
AlissonRibeiroLucena Motiva CampinaGrande PB
PedroHenriqueFavaro SoJos Apucarana PR
AnaCarolinaT.R.deMedeiros Sag.CoraodeMaria Vitria ES
LeandroGomesSantanadeSouza Anchieta Salvador BA
MarianaRezendedeOliveira EducaoCriativa Ipatinga MG
LucasPradoChagas LatoSensu Manaus AM
CarolinaBordinho LiberatoSalzano NovoHamburgo RS
HlderLimadaRocha Aplicao Aracaju SE
TarsoSaidelesPizarro PolitcnicodaUFSM SantaMaria RS
LaersonSaraivaVerissimo Motiva CampinaGrande PB
FelipeSousadeAndrade GGE Recife PE
DanielleOliveiradeSouza Ideal Belm PA
CarlosHenriquedaSilva EREMProf.AdautoCarvalho Recife PE
GiovanniSimoesMedeiros Motiva JooPessoa PB
IsmaelRoggiaSilva PolitcnicodaUFSM SantaMaria RS
JooPedroLimadaCosta PolitcnicodaUFSM SantaMaria RS
Resultado - Fase III
| 87
Olimpada Brasileira
de Qumica 2011
Ano Internacional de Qumica
MaralButtenberg SagradaFamlia Blumenau SC
HenriqueArenaredeOliveira Militar BeloHorizonte MG
LucasSinicioMolina Militar Braslia DF
ThaniaGomesBraz ColgioArnaldo BeloHorizonte MG
LucasTadeuKrgerPoffo Posiville Joinville SC
KauUedaSilveira MilleniumClasse Goinia GO
EduardaKarynneSouza EremLuizAlvesSilva Recife PE
DanielSchulzNogueira IF-RJ RiodeJaneiro RJ
MatheusFernandoC.L.Nascimento DomBarreto Teresina PI
AmandaPierredeMoraesMoreira Crescimento SoLus MA
FranklinMendoaFloresta Alternativo Aracaju SE
RicardoStaryRodrigues ObjetivoIndaiatuba Indaiatuba SP
RennanMartinsViana MadreVillac Teresina PI
BernardoO.deCarvalho IF-RJ RiodeJaneiro RJ
AmandaGiubertiPoltronieri SoDomingos Vitria ES
ErikaRodriguesVieiradeMacdo IF-PE Recife PE
EduardoAriTurconi MaristaRosrio PortoAlegre RS
SarahSantanaDiniz Coesi Aracaju SE
AmandaVielmoSagrilo PolitcnicodaUFSM SantaMaria RS
EduardoBrandtNunes Bionatus CampoGrande MS
BrendaDiasMarques Cepron DomPedro MA
GabrielAlvesCarrio Prevest Goinia GO
GustavoCoqueirodeAraujo Anchieta Salvador BA
JosAlbertoSouzaAbdon LeonardodaVinci Braslia DF
PedroHenriqueSilvadosAnjos Militar RiodeJaneiro RJ
FernandoCarlosBrandoFilho Expresso Goinia GO
DaniloPequeno Motiva CampinaGrande PB
YanMouraQuintino IF-AL Macei AL
GiovanneAdaltonKellerStivari Ateneu Londrina PR
Resultado - Fase III
| 89 88 |
Programa Nacional
Olimpadas de Qumica
Ano Internacional de Qumica
GuilhermeLima BomJesusLourdes Curitiba PR
JosMatheusSantosPereira CEFET Salvador BA
AdoniasdeFreitasNunes MilleniumClasse Goinia GO
EduardoJoseRochadeMoraes Amadeus Aracaju SE
AlequineBatistadeLima Lourdinas CampinaGrande PB
JulieFujishima Moderno Macap AP
RafaelaNunesCouto LeonardodaVinci Braslia DF
JooPedrodeC.Magalhes Militar Recife PE
JooPedroSantosWanderley Motivo Recife PE
MarinaPerboneLen ColgioClasseA PortoVelho RO
AlexandreChavesFilho ConexoAquarela Macap AP
LuizFelipeManfodosSantos PolitcnicodaUFSM SantaMaria RS
Acinciaestdestinadaadesempenharumpapelcada
vezmaispreponderantenaproduoindustrial.Easnaes
quedeixaremdeentenderessaliohoinevitavelmentede
serrelegadasposiodenaesescravas:cortadorasdelen-
haecarregadoresdeguaparaospovosmaisesclarecidos.
LordRutherford
Resultado - Fase III
| 89
Olimpada Brasileira
de Qumica 2011
Ano Internacional de Qumica
Processoseletivo
paraasolimpadasinternacionais
Aps concludas as trs primeiras fases da Olimpada Brasileira de Qu-
micarealiza-seasolenidadenacionaldepremiaonaqualsoagraciados
commedalhasouro,prataebronzeecomcertifcadosdeMenoHonrosa
osestudantesquetiverammaiordestaque.Essapremiaoaconteceanual-
mentenaltimasemanadomsdenovembro.Aolimpadacontinuacom
o processo seletivo para escolher os estudantes que representam o Brasil
nasolimpadasinternacionais:InternationalChemistryOlympiadeOlimpa-
daIberoamericanadeQumica.Todososestudantesagraciadoscommeda-
lhassoconvocadosparaessaseletivaconstituda,tambm,detrsfases:a
primeira,FASEIV,tratadeavaliarconhecimentosdelaboratrio,aFASEV
constitudapelocursodeAprofundamentoeExcelnciaemQumicaoqual
rene 15 estudantes classifcados na fase anterior, o curso realizado em
umadasuniversidadesparticipantesquepossuicursodeps-graduaoem
qumica.Em2011,ocursofoirealizadonoInstitutodeQumicadaUNICAMP
no prximo ano ser na UFMG. A FASE VI fnaliza o processo seletivo com
a escolha dos quatro estudantes que compem a delegao brasileira nas
olimpadasinternacionaisdequmica.
AFASEIVdaOBQ-2010foirealizadaem1fevereirode2011,umexa-
me fundamentado na exibio de um flme com experimentos de qumica
acompanhadodeumquestionrio.Paraestafaseforamconvocadostodos
osestudantesmedalhadosnaFASEIIIdaOBQ-2010,ototaldetrintaecin-
coestudantes,listadosemwww.obq.ufc.br/resultadovideo2011.htm.Quinze
dessesestudantesseclassifcaramparaoCursodeAprofundamentoeExce-
lnciaemQumicaqueaconteceude21/03a01/04/2011nasdependndias
doInstitutodeQumicadaUnicamp(FaseVdaOBQ-2010).
Os professores que ministraram esse curso foram os responsveis pela
elaboraoecorreodoexameaplicadoem16/04/2011(FaseVIdaOBQ-
2010)edefniuosestudantesquerepresentaramoBrasilnasolimpadasin-
ternacionais.ParticiparamdoXCursodeAprofundamentoeExcelnciaem
Qumicaosseguintesestudantes:
Seletiva internacional
| 91 90 |
Programa Nacional
Olimpadas de Qumica
Ano Internacional de Qumica
RaulBrunoMachadodaSilva FariasBrito CE
DaviRodriguesChaves ArideS CE
TabataClaudiaAmaraldePontes Etapa SP
DavidsonAnthonyAragoFreire ArideS CE
DanielArjonadeAndradeHara ObjetivoGranjaViana SP
LaraMulatoLima ArideS CE
TaynaraCarvalhoSilva MASTER CE
BrunoLimaverdeVillarLbo FariasBrito CE
BrenoSaldanhaSousa FariasBrito CE
SergioPereiradeOliveiraJnior FariasBrito CE
BiancaRohsnerBezerra FariasBrito CE
EmersonHolandaMarinho FariasBrito CE
YuriJernimoMoreira FariasBrito CE
NatliaAragoDias MASTER CE
PedroVictorBarbosaNolto InstitutoDomBarreto PI
OsquatroprimeirosalunoslistadosacimarepresentaramoBrasilna43
rd
In-
ternationalChemistryOlympiadrealizadaemAnkara-Turquiaeoestudante
DanielArjonasubstituiuDavidsonAnthonynadelegaoqueparticipouna
XVIIOlimpadaIberoamericanadeQumicarealizadaemTeresina-Brasil.
As coisas que queremos e parecem impossveis s
podemserconseguidascomumateimosiapacfca.
MahatmaGandi
Seletiva internacional
| 91
Olimpada Brasileira
de Qumica 2011
Ano Internacional de Qumica
Estudantes convocados para participar do X Curso de Aprofundamento e
ExcelnciaemQumicaministradonoInstitutodeQumicadaUNICAMP.
Nodiaemqueohomemsedercontadeseusprofundos
equcocos,terterminadooprogessodacincia.
MarieCurie(1867-1934)
Seletiva internacional
| 93 92 |
Programa Nacional
Olimpadas de Qumica
Ano Internacional de Qumica
EstudantesparticipantesnoXCursodeAprofundamentoeExcelnciaemQumicaeprofessoresdo
InstitutodeQumicadaUNICAMPqueministraramocurso.
Seletiva internacional
| 93
Olimpada Brasileira
de Qumica 2011
Ano Internacional de Qumica
Processo seletivo para as Olimpadas Internacionais
Olimpada Brasileira de Qumica - 2010 Fase IV
Seletivaparaa43
rd
InternationalChemistryOlympiad,Turquia
Filmeexibidoem01.02.2011,14horas
Caroestudante,
Esteexamedecunhoexperimentaltemporfnalidadeselecionaros15(quin-
ze)estudantesqueparticiparodoXCursodeAprofundamentoeExcelncia
que ser ministrado na UNICAMP (Campinas) no perodo de 21 de maro
a1deabrilprximo.Umnovoexameseraplicado,apsarealizaodo
curso,paradefniraequipe(4estudantes)querepresentaroBrasilna43
rd

IChOqueaconteceremjulhoprximo,emAnkara-Turquia.
Vocdispede2he30minutosparaverovdeoeresponderasquestes
desteexame.
INSTRUES
1.Vejaatentamente,notelevisorounateladeprojeo,asimagensdoflme
quecontmosfundamentosdesteexame.
2.Seucoordenador,inicialmente,exibiragravaocompletadoexame(18
min)e,aseguir,apresentarcadaparteseparadamente.Senecessrio,re-
passarasimagensvriasvezesatesclarecersuasdvidas.
3. Leia as perguntas relativas a cada experimento e escreva a resposta nas
folhasofciais.
4.Aprovaestdivididaem7(sete)atividades.
5.Osresultadosseroencaminhadosat10/02paraseucoordenador(etam-
bmdiretamenteparavoc,casotenhae-mail).Vejaoresultadonainternet
emwww.obquimica.org.
6.Osestudantesconvocadosparaaetapaseguinte(CursonaUNICAMP)de-
vemfcaratentosnossapginanainternet.Provavelmente,naprxima
semana,serdivulgadaalistadeexercciospreparatriosparaaOlimp-
adaInternacionaldeQumica.importantssimoqueoestudanteresolva
todosessesexerccioscomantecednciaediscuta-osduranteocursoque
serministradonaUniversidadeEstadualdeCampinas.
Seletiva - Fase IV
| 95 94 |
Programa Nacional
Olimpadas de Qumica
Ano Internacional de Qumica
Questionrio
Atividade1 20pontos
A - D o nome e indicaes de uso para cada uma das vidrarias aresenta-
das..
Atividade2 10pontos
AIndiqueonomedesseequipamentoeasuafuno
Atividade3 20pontos
ADiferencieautilizaodecadaumdossistemasindicados.
BDonomedecadavidraria,materialeequipamentodossistemasAeB.
Atividade4 20pontos
AExpliqueadiferenadospontosdeebuliodosismeroseindiqueasestru-
turasdeAeB.
BQualafunodacolunadeVigreaux?
Atividade5 10pontos
AExpliqueadiferenaobservadanascoloraesdosdoisbqueres..
Atividade6 10pontos
AIndiqueogsqueformadoemcadatubodeensaio
Atividade7 10pontos
AIndiquearepresentaoestruturaldaamidaaromticaproduzida(slido
formado).
Seletiva - Fase IV
| 95
Olimpada Brasileira
de Qumica 2011
Ano Internacional de Qumica
OlimpadaBrasileiradeQumica
OBQ-2010FASEVI
Seletivaparaa43
rd
InternationalChemistryOlympiad
Ankara-Turquia16.04.2011
NOTA:Observeosalgarismossignifcativos.
CADERNODEQUESTES
QUESTO1 (10pontos)
Umaalquotade5,00mLdeumaamostradeconhaquefoidiludapara
1000,00 mL em balo volumtrico. Desta soluo diluda, uma alquota de
25,00 mL foi submetida a uma destilao. O etanol destilado foi recolhido
em50,00mLdeumasoluodedicromatodepotssio0,02000molL
-1
.A
oxidaodoetanollevaaformaodecidoactico.Apsresfriamento,fo-
ramadicionados20,00mLdeumasoluodeFe
2+
0,1253molL
-1
.Oexcesso
doFe
2+
foitituladocom7,46mLdeumasoluopadrodedicromatode
potssio0,02000molL
-1
.
a)Escrevatodasasreaesqumicasenvolvidasnaanlise;
b)Especifqueosagentesoxidanteseredutores;
c)Calculeapercentagem(m/v)deetanolnoconhaque.
QUESTO2 (15pontos)
Aguapresentenospoosdepetrleo,denominadaguadeformao
umasoluosalinamuitoconcentradaquecontmcloretodesdiocomo
salmajoritrioetambmoutrosons,como:Ca
2+
,Sr
2+
,Ba
2+
,carbonatoehi-
drogenocarbonato.OpHdaguadeformao7,2.
Umaamostrade1Ldeguadeformaofoirecolhidadeumpoode
petrleo e acidifcada. A acidifcao resultou na liberao de 1L de gs
medidoa27Ce1atm.Apsaacidifcaofoiadicionadosulfatodesdio
amostraresultandoem5gdeprecipitadobranco.Arazoentreasconcen-
traesdosonsCa
2+
:Sr
2+
:Ba
2+
,naguadeformaoiguala10.000:100:1.
Seletiva Fase VI
| 97 96 |
Programa Nacional
Olimpadas de Qumica
Ano Internacional de Qumica
a)Escrevaasreaesqueresultaramnaliberaodegspelaacidifcaoda
guadeformao.
b)Escrevaasreaesqueresultaramnaformaodoprecipitadobranco.
c)Calculeaconcentraodosonscarbonatoehidrogenocarbonatopresen-
tenaguadeformao.pKaHCO
3
-
=6,3.
d)CalculeaconcentraodeCa
2+
,Sr
2+
eBa
2+
naguadeformaoconside-
randoquehouveprecipitaocompletadosons.
e)Paraossaisinsolveisdosonsdosonsclcio,estrncioebrio,quefor-
mamoprecipitadobranco,asconstantesdesolubilidadesorespectiva-
mente:3.10
-5
;3.10
-7
e10
-10
.Calculearazoentreasconcentraesonsna
soluoapsprecipitao.
QUESTO3 (10pontos)
OmodeloatmicopropostoporBohrlimitadoasistemashidrogeni-
des,cujaenergia(emeV)doestadofundamentaldadapor:
En=-13,6Z
2
/n
2
(1).
a)Calculeas1e2energiasdeionizao(emkJ/mol)para1moldetomos
deHeHe,respectivamente.
b)UmtomodeHnoestadofundamentalabsorveumftondeluzdecompri-
mentodeondaiguala97,2nm.Eleentoemiteumftondecomprimento
deondaiguala486nm.Qualoestadofnaldotomoemquesto?
QUESTO4 (15pontos)
1) Muitos compostos binrios obtidos com halognios e oxignio so
conhecidos apesar de alguns deles apresentarem instabilidade elevada.
Dentreessescompostos,odifuoretodeoxignio(PF=-224
o
C,PE=-145
o
C)
eodifuoretodedioxignio(PF=-154
o
C,PE=-57
o
C)soosnicosexemplos
obtidoscomfuoretambmasnicasmolculasemqueoelementooxignio
apresentaumNoxpositivo.Oprimeirocompostopodeserpreparadoatravs
dareaoentregsfuoreumasoluoaquosaalcalina.Josegundo,apenas
pelafotlisedeumamisturagasosadosdoiselementos.
Questes
| 97
Olimpada Brasileira
de Qumica 2011
Ano Internacional de Qumica
a)Apresente as equaes qumicas globais que descrevem as snteses dos
doiscompostosbinrioscitadosnotexto.
b)ApresenteasestruturasdeLewis,osnmerosdeoxidaoeascargasformais
deambososcompostos.
c) Apresente a geometria dos compostos e a hibridizao em ambos os
casos.
2)Emoposiorestritaqumicadoscompostosbinriosdefuoreoxignio,
os anlogos de cloro apresentam-se em vrios estados de oxidao e uma
variedade de estruturas. O dixido de cloro o nico produzido em larga
escala e encontra aplicao como agente alvejante na indstria do papel-
celulose e tambm na desinfeco de guas para consumo. A obteno
industrial realizada a partir da reduo do on clorato (de fonte mineral)
em soluo aquosa usando gs clordrico ou dixido de enxofre em meio
fortementecido.
a)Apresente as equaes qumicas globais que descrevem as snteses do
dixidodecloroapartirdosdoisagentesredutorescitadosnotexto.
b)Apresente as estruturas de Lewis, os nmeros de oxidao e as cargas
formais de todos os compostos (reagentes e produtos) envolvidos nas
equaesdotemanterior.
c)Apresente a geometria dos compostos e a hibridizao em todos os
casos.
d)Apresenteumaexplicaoparaofatodocloroapresentarmaiscompostos
binrioscomooxigniodoqueofor.

Seletiva Fase VI
| 99 98 |
Programa Nacional
Olimpadas de Qumica
Ano Internacional de Qumica
QUESTO5 (10pontos)
Considereosseguintesdadostermodinmicos:

Pagina80-temdadosfaltandonoquadro

Considereosseguintesdadostermodinmicos:

substncia

f

H
0
298
/
(kJmol
-1
)

S
0
298/
(JK
-1
mol
-1
)

0
p
C 298
/
(JK
-1
mol
-1
)
Glicose(s) 1273,2 211,9 218,6
O
2
(g) 0 204,8 29,3
CO
2
(g) 393,1 213,4 37,1
H2O(l) 285,6 69,8 75,2
H2O(g) 241,6 188,5 33,5

Tambemnapagina80

O tratamento espectroscpico de ligaes pode ser feito de forma muito aproximada


considerando-se um modelo denominado de partcula na caixa. No modelo da partcula na caixa
unidimensional, a energia de um determinado nvel quntico para uma cadeia carbnica pode ser
definidapor:
2
2 2
8mL
h n
E
n
=
sendohaconstantedePlanck,mamassadapartcula,Lotamanhodacadeiamolecularen=1,
2,3,...
Estemodelopodeseraplicadoconsiderando-seasseguintescondies:1)Cadanveldeenergiada
partcula na caixa s pode acomodar no mximo dois eltrons; 2) A energia total da distribuio
eletrnicacorrespondesomadaenergiadecadaumdoseltrons,porexemplo,paracincoeltrons
ocupando os primeiros nveis de energia, a expresso ser:
] 3 2 2 1 2 [
8 8
3
8
2
2
8
2
2 2 2
2
2
2
2 2
2
2 2
2
2
+ + = + + =
mL
h
mL
h
mL
h
mL
h
E
total
;3)Atransioeletrnicadoltimo
nvelocupadoparaoprimeironvelvaziodefinidacomo:

= = + =
c
h h n
mL
h
E ) 1 2 (
8
max
2
2
,sendo
n
max
onmeroqunticodoltimonvelocupado,cavelocidadedaluz,ocomprimentodeondae
afreqnciadaradiao.
Considereostrscompostosorgnicosaseguirerespondaositensabaixo:

Respondaasseguintesquestes:
a)Calculeovalordavariaodeenergialivremolarnatemperaturade
298Ke1atm,paraareaodecombustocompletaparaglicose.
b)Calcule a variao de entropia molar para a reao na temperatura
de298Ke1atmejustifquecombasenareaoovalorpositivoda
mesma.
c)Calculeovalordavariaodeenergialivremolarnatemperaturade
400Ke1atm,paraareaodecombustocompletaparaglicose.
QUESTO6 (15pontos)
Otratamentoespectroscpicodeligaesppodeserfeitodeformamuito
aproximadaconsiderando-seummodelodenominadodepartculanacaixa.
Nomodelodapartculanacaixaunidimensional,aenergiadeumdetermina-
donvelqunticoparaumacadeiacarbnicapodeserdefnidapor:

sendohaconstantedePlanck,mamassadapartcula,Lotamanhodacadeia
molecularen=1,2,3,...
Estemodelopodeseraplicadoconsiderando-seasseguintescondies:
Questes
| 99
Olimpada Brasileira
de Qumica 2011
Ano Internacional de Qumica
1)Cadanveldeenergiadapartculanacaixaspodeacomodarnomximo
doiseltrons;
2)Aenergiatotaldadistribuioeletrnicacorrespondesomadaenergia
decadaumdoseltrons,porexemplo,paracincoeltronsocupandoos
primeirosnveisdeenergia,aexpressoser:

3)Atransioeletrnicadoltimonvelocupadoparaoprimeironvelvazio
defnidacomo:
sendon
max
onmeroqunticodoltimonvelocupado,cavelocidadeda
luz,locomprimentodeondaenafreqnciadaradiao.
Considereostrscompostosorgnicosaseguirerespondaositensabaixo:
CH
3
-(CH=CH)
3
-CH
3

CH
3
-(CH
3
=CH)
4
-CH
3

CH
3
-(CH=CH)
5
-CH
3
a)Faaumdesenhodecadaumadasmolculasacimaexplicitandotodasas
ligaesqumicaseindiquequaissoostomosdecarbonoquepossuem
ligaesp.
b)Determineonmerodeeltronspemcadamolcula,faaumdiagrama
deenergiaqualitativodistribuindoesseseltronsparacadaumadastrs
molculasedetermineonmeron
max
doltimonveldeenergiaocupado.
c) CalculeaenergiatotalpparaocompostocomfrmulaCH
3
(CH=CH)
4
CH
3
.
Seletiva Fase VI
| 101 100 |
Programa Nacional
Olimpadas de Qumica
Ano Internacional de Qumica
d)Identifquequaldastrsmolculasdeveapresentaratransioeletrnica
com maior comprimento de onda e calcule o valor desse comprimento
deondaemnm(10
-9
m)assumindoqueocomprimentodecadaumadas
ligaesC-CeC=Csoiguaisa1,40x10
-10
m.
e)Ovalorexperimentaldocomprimentodeondaparaatransioeletrnica
pentreoltimoorbitalocupadoeoprimeiroorbitalvazioparaamol-
culaCH
3
(CH=CH)
3
CH
3
iguala274nm.Calculeocomprimentodacadeia
carbnicapemnmutilizandoomodelodapartculanacaixaapartirdo
comprimentodeondaexperimental.Qualdeveserarazoentreocom-
primentodacadeiapestimadaapartirdodadoexperimentalemrelao
aovalorobtidocomoasomadoscomprimentosdeligaodoitem(d)?
QUESTO7 (15pontos)
AsubstnciadenominadaAI-77-BfoiisoladadoBacilluspumiluseapre-
sentaatividadeantiinfamatriaegastroprotetora,oqueacolocaemgrande
vantagememrelaoaosantiinfamatriostradicionais.

a) Faaumdesenhodecadaumadasmolculasacimaexplicitandotodasasligaesqumicase
indiquequaissoostomosdecarbonoquepossuemligaes.
b) Determine o nmero de eltrons em cada molcula, faa um diagrama de energia
qualitativo distribuindo esses eltrons para cada uma das trs molculas e determine o
nmeron
max
doltimonveldeenergiaocupado.
c) CalculeaenergiatotalparaocompostocomfrmulaCH
3
(CH=CH)
4
CH
3.

d) Identifique qual das trs molculas deve apresentar a transio eletrnica com maior
comprimento de onda e calcule o valor desse comprimento de onda em nm (10
-9
m)
assumindoqueocomprimentodecadaumadasligaesC-CeC=Csoiguaisa1,40x10
-10
m.
e) O valor experimental do comprimento de onda para a transio eletrnica entre o ltimo
orbital ocupado e o primeiro orbital vazio para a molcula CH
3
(CH=CH)
3
CH
3
igual a 274
nm.Calculeocomprimentodacadeiacarbnicaemnmutilizandoomodelodapartculana
caixa a partir do comprimento de onda experimental. Qual deve ser a razo entre o
comprimentodacadeiaestimadaapartirdodadoexperimentalem

Pagina82a85

A substncia denominada AI-77-B foi isolada do Bacillus pumilus e apresenta atividade


antiinflamatria e gastroprotetora, o que a coloca em grande vantagem em relao aos
antiinflamatriostradicionais.

(CH CH)
3
CH
3
CH
3
(CH CH)
5
CH
3
CH
3
(CH CH)
4
CH
3
CH
3
OH
O
O
H
H
N
H
O
HO H
H
HO
NH
2
H
CO
2
H
AI-77-B
UmadesuassntesesenvolveoacoplamentodosfragmentosI,quecontm
oanelaromtico,eofragmentoII.
Questes
| 101
Olimpada Brasileira
de Qumica 2011
Ano Internacional de Qumica
UmadesuassntesesenvolveoacoplamentodosfragmentosI,quecontmoanelaromtico,
eofragmentoII.

ParteA:sntesedointermedirioI
NaprimeirasnteseracmicadointermedirioIfoipossvelcontrolaraconfiguraorelativadosdois
centros estereognicos presentes. Esta sntese comea com o lcool A, que transformado no
intermedirio B, aps reao com PCC (clorocromato de piridnio). Este intermedirio B ser
submetidoaumareaodeWittigcomoreagenteCparaformaraolefinaD,cujaconfiguraono
estrepresentadanoesquema.ApsaobtenodeD,ahidrlisedafunoamidaemmeiobsico
forma o cido E, que em seguida sofre uma reao que forma predominantemente a lactona F,
embora a G, tambm seja formada. A lactona F ser convertida no intermedirio H, que, aps uma
reao de hidrogenao e remoo do grupo de proteo da hidroxila fenlica produz o
intermedirioI.

Pergunta-se:

a)QualaestruturadointermedirioB?
b)QualaestruturadocompostoC,queirreagircomB?
c)QualseraconfiguraodaligaoduplapresentenaolefinaD?
d)QuereagentessoutilizadosparaconverterocidoEnaslactonasFeG?
e)RepresenteaestruturadointermediriodareaoquetransformaocidoEnalactonaF?.
f)RepresentecorretamenteaconfiguraorelativadoscentrosestereognicospresentesnalactonaF.
g) Qual a estrutura do produto H? Represente corretamente a configurao relativa dos seus
centrosestereognicos.

OH
O
O
H
NH
2
H
I
O
O
NH
2
OH
O
HO
II
ParteA:sntesedointermedirioI
NaprimeirasnteseracmicadointermedirioIfoipossvelcontrolara
confguraorelativadosdoiscentrosestereognicospresentes.Estasntese
comeacomolcoolA,quetransformadonointermedirioB,apsreao
com PCC (clorocromato de piridnio). Este intermedirio B ser submetido
a uma reao de Wittig com o reagente C para formar a olefna D, cuja
confguraonoestrepresentadanoesquema.ApsaobtenodeD,a
hidrlisedafunoamidaemmeiobsicoformaocidoE,queemseguida
sofreumareaoqueformapredominantementealactonaF,emboraaG,
tambmsejaformada.AlactonaFserconvertidanointermedirioH,que,
apsumareaodehidrogenaoeremoodogrupodeproteodahi-
droxilafenlicaproduzointermedirioI.
Seletiva Fase VI
| 103 102 |
Programa Nacional
Olimpadas de Qumica
Ano Internacional de Qumica

h) O composto H reage em meio cido com a cetona J, para formar os produtos K e L. D as


estruturasdeKeL.Qualdelesseropredominante?

ParteB:sntesedointermedirioII
AsduasprimeirasetapasdasntesedointermedirioIIsotransformaessucessivasdodoterde
enolA.Naprimeiraetapa(etapaA)temosumareaodecicloadioparaformarointermedirioB,
que depois convertido na lactona C. Aps sucessivas interconverses de grupos funcionais,
chegamos lactona D, cuja hidrlise levar ao precursor imediato do intermedirio II, aps a
remoodogrupoprotetordahidroxilasecundria.

OG
N(CH
3
)
2
O
A
G = grupo de proteo
OH
PCC
CH
2
Cl
2
B
C
OG
N(CH
3
)
2
O
D
OG
O
O
F
OG
OH
O
E
hidrlise
bsica
?
I
H
NaN
3
DMF
intermedirio I
+
G
O
J
+ H
HBF
4
K + L
Pergunta-se:
a)QualaestruturadointermedirioB?
b)QualaestruturadocompostoC,queirreagircomB?
c)QualseraconfguraodaligaoduplapresentenaolefnaD?
d)QuereagentessoutilizadosparaconverterocidoEnaslactonasFeG?
e)Representeaestruturadointermediriodareaoquetransformaocido
EnalactonaF?
f) Represente corretamente a confgurao relativa dos centros estereognicos
presentesnalactonaF.
g)QualaestruturadoprodutoH?Representecorretamenteaconfgurao
relativadosseuscentrosestereognicos.
OcompostoHreageemmeiocidocomacetonaJ,paraformarosprodutos
KeL.

h) O composto H reage em meio cido com a cetona J, para formar os produtos K e L. D as


estruturasdeKeL.Qualdelesseropredominante?

ParteB:sntesedointermedirioII
AsduasprimeirasetapasdasntesedointermedirioIIsotransformaessucessivasdodoterde
enolA.Naprimeiraetapa(etapaA)temosumareaodecicloadioparaformarointermedirioB,
que depois convertido na lactona C. Aps sucessivas interconverses de grupos funcionais,
chegamos lactona D, cuja hidrlise levar ao precursor imediato do intermedirio II, aps a
remoodogrupoprotetordahidroxilasecundria.

OG
N(CH
3
)
2
O
A
G = grupo de proteo
OH
PCC
CH
2
Cl
2
B
C
OG
N(CH
3
)
2
O
D
OG
O
O
F
OG
OH
O
E
hidrlise
bsica
?
I
H
NaN
3
DMF
intermedirio I
+
G
O
J
+ H
HBF
4
K + L
h)DasestruturasdeKeL.Qualdelesseropredominante?
Questes
| 103
Olimpada Brasileira
de Qumica 2011
Ano Internacional de Qumica
ParteB:sntesedointermedirioII
AsduasprimeirasetapasdasntesedointermedirioIIsotransforma-
essucessivasdoterdeenolA.Naprimeiraetapa(etapaA)temosuma
reaodecicloadioparaformarointermedirioB,quedepoisconvertido
nalactonaC.Apssucessivasinterconversesdegruposfuncionais,chega-
moslactonaD,cujahidrliselevaraoprecursorimediatodointermedirio
II,apsaremoodogrupoprotetordahidroxilasecundria.

Pergunta-se:

a)QuaissoosreagentesdaetapaAequalaestruturadaespciequereagircomoterdeenol
A?
b)DreagentesparaaetapaB.
c)Qualaconfiguraorelativadoscentrosestereognicosassinalados(*)naestruturadocomposto
C?
d)QualoprodutoformadonareaodoterdeenolAcomofurano?

BnO
GO
OMe O
A
O
O
Cl
Cl
BnO
Bn = CH
2
Ph
G = grupo de proteo
O
O
OG
D
EtO
O
H
2
N
etapa A
B
etapa B
CO
2
Me
GO
C
O
furano
E
*
*
Pergunta-se:
a)QuaissoosreagentesdaetapaAequalaestruturadaespciequerea-
gircomoterdeenolA?
b)DreagentesparaaetapaB.
c)Qualaconfguraorelativadoscentrosestereognicosassinalados(*)na
estruturadocompostoC?
d)QualoprodutoformadonareaodoterdeenolAcomofurano?
Seletiva Fase VI
| 105 104 |
Programa Nacional
Olimpadas de Qumica
Ano Internacional de Qumica
QUESTO8 (10pontos)
1)Escrevaasestruturaseosnomesdopardeismeros(AeB),defrmula
C
5
H
9
BrO
2
, cujos sinais dos espectros de RMN
1
H esto listados abaixo. Os
espectrosdeinfravermelhodestesismerosnoapresentamsinaisdeabsor-
bnciadeOH.
IsmeroA:
Sinaisemd(ppm)=4,4(q,1H);4,2(q,2H);1,8(d,3H)e1,3(t,3H)
IsmeroB:
Sinaisemd(ppm)=4,2(q,2H);3,6(t,2H);2,9(t,2H)e1,3(t,3H)
2)NoespectrodeRMN
1
HdocompostoC,defrmulaC
10
H
10
O
2
,observam-
seosseguintessinais:
d ppm)=1,4(t,3H);4,3(q,2H);7,4(t,2H);7,5(t,1H);8,0(d,2H).
EnoespectrodeRMN
13
C,domesmocomposto,observam-se7(sete)sinaisem:
d ppm)=14,4(CH
3
);60,8(CH
2
);128,4(CH);129,7(CH);130,9(C);132,8
(CH)e166,3(C).
EscrevaaestruturaeonomedocompostoC
Obs:
1. Ao lado de cada sinal de RMN
1
H, esto apresentadas, entre parntesis,
as letras referentes s multiplicidades desses sinais (d=dupleto, t=tripleto,
q=quarteto)eonmerodehidrogniosrelacionadosaosmesmos(1H,2H
ou3H).
2. As multiplicidades apresentadas para os sinais na regio entre 7,0 e 8,0
ppmestolevandoemcontaapenasosacoplamentosorto.
ValoresaproximadosdefaixasdedeslocamentosqumicosdeRMN
1
He
13
C

Questes
| 105
Olimpada Brasileira
de Qumica 2011
Ano Internacional de Qumica
Seletiva Fase VI
Hidrognio/Carbono d
H
(ppm) d
C
(ppm)
CH
x
-C 0,8-1,6 5-40
CH
x
-C=C 1,6-2,3 10-30
CH
x
C=O 1,9-3,0 23-48
CH
x
-O 1,9-3,0 50-85
Alcenos 4,5-8,0 95-155
Aromticos 4,5-8,0 95-155
H-C=O 9,0-10,0 190-220
COOH 10,0-12,0 160-185
C=O(cetonas) - 190-220
C=O(steres) - 160-185
CHx-Br 3,0-4,5 20-40
CH
x
x=1,2ou3
-----------------------------------------------------------------------------------
ConstantesFundamentaiseFatoresdeConverso:
h=6,62x10
-34
J.s;NA=6,02x10
23
mol
-1
;c=3,00x10
8
m.s
-1
;1eV
=1,602x10
-19
J
| 107 106 |
Programa Nacional
Olimpadas de Qumica
Ano Internacional de Qumica

43
rd
International
ChemistryOlympiad
ProblemasExperimentais
12deJulhode2011
Ankara,Turkey
PROBLEMA1
Anlisedemisturasdecloretos
AcomposiodeumasoluoquecontmapenasMgCl
2
eNaClpodeser
determinada por um mtodo de titulao indireta onde se faz uma titula-
ocomprecipitaoquedeterminaaquantidadetotaldecloretopresente,
acompanhadaporumatitulaocomformaodecomplexoquedetermina
aquantidadedeonsmagnsio.Umatcnicacomumdetitulaocompre-
cipitaousadaparadeterminaraquantidadedeonscloretopresenteso
mtododeFajans.Nesteprocedimentoargentimtrico,onitratodeprata
usadocomotitulantenaprecipitaodeonscloretopresentesnasoluo.
Opontofnaldetectadocomousodeumindicadordeadsoro,tipica-
mente diclorofuorescena, um cido orgnico fraco. Antes do ponto fnal,
as partculas de cloreto de prata esto carregadas negativamente por cau-
sadaadsorodeonscloretoemexcessopresentesnasoluo.Osnions
doindicadorsorepelidospelasuperfciedaspartculasdecloretodeprata
carregadas negativamente dando soluo uma cor amarelo-esverdeada.
Entretanto,apsopontodeequivalncia,aspartculasdecloretodeprata
adsorvemonsdeprata.Destamaneiraumacamadacarregadapositivamen-
teformadaatraindoosonsdiclorofuoresceinatoeexibindoumacorrosa-
avermelhada.Adextrinausadaparaestabilizaraspartculasdecloretode
prataevitandoacoagulao.
Poroutrolado,aquantidadedeonsmagnsiopresentesnasoluopode
serdeterminadaporumatitulaocomplexomtricacomcidoetilenodia-
minotetraactico, EDTA. Como um ligante hexadentado, EDTA forma que-
latoscomtodososonsmetlicos,excetocomosalcalinos,numarazode
43
rd
IChO
| 107
Olimpada Brasileira
de Qumica 2011
Ano Internacional de Qumica
1:1independentementedacargadoction.EriochromeBlackT(EBT)um
indicadorcomumusadonastitulaescomEDTA.QuandoopH>7,00o
EBTimpeumacorazulsoluonaausnciadeonsmetlicoseformauma
corvermelhaquandocoordenadocomosonsmetlicos.
Nesteexperimento,serdeterminadooteordecloretodeumasoluoque
contmMgCl
2
eNaClusandoomtododeFajans.Aconcentraodeons
magnsioserdeterminadaporumatitulaocomEDTA.
Umasoluode100mLpreparadapeladissoluodeMgCl
2
eNaClemgua
fornecida como amostra desconhecida (unknown sample). O objetivo
determinaraconcentraodeMgCl
2
eNaClemg/100mL.
A.DeterminaodecloretototalpeloMtododeFajans
1.Usandoumapipetade10mL,transfraumaalquotade10,0mLdofrasco
rotuladocomounknownsolutionparaoErlenmeyerde250mL.Comple-
teovolumeparaaproximadamente100mL,adicionandoguadestilada.
2.Pegue um dos tubos de Eppendorf fornecido em saco plstico com zi-
pperrotuladocomodextrinetransfratodooseucontedoparaoErlen-
meyer.
3.Adicione5gotasdesoluodeindicadordiclorofuorescena.
4.AnoteaexataconcentraodasoluopadrodeAgNO
3
.
5.EnchaumadasburetascomasoluopadrodeAgNO3.
6.Tituleasoluodesconhecidaattodaasoluofcarrosa-avermelhada.
7.AnoteovolumedeAgNO
3
usado,emmL.
8.UseomesmoErlenmeyer,quandorepetiratitulao.Antesdisso,despeje
ocontedodoErlenmeyernofrascorotuladocomAqueousWastecon-
tainerelave-oduasvezescomguadestilada.
B.DeterminaodeMg
2+
portitulaodiretacomEDTA
1.EnchaasegundaburetacomasoluopadrodeEDTA.
2.AnoteaexataconcentraodeEDTAnasoluopadro.
3.Usandoumapipetade25mL,transfraumaalquota25,0mLdasoluo
desconhecida(unknownsolution)paraumErlenmeyer250mL.Comple-
teovolumeparaaproximadamente100mLadicionandoguadestilada.
4.Usandoumapipetade1mL,adicione1,0mLdotampo(buffer)depH10.
5.Adicione3-4gotasdasoluodeindicadorEBT.
Exame Prtico
| 109 108 |
Programa Nacional
Olimpadas de Qumica
Ano Internacional de Qumica
6.Titule a soluo desconhecida com a soluo padro de EDTA at a cor
mudardevermelhoparaazul.
7.RegistreovolumedesoluodeEDTAusada,emmL.
8.UseomesmoErlenmeyerquandorepetiratitulao.Antesdisso,despeje
ocontedodoErlenmeyernofrascorotuladocomAqueousWastecon-
tainerelave-oduasvezescomguadestilada.
Tratamentodedados
1.DetermineaquantidadeonsCl
-
emmilimols,em100mLdesoluodes-
conhecida(unknownsolution).
2.DetermineaquantidadeonsMg
2+
emmilimols,em100mLdesoluo
desconhecida(unknownsolution).
3.CalculeaconcentraodeMgCl
2
eNaClnasoluodesconhecida(unk-
nownsolution),emg/100mL.
43
rd
IChO
| 109
Olimpada Brasileira
de Qumica 2011
Ano Internacional de Qumica
PROBLEMA2
Geraodehidrognioapartirdeamnia-borano
Hidrognioconsideradocomoumafontedeenergialimpaepoucoagres-
siva ao meio ambiente e caminha para se tornar a energia sustentvel do
futuro. Uma estocagem segura e efetiva de hidrognio a chave para a
economia de hidrognio. Dentre os hidretos qumicos, considerados como
umpotentematerialparaestocagemdehidrognioslido,aamnia-borano
(H
3
NBH3)temdespertadoumagrandeateno,devidoaoseualtoconte-
dodehidrognioeestabilidadenascondiesdeoperaodeclulascom-
bustveis.Sobhidrlise,amnia-boranopodeliberarhidrognio,Equao1:
H
3
NBH
3
(aq)+2H
2
O(l)

NH
4
BO
2
(aq)+3H
2
(g) (1)
Soluesaquosasdeamniaboranosoestveisesuahidrliseocorreso-
mentenapresenadeumcatalisadoradequado.Recentesestudosmostraram
quenanoclustersdepaldio(0)estabilizadosporpolmerossolveisemgua
socatalisadoresaltamenteativosnahidrlisedeamnia-borano.Nanoclus-
tersdepaldio(0)sogeradosinsitupelareduodetetracloropaladato(II)
depotssiocomamnia-boranonapresenadecopolmerodecido4-esti-
renossulfnicoecidomaleico.
Neste experimento, a hidrlise cataltica de amnia-borano ser realizada
comtetracloropaladato(II)depotssioemumasoluocontendocopolme-
rodecido4-estirenossulfnicoecidomaleico.Otetracloropaladato(II)de
potssioserusadocomopr-catalizador,oqualserreduzidoporamnia-
boranoformandonanoclustersdepaldio(0),estabilizadospelocopolmero
decido4-estirenossulfnicoecidomaleico.Estesnanoclusterscatalisaro
ahidrlisedeamnia-borano.
I.VerifcaodoSistemaExperimental
1.Verifqueseosistemaexperimental,mostradoabaixo,estfxonosuporte,
seotubograduadoestconectadoaotubodeSchlenkpelamangueirade
plsticoeseabarramagnticaestdentrodotudodeSchlenk.
Exame Prtico
| 111 110 |
Programa Nacional
Olimpadas de Qumica
Ano Internacional de Qumica
2.Oseptodeveestarforaeavlvuladeveestaraberta.
3.Acerte a altura do bulbo para ajustar em zero o nvel da gua no tubo
graduado.
4.FecheavlvulanotubodeSchlenk.
II.Hidrlisedeamnia-borano
A.Naausnciadecatalisador
1.Usandoofunil,transfratodoocontedodasoluodeamnia-borano
contidanofrasco(Solution-A)paraotubodeSchlenk.
2.Adicione a soluo de polmero contida no frasco (Solution-B) para o
tubodeSchlenkatravsdofunil.
3.FecheotubodeSchlenkcomosepto,ajusteaagitaoem600rpm(como
marcadonoagitador),eabraavlvulaconectadaaotubograduado.Re-
gistreonveldaguanotempozerocomoV
o
.Acioneocronmetro.
4.Acadaminuto,leiaovolumetotaldegsproduzidoeescrevanatabela
dadanafolhaderespostas.Faaistopor10minutos.Pareocronmetro.
43
rd
IChO
| 111
Olimpada Brasileira
de Qumica 2011
Ano Internacional de Qumica
B.Napresenadecatalisador
1.Enquantoagita,transfratodoocontedodasoluodetetracloropaladato(II)
depotssiocontidanofrasco(Solution-C)paraotubodeSchlenk,inje-
tandoatravsdoseptocomaseringade2.0mL.Mantenhaaseringainse-
ridanoseptodurantetodooexperimento.Acioneocronmetro.
2.Acadaminuto,leiaovolumetotaldegsproduzidoeescrevaovalorna
tabeladadanafolhaderespostas.Faaistopor10minutos.Pareocron-
metro.
Tratamentodedados
A.Reaodeamnia-boranosemcatalisador
1.PloteovolumedegsregistradoversustemponoGrfco1.
2.AnoteovolumedegasproduzidocomoV
sem-catalisador
.
B.Reaodeamnia-boranocomcatalisador
1.PloteovolumedegsregistradoversustemponoGrfco2.
2.Calculeonmeromximodemolseovolumemximo(mL)degshidro-
gnioqueseriaproduzidoteoricamenteapartirdahidrlisede29,5mg
amnia-boranecompurezade97%m/ma25C.Apressoatmosfrica
de690torr.
3.Calculeavelocidadedegeraodehidrognioemseuexperimento
i)emmLH
2
/min.
ii)emmmolH
2
/min,assumindoqueatemperaturade25C.Apresso
atmosfricade690torr.
4.Calcule a velocidade de produo de hidrognio por mol de pal-
dio em (mol H
2
)(mol Pd)
-1
(min)-
1
em seu experimento. A pureza do
tetracloropaladato(II)depotssio98%m/m.
Exame Prtico
| 113 112 |
Programa Nacional
Olimpadas de Qumica
Ano Internacional de Qumica
PROBLEMA3
Sntese,purifcaoeseparaodeumamisturadiastereomrica
A natureza tem muitos compostos enantiomrica e diastereomericamente
puros,comoporexemploacares,aminocidos,esterides,etc.Algunsdes-
tescompostossobiologicamenteativoseusadosemmedicamentos.Con-
sequentementeasnteseorgnicaassimtricamuitoimportante.Umdos
mtodosusadosnasnteseassimtricadecompostosorgnicosutilizacata-
lisadoresquecontmmetais,nosquais,ometalestcoordenadocomuma
molcula orgnica quiral, designada por ligante quiral. Nesta experincia
doisligantesquiraisserosintetizados.

A.Sntese
1.Comoauxliodaseringaeatravsdoseptotransfraasoluodetrietila-
minacontidaemV1,paraobalodefundoredondode10mL(RxnRB)
quecontm0,50mmol2,3-dibromo-1-ferrocenilpropan-1-ona.
2.Agiteamistura,colocandoabarramagnticaa600rpm(marcadanapla-
cadeagitao),temperaturaambientedurante30minutos.
3.No fnal dos 30 minutos, transfra atravs do septo, a soluo de (R)-1-
feniletanaminacontidaemV2paraobalodareao,usandoamesma
seringa.
4.Agiteamistura,temperaturaambiente,durantemais60minutos.
5.Nofnaldos60minutosdesligueaagitaoeefetueaanliseporcroma-
tografaemcamadadelgada,TLC.Efetueaanlisedaseguinteforma:
i) VerifqueoestadodasplacasdeTLCantesdeus-las.Asplacasestraga-
dasserosubstitudassempunio.
ii)Desenhe,comolpis,alinhadeaplicaonaparteinferiordaplacade
TLC(Fig.2.1).
43
rd
IChO
| 113
Olimpada Brasileira
de Qumica 2011
Ano Internacional de Qumica
iii)ApliqueomaterialdepartidacontidonofrascoetiquetadocomoSM,
duasvezesnoladoesquerdodalinhadeaplicaoeduasvezesnomeio
dalinha.Namesmaplacaaplique,umaveznapartedireitadalinhade
aplicaoeoutranomeiodalinha,amisturareacional(RM)retiradado
balodareao.FaacomomostradonaFigura2.1eusandodiferentes
capilaresparacadaamostra.
iv)EluaaplacadeTLCnacmaradeeluio,usandooeluentefornecido.
Marquecomolpisalinhadofrontdosolvente.
v)QuandoaplacadeTLCestiverseca,coloque-anosacodeplsticocom
zipper,etiquetadoTLC1..

Figure2.1.PlacadeTLCFig.2.2PlacadeTLCcolocadanacubadeeluio.

Exame Prtico
| 115 114 |
Programa Nacional
Olimpadas de Qumica
Ano Internacional de Qumica
B.ColunadeCromatografa
1.Retirearolha,abraatorneiraedeixeoeluentecorreratatingirotopoda
slicagel.
2.FecheatorneiraeusandoumapipetadePasteurapliqueamisturareacio-
nal(Fig.2.3).

Figura2.3.ColunaFlashdecromatografa
3.Laveobalodareaocom0,5mLdeeluenteretiradosdofrascorotulado
ELUENT.Utilizeumaseringalimpaparaisso.Utilizeamesmapipetade
Pasteurparaaplicarestasoluodelavagemnotopodacoluna.
4.Abraatorneiradacolunaedeixeosolventecorreratatingirotopoda
slicagel.
5.FecheatorneiraeadicionecomumapipetadePasteur1,0mLdeeluente.
Abraatorneira.Quandooeluenteestivernotopodaslica,adicionelen-
tamente2a3mLdeeluentesemfecharatorneira.
43
rd
IChO
| 115
Olimpada Brasileira
de Qumica 2011
Ano Internacional de Qumica
6.Encha a coluna com mais eluente. ATENO: Seja cuidadoso ao adi-
cionaroeluentedeformaanoperturbarotopodacolunadeslica
gel.
7.Deformaaacelerarapurifcao,apliqueumpoucodepressonotopo
dacoluna,usandoparaoefeitooadaptadorcomapera.ATENO:Seja
cuidadosoaoaplicarpresso.Adicioneeluentedetemposemtem-
posparaevitarqueaslicaseque.
8.Espera-sequeduasfraesmajoritriassejamrecolhidas:asfraesAeB.
ColoquenorecipientedosresduosetiquetadosOrganicWaste,oque
foreluidoantesdafraoAeentreasfraesAeB.
9.RecolhaaprimeirafraomajoritrianumErlenmeyerde100mLeidenti-
fque-acomofraoA.
10.RecolhaasegundafraomajoritrianumErlenmeyerde250mLeiden-
tifque-acomofraoB.
11.DepoisderecolherafraoB,termineaeluiofechandoatorneira.
C.Anlise
1.UseasegundaplacadeTLCaplicandoomaterialdepartida(SM)duasve-
zesnoladoesquerdodalinhadeaplicao,afraoAduasvezesnomeio
dalinhaeafraoB,cincovezesnoladodireitodalinha.Apsaeluio
easecagemdaplacadeTLCcoloque-anosacodeplsticocomzipper,
etiquetadocomTLC2.
2.MeaovolumedafraoArecolhidousandoaprovetade50mLeescreva
essevolumenafolhaderespostas.
3.MeaovolumedafraoBrecolhidousandoaprovetade250mLeescre-
vaessevolumenafolhaderespostas.
4.Usandoapipetade2mLtransfra2,0mLdafraoAparaobalovolu-
mtricode10mLecompleteovolumeat10mLporadiodeeluente.
AgiteasoluopreparadaeenchausandoumapipetadePasteurencha
uma cubeta de UV-visvel (pelo menos do seu volume total). Pea ao
supervisorquedetermineaabsorbncia,a450nm,usandoumespectrofo-
tmetroeescrevaovalorobtidonafolhaderespostas.
5.ComafraoB(nonecessriadiluio),encha,usandoumapipetade
Pasteur,aoutracubetadeUV-visvel(pelomenosdoseuvolumetotal).
Pea ao supervisor que determine a absorbncia, a 450, nm usando um
espectrofotmetroeescrevaovalorobtidonafolhaderespostas.
Exame Prtico
| 117 116 |
Programa Nacional
Olimpadas de Qumica
Ano Internacional de Qumica
Tratamentodosdados
1.DesenheoresultadodaplacaTLC1nafolhaderespostas.
2.DesenheoresultadodaplacaTLC2nafolhaderespostas.
3.CalculeeescrevaosvaloresdeRfdasmanchas(fraoA,fraoBemate-
rialdepartidaSM)usandoaplacadeTLC2.
4.Ocoefcientedeextinomolar,e,a450nm:404L.mol
-1
cm
-1
paraAe
400L.mol
-1
cm
-1
paraB.Calcule:
i) Tendo em conta a quantidade de material de partida calcule o rendi-
mentodafraoA.
ii)Tendo em conta a quantidade de material de partida calcule o rendi-
mentodafraoB.
Ascoisasquerealizamos,nuncasotobelas
quantosquesonhamos.
Massvezes,nosacontecemcoisastobelas,
quenuncapensamosemsonh-las.

(desconhecido)
43
rd
IChO
| 117
Olimpada Brasileira
de Qumica 2011
Ano Internacional de Qumica
Exame Terico

43
rd
International
ChemistryOlympiad
ProblemasTericos
14deJulhode2011
Ankara,Turquia
PROBLEMA1
Problema 1 7,0 % do total
Os xidos de nitrognio, poluentes comuns do ar atmosfrico so formados principalmente por
xido ntrico, NO, e dixido de nitrognio, NO
2
. O xido ntrico atmosfrico essencialmente
formado durante as tempestades e nos motores de combusto interna. Em altas temperaturas o
NO reage com H
2
e produz xido nitroso, N
2
O, um gs de efeito estufa.
2 NO(g) + H
2
(g) N
2
O(g) + H
2
O(g)
No estudo da cintica desta reao a 820 C, as velocidades iniciais para a formao de N
2
O
foram medidas usando vrias presses parciais iniciais de NO e H
2
.
Exp.
Presso inicial, torr
Velocidade inicial de produo
de N
2
O, torrs
-1
PNO

H
2
1 120,0 60,0 8,6610
-2
2 60,0 60,0 2,1710
-2
3 60,0 180,0 6,6210
-2
Neste problema no use concentraes. Use unidades de presso em torr e tempo em
segundos.
a. Determine a lei de velocidade experimental e calcule a constante de velocidade.
b. Calcule a velocidade inicial de consumo de NO, se 2,0010
2
torr de NO e 1,0010
2
torr H
2
forem misturados a 820 C. (Se voc no tiver o valor da constante de velocidade, use 210
7
com a unidade apropriada.)
c. Calcule o tempo gasto para reduzir a presso parcial de H
2
a metade do seu valor, se
8,0010
2
torr de NO e 1,0 torr de H
2
forem misturados a 820 C. (Se voc no tiver o valor da
constante de velocidade, use 210
7
com a unidade apropriada.)
a b c
d
e
Problema 1 x%
i ii iii
3 2 6 6 1,5 1 2,5 22 7,0
Osxidosdenitrognio,poluentescomunsdoaratmosfricosoformados
principalmenteporxidontrico,NO,edixidodenitrognio,NO
2
.Oxido
ntricoatmosfricoessencialmenteformadoduranteastempestadesenos
motoresdecombustointerna.EmaltastemperaturasoNOreagecomH
2
e
produzxidonitroso,N
2
O,umgsdeefeitoestufa.
2NO(g)+H
2
(g) N
2
O(g)+H
2
O(g)
Noestudodacinticadestareaoa820C,asvelocidadesiniciaisparaaforma-
odeN
2
OforammedidasusandovriaspressesparciaisiniciaisdeNOeH
2
.
Problema 1 7,0 % do total
Os xidos de nitrognio, poluentes comuns do ar atmosfrico so formados principalmente por
xido ntrico, NO, e dixido de nitrognio, NO
2
. O xido ntrico atmosfrico essencialmente
formado durante as tempestades e nos motores de combusto interna. Em altas temperaturas o
NO reage com H
2
e produz xido nitroso, N
2
O, um gs de efeito estufa.
2 NO(g) + H
2
(g) N
2
O(g) + H
2
O(g)
No estudo da cintica desta reao a 820 C, as velocidades iniciais para a formao de N
2
O
foram medidas usando vrias presses parciais iniciais de NO e H
2
.
Exp.
Presso inicial, torr
Velocidade inicial de produo
de N
2
O, torrs
-1
P
NO

H
2
1 120,0 60,0 8,6610
-2
2 60,0 60,0 2,1710
-2
3 60,0 180,0 6,6210
-2
Neste problema no use concentraes. Use unidades de presso em torr e tempo em
segundos.
a. Determine a lei de velocidade experimental e calcule a constante de velocidade.
b. Calcule a velocidade inicial de consumo de NO, se 2,0010
2
torr de NO e 1,0010
2
torr H
2
forem misturados a 820 C. (Se voc no tiver o valor da constante de velocidade, use 210
7
com a unidade apropriada.)
c. Calcule o tempo gasto para reduzir a presso parcial de H
2
a metade do seu valor, se
8,0010
2
torr de NO e 1,0 torr de H
2
forem misturados a 820 C. (Se voc no tiver o valor da
constante de velocidade, use 210
7
com a unidade apropriada.)
a b c
d
e
Problema 1 x%
i ii iii
3 2 6 6 1,5 1 2,5 22 7,0
| 119 118 |
Programa Nacional
Olimpadas de Qumica
Ano Internacional de Qumica
Neste problema no use concentraes. Use unidades de presso em
torretempoemsegundos.
a.Determinealeidevelocidadeexperimentalecalculeaconstantedevelo-
cidade.
b.CalculeavelocidadeinicialdeconsumodeNO,se2,0010
2
torrdeNOe
1,0010
2
torrH
2
foremmisturadosa820C.(Sevocnotiverovalorda
constantedevelocidade,use2

10
-7
comaunidadeapropriada.)
c.CalculeotempogastoparareduzirapressoparcialdeH2ametadedo
seuvalor,8,00102torrdeNOe1,0torrdeH
2
foremmisturadosa820C.
(Sevocnotiverovalordaconstantedevelocidade,use210
-7
coma
unidadeapropriada.)
d. UmmecanismopropostoparaareaoentreNOeH
2
dadoabaixo:

d. Um mecanismo proposto para a reao entre NO e H


2
dado abaixo:
2 NO(g) N
2
O
2
(g)
k
1
k
-1
N
2
O
2
(g) + H
2
(g)
k
2
N
2
O(g) + H
2
O(g)
i. Deduza a lei de velocidade para a formao de N
2
O a partir do mecanismo proposto
usando a aproximao do estado estacionrio para o intermedirio.
ii. Sob que condies esta lei de velocidadede se reduz lei obtida experimentalmente na Parte a?
Se k
-1
<< k
2
P
H
2

Se k
-1
>> k
2
P
H
2

Se k
-1
> k
2

Se k
1
> k
-1


iii. Expresse a constante de velocidade determinada experimentalmente k em termos de k
1
,
k
1
e k
2
.
e. Assinale no quadro a letra correspondente ao diagrama de energia que consistente com o
mecanismo de reao proposto e com a lei de velocidade experimental.


a. b. c.
d. e. f.
energy
reaction coordinate
energy
reaction coordinate
energy
reaction coordinate
energy
reaction coordinate
energy
reaction coordinate
energy
reaction coordinate
i.DeduzaaleidevelocidadeparaaformaodeN
2
Oapartirdomeca-
nismo proposto usando a aproximao do estado estacionrio para o
intermedirio.
ii.Sobquecondiesestaleidevelocidadedesereduzleiobtidaexpe-
rimentalmentenaPartea?
Sek
-1
<<k
2
P_(
H2
)
Sek
-1
>>k
2
P_(
H_2
)
Sek
-1
>k
2
Sek
1
>k
-1


iii.Expresseaconstantedevelocidadedeterminadaexperimentalmentek
emtermosdek
1
,k
-1
ek
2
.
43
rd
IChO
| 119
Olimpada Brasileira
de Qumica 2011
Ano Internacional de Qumica
Exame Terico
e.Assinalenoquadroaletracorrespondenteaodiagramadeenergiaque
consistentecomomecanismodereaopropostoecomaleidevelocida-
deexperimental.
d. Um mecanismo proposto para a reao entre NO e H
2
dado abaixo:
2 NO(g) N
2
O
2
(g)
k
1
k
-1
N
2
O
2
(g) + H
2
(g)
k
2
N
2
O(g) + H
2
O(g)
i. Deduza a lei de velocidade para a formao de N
2
O a partir do mecanismo proposto
usando a aproximao do estado estacionrio para o intermedirio.
ii. Sob que condies esta lei de velocidadede se reduz lei obtida experimentalmente na Parte a?
Se k
-1
<< k
2
P
H
2

Se k
-1
>> k
2
P
H
2

Se k
-1
> k
2

Se k
1
> k
-1


iii. Expresse a constante de velocidade determinada experimentalmente k em termos de k
1
,
k
1
e k
2
.
e. Assinale no quadro a letra correspondente ao diagrama de energia que consistente com o
mecanismo de reao proposto e com a lei de velocidade experimental.


a. b. c.
d. e. f.
energy
reaction coordinate
energy
reaction coordinate
energy
reaction coordinate
energy
reaction coordinate
energy
reaction coordinate
energy
reaction coordinate
a) b) c) d) e) f)

| 121 120 |
Programa Nacional
Olimpadas de Qumica
Ano Internacional de Qumica
PROBLEMA2
Problema 2 7,0 % do total
a
b
Problema 2 x%
i ii iii
6 9 6 2 23 7,0
Amnia anidra um combustvel lquido alternativo muito limpo e de alta densidade de energia.
Ela no libera nenhum gs estufa na sua combusto.
Em um experimento, NH
3
gasosa queimada com O
2
em um recipiente de volume fixo de acordo
com a equao abaixo.
4 NH
3
(g) + 3 O
2
(g) 2 N
2
(g) + 6 H
2
O(l)
Os estados iniciais e finais esto a 298 K. Aps a combusto com 14,40 g de O
2
, uma parte de
NH
3
no consumida.
a. Calcule o calor liberado no processo.
Dados:
f
H(NH
3
(g)) = -46.11 kJmol
-1
e
f
H(H
2
O(l)) = -285.83 kJmol
-1
b. Para determinar a quantidade de gs NH
3
dissolvida na gua produzida durante o processo
de combusto, uma amostra de 10,00 mL da soluo aquosa foi retirada do frasco onde
ocorreu a reao e adicionada a 15,0 mL de uma soluo 0,0100 M de H
2
SO
4
. A soluo
resultante foi titulada com uma soluo padronizada 0,0200 M de NaOH e o ponto de
equivalncia foi atingido em 10,64 mL. (K
b
(NH
3
) = 1,8 10
-5
; K
a
(HSO
4
-
) = 1,1 10
-2
)
i. Calcule o pH da soluo no recipiente aps a combusto.
ii. No ponto final da titulao, ons NH
4
+
e SO
4
2-
esto presentes na soluo. Escreva as
equaes para os equilbrios relevantes que mostrem como a presena destes dois ons
afeta o pH e calcule as suas constantes de equilbrio.

iii. Assinale a alternativa que indica corretamente o pH da soluo no ponto de equivalncia.


pH > 7,0 pH =7,0 pH <7,0


Amniaanidraumcombustvellquidoalternativomuitolimpoedealta
densidadedeenergia.Elanoliberanenhumgsestufanasuacombusto.
Emumexperimento,NH
3
gasosaqueimadacomO
2
emumrecipientede
volumefxodeacordocomaequaoabaixo.
4NH
3
(g)+3O2(g) 2N
2
(g)+6H
2
O(l)
Osestadosiniciaisefnaisestoa298K.Apsacombustocom14,40gde
O
2
,umapartedeNH
3
noconsumida.
a.Calculeocalorliberadonoprocesso.
Problema 2 7,0 % do total
a
b
Problema 2 x%
i ii iii
6 9 6 2 23 7,0
Amnia anidra um combustvel lquido alternativo muito limpo e de alta densidade de energia.
Ela no libera nenhum gs estufa na sua combusto.
Em um experimento, NH
3
gasosa queimada com O
2
em um recipiente de volume fixo de acordo
com a equao abaixo.
4 NH
3
(g) + 3 O
2
(g) 2 N
2
(g) + 6 H
2
O(l)
Os estados iniciais e finais esto a 298 K. Aps a combusto com 14,40 g de O
2
, uma parte de
NH
3
no consumida.
a. Calcule o calor liberado no processo.
Dados:
f
H(NH
3
(g)) = -46.11 kJmol
-1
e
f
H(H
2
O(l)) = -285.83 kJmol
-1
b. Para determinar a quantidade de gs NH
3
dissolvida na gua produzida durante o processo
de combusto, uma amostra de 10,00 mL da soluo aquosa foi retirada do frasco onde
ocorreu a reao e adicionada a 15,0 mL de uma soluo 0,0100 M de H
2
SO
4
. A soluo
resultante foi titulada com uma soluo padronizada 0,0200 M de NaOH e o ponto de
equivalncia foi atingido em 10,64 mL. (K
b
(NH
3
) = 1,8 10
-5
; K
a
(HSO
4
-
) = 1,1 10
-2
)
i. Calcule o pH da soluo no recipiente aps a combusto.
ii. No ponto final da titulao, ons NH
4
+
e SO
4
2-
esto presentes na soluo. Escreva as
equaes para os equilbrios relevantes que mostrem como a presena destes dois ons
afeta o pH e calcule as suas constantes de equilbrio.

iii. Assinale a alternativa que indica corretamente o pH da soluo no ponto de equivalncia.


pH > 7,0 pH =7,0 pH <7,0


b.Para determinar a quantidade de gs NH
3
dissolvida na gua produzida
duranteoprocessodecombusto,umaamostrade10,00mLdasoluo
aquosafoiretiradadofrascoondeocorreuareaoeadicionadaa15,0
mLdeumasoluo0,0100MdeH
2
SO
4
.Asoluoresultantefoititulada
comumasoluopadronizada0,0200MdeNaOHeopontodeequiva-
lnciafoiatingidoem10,64mL.(K
b
(NH
3
)=1,8x10
-5
;K
a
(HSO
4-
)=1,1x10-
2
)
i.CalculeopHdasoluonorecipienteapsacombusto.
ii.No ponto fnal da titulao, ons NH
4
+
e SO
4
2-
esto presentes na so-
luo.Escrevaasequaesparaosequilbriosrelevantesquemostrem
comoapresenadestesdoisonsafetaopHecalculeassuasconstantes
deequilbrio.
iii.AssinaleaalternativaqueindicacorretamenteopHdasoluonopon-
todeequivalncia.
pH>7,0 pH=7,0 pH<7,0
43
rd
IChO
| 121
Olimpada Brasileira
de Qumica 2011
Ano Internacional de Qumica
Exame Terico
PROBLEMA3
Problema 3 8,0 % do total
a b
c
d Problema 3 x%
i ii
7 4 2 5 5 23 8,0
A zero Kelvin, a energia total de uma molcula diatmica gasosa AB dada aproximadamente
por:
E = E
o
+ E
vib
onde E
o
a energia eletrnica do estado fundamental, e E
vib
a energia vibracional.
Valores permitidos de energias vibracionais so dados pela expresso:
E
vib
= (v +
1
2
) v = 0, 1, 2,

=


(AB) =
m
A
m
B
m
A
+ m
B

Onde h a constante de Planck, v nmero quntico vibracional, k a constante de fora, e a
massa reduzida da molcula. A zero Kelvin, pode ser seguramente assumido que v zero, e E
o
e
k so independentes de substituio isotpica na molcula.
a. Calcule a variao de entalpia, H, em kJmol
-1
para a seguinte reao a 0 K.
H
2
(g) + D
2
(g) 2 HD(g)
Deutrio, D, um istopo do hidrognio com nmero de massa 2. Para a molcula de H
2
, k
575,11 Nm
-1
, e as massas molares isotpicas de H e D so 1,0078 and 2,0141 gmol
-1
,
respectivamente. Dados:
H
2
= 1,1546
HD
e
D
2
= 0,8167
HD
a zero Kelvin.
b. Calcule a frequncia, em s
-1
, de ftons infravemelhos que podem se absorvidos pela
molcula de HD. (Se voc no tem o valor para
HD
ento use 8,00010
-20
J para o clculo.)
c. As energias eletrnicas permitidas para o tomo de H so dadas pela expresso
, 2 , 1 ,
2
= = n
n
R
E
H
onde R
H
= 13,5984 eV, 1 eV = 1,60210
-19
J
i. A energia total da molcula de H
2
no estado fundamental -31,675 eV, em relao a
mesma referncia do tomo de hidrognio. Calcule a energia de dissociao, em eV, da
AzeroKelvin,aenergiatotaldeumamolculadiatmicagasosaABdada
aproximadamentepor:
E=E
o
+E
vib

onde E
o
a energia eletrnica do estado fundamental, e Evib a energia
vibracional.
Valorespermitidosdeenergiasvibracionaissodadospelaexpresso:
Problema 3 8,0 % do total
a b
c
d Problema 3 x%
i ii
7 4 2 5 5 23 8,0
A zero Kelvin, a energia total de uma molcula diatmica gasosa AB dada aproximadamente
por:
E = E
o
+ E
vib
onde E
o
a energia eletrnica do estado fundamental, e E
vib
a energia vibracional.
Valores permitidos de energias vibracionais so dados pela expresso:
E
vib
= (v +
1
2
) v = 0, 1, 2,

=


(AB) =
m
A
m
B
m
A
+ m
B

Onde h a constante de Planck, v nmero quntico vibracional, k a constante de fora, e a
massa reduzida da molcula. A zero Kelvin, pode ser seguramente assumido que v zero, e E
o
e
k so independentes de substituio isotpica na molcula.
a. Calcule a variao de entalpia, H, em kJmol
-1
para a seguinte reao a 0 K.
H
2
(g) + D
2
(g) 2 HD(g)
Deutrio, D, um istopo do hidrognio com nmero de massa 2. Para a molcula de H
2
, k
575,11 Nm
-1
, e as massas molares isotpicas de H e D so 1,0078 and 2,0141 gmol
-1
,
respectivamente. Dados:
H
2
= 1,1546
HD
e
D
2
= 0,8167
HD
a zero Kelvin.
b. Calcule a frequncia, em s
-1
, de ftons infravemelhos que podem se absorvidos pela
molcula de HD. (Se voc no tem o valor para
HD
ento use 8,00010
-20
J para o clculo.)
c. As energias eletrnicas permitidas para o tomo de H so dadas pela expresso
, 2 , 1 ,
2
= = n
n
R
E
H
onde R
H
= 13,5984 eV, 1 eV = 1,60210
-19
J
i. A energia total da molcula de H
2
no estado fundamental -31,675 eV, em relao a
mesma referncia do tomo de hidrognio. Calcule a energia de dissociao, em eV, da
OndehaconstantedePlanck,vnmeroqunticovibracional,kacons-
tantedefora,emamassareduzidadamolcula.AzeroKelvin,podeser
seguramenteassumidoquevzero,eE
o
eksoindependentesdesubstitui-
oisotpicanamolcula.
a.Calculeavariaodeentalpia,D H,emkJmol-1paraaseguintereaoa
0K.
H
2
(g)+D2(g) 2HD(g)
Deutrio,D,umistopodohidrogniocomnmerodemassa2.Paraa
molculadeH
2
,k575,11Nm
-1
,easmassasmolaresisotpicasdeHe
Dso1,0078and2,0141gmol
-1
,respectivamente.Dados:
Problema 3 8,0 % do total
a b
c
d Problema 3 x%
i ii
7 4 2 5 5 23 8,0
A zero Kelvin, a energia total de uma molcula diatmica gasosa AB dada aproximadamente
por:
E = E
o
+ E
vib
onde E
o
a energia eletrnica do estado fundamental, e E
vib
a energia vibracional.
Valores permitidos de energias vibracionais so dados pela expresso:
E
vib
= (v +
1
2
) v = 0, 1, 2,

=


(AB) =
m
A
m
B
m
A
+ m
B

Onde h a constante de Planck, v nmero quntico vibracional, k a constante de fora, e a
massa reduzida da molcula. A zero Kelvin, pode ser seguramente assumido que v zero, e E
o
e
k so independentes de substituio isotpica na molcula.
a. Calcule a variao de entalpia, H, em kJmol
-1
para a seguinte reao a 0 K.
H
2
(g) + D
2
(g) 2 HD(g)
Deutrio, D, um istopo do hidrognio com nmero de massa 2. Para a molcula de H
2
, k
575,11 Nm
-1
, e as massas molares isotpicas de H e D so 1,0078 and 2,0141 gmol
-1
,
respectivamente. Dados:
H
2
= 1,1546
HD
e
D
2
= 0,8167
HD
a zero Kelvin.
b. Calcule a frequncia, em s
-1
, de ftons infravemelhos que podem se absorvidos pela
molcula de HD. (Se voc no tem o valor para
HD
ento use 8,00010
-20
J para o clculo.)
c. As energias eletrnicas permitidas para o tomo de H so dadas pela expresso
, 2 , 1 ,
2
= = n
n
R
E
H
onde R
H
= 13,5984 eV, 1 eV = 1,60210
-19
J
i. A energia total da molcula de H
2
no estado fundamental -31,675 eV, em relao a
mesma referncia do tomo de hidrognio. Calcule a energia de dissociao, em eV, da
azeroKelvin.
| 123 122 |
Programa Nacional
Olimpadas de Qumica
Ano Internacional de Qumica
b.Calculeafrequncia,ems-1,deftonsinfravemelhosquepodemseab-
sorvidospelamolculadeHD.(Sevocnotemovalorparae
HD
entouse
8,000

10-
20
Jparaoclculo.)
c.AsenergiaseletrnicaspermitidasparaotomodeHsodadaspelaex-
presso
Problema 3 8,0 % do total
a b
c
d Problema 3 x%
i ii
7 4 2 5 5 23 8,0
A zero Kelvin, a energia total de uma molcula diatmica gasosa AB dada aproximadamente
por:
E = E
o
+ E
vib
onde E
o
a energia eletrnica do estado fundamental, e E
vib
a energia vibracional.
Valores permitidos de energias vibracionais so dados pela expresso:
E
vib
= (v +
1
2
) v = 0, 1, 2,

=


(AB) =
m
A
m
B
m
A
+ m
B

Onde h a constante de Planck, v nmero quntico vibracional, k a constante de fora, e a
massa reduzida da molcula. A zero Kelvin, pode ser seguramente assumido que v zero, e E
o
e
k so independentes de substituio isotpica na molcula.
a. Calcule a variao de entalpia, H, em kJmol
-1
para a seguinte reao a 0 K.
H
2
(g) + D
2
(g) 2 HD(g)
Deutrio, D, um istopo do hidrognio com nmero de massa 2. Para a molcula de H
2
, k
575,11 Nm
-1
, e as massas molares isotpicas de H e D so 1,0078 and 2,0141 gmol
-1
,
respectivamente. Dados:
H
2
= 1,1546
HD
e
D
2
= 0,8167
HD
a zero Kelvin.
b. Calcule a frequncia, em s
-1
, de ftons infravemelhos que podem se absorvidos pela
molcula de HD. (Se voc no tem o valor para
HD
ento use 8,00010
-20
J para o clculo.)
c. As energias eletrnicas permitidas para o tomo de H so dadas pela expresso
, 2 , 1 ,
2
= = n
n
R
E
H
onde R
H
= 13,5984 eV, 1 eV = 1,60210
-19
J
i. A energia total da molcula de H
2
no estado fundamental -31,675 eV, em relao a
mesma referncia do tomo de hidrognio. Calcule a energia de dissociao, em eV, da
i.A energia total da molcula de H
2
no estado fundamental -31,675
eV,emrelaoamesmarefernciadotomodehidrognio.Calculea
energia de dissociao, em eV, da molcula de hidrognio no estado
fundamentaldeformaqueostomosdeHsejamproduzidosnosseus
estadosfundamentais.
ii. Uma molcula de H
2
no estado fundamental dissocia em seus to-
mosapsaabsorodeftonsnocomprimentodeondade77,0nm.
Determinetodasaspossibilidadesparaosestadoseletrnicosdosto-
mosdeHproduzidos.Emcadacaso,qualaenergiacintica,emeV,
dostomosdehidrogniodissociados?
d.CalculeaafnidadeeletrnicadoonH
2
+
,emeV,sabendoquesuaenergia
dedissociaode2,650eV.(Sevocnotemovalordaenergiadedis-
sociaodoH
2
entouse4,500eVparaoclculo.)
43
rd
IChO
| 123
Olimpada Brasileira
de Qumica 2011
Ano Internacional de Qumica
Exame Terico
PROBLEMA4
9,0%dototal
a b c d e f g Problema4 x%
4 3 6 3 4 6 4 30 9,0
Para a energia sustentvel, o hidrognio torna-se a melhor opo. O mais
efcientecaminhoparaousodehidrognioageraodeenergiaeltrica
emclulascombustveis.Entretanto,estocarhidrognioemgrandesquanti-
dadesumdesafonaaplicaodeclulascombustveis.Dentreoshidretos
qumicos, considerados como um material para estocagem de hidrognio
slido, o boro-hidreto de sdio (NaBH
4
), sendo no-txico, estvel e no-
agressivo ao meio ambiente, aparece como o mais promissor. A hidrlise
deboro-hidretodesdio,queliberaH
2
,umareaolentatemperatura
ambientee,assim,precisasercatalisada.
Nanoclusters de rutnio(0) so os catalisadores mais ativos nestas hidrli-
ses,temperaturaambiente,elevacompletaliberaodeH
2
apartirdo
boro-hidretodesdio.Estudoscinticosmostramqueahidrlisecataltica
deNaBH
4
deprimeiraordememrelaoaocatalisador,pormdeordem
zeroemrelaoaosubstrato.Avelocidadedeproduodehidrogniopor
molderutniode92molH
2-
(molRu)
-1
-
min
-1
a25
0
C.
a.Calculeaquantidadedocatalisadorderutnio(emmg)quedeveseradi-
cionadoa0,100LdeumasoluodeNaBH
4
1,0mol-L
-1
paraproduziro
hidrogniogasoso,necessrioparaumaclulacombustvelporttil,auma
velocidadede0,100L-min
-1
a25Ce1,0atm.
b.Durantequantosminutosestesistemaproduzirhidrogniogasosoaesta
velocidade?
c.AenergiadeativaodeArrheniusparaestahidrlisecatalticadeboro-
hidretodesdiodeE
a
=42,0kJ-mol
-1
.Calculeatemperaturanecess-
riaparaobteramesmavelocidadedeproduodehidrognio,usandoa
metade da quantidade de catalisador de rutnio usada anteriormente a
25,0C.
| 125 124 |
Programa Nacional
Olimpadas de Qumica
Ano Internacional de Qumica
d.Aclulacombustvelconstitudadetrssegmentosjuntos(naformade
sanduiche):oanodo,oeletrlito,eocatodo.Ohidrogniousadocomo
combustveleoxigniocomooxidante.Nasinterfacesdostrsdiferentes
segmentosocorremduasreaesqumicas.
O
2
(g)+2H
2
O(l)+4e
-
4OH
-
(aq)
H
2
(g)+2OH
-
(aq) 2H
2
O(l)+2e
-

Oresultadoliquidodasduasrees:
2H
2
(g)+O
2
(g) 2H
2
O(l)
Ohidrognioparaaclulacombustvelproduzidoapartirdahidrlisedo
boro-hidretodesdio.
Calculeopotencialpadroparaasemi-reaodocatodoseopotencialpa-
drodereduoparaasemi-reaodoanodode-0,83D
f
G(H
2
O(l)de
-237kJ-mol
-1
.
e.Calculeovolumedeara25Ce1,0atmnecessrioparagerarumacor-
renteconstantede2,5Apor3,0hnestaclulacombustvel.Considereque
oarcontm20%deO
2
(g)porvolume.
f.Aefcinciadeumaclulacombustveldadapelarazoentreotrabalho
produzidoeocalordissipadopelareaodaclula.Assim,aefcincia
mximaparaaclulacombustveldadapor:
electrlito
O
2 H
2
O
anodo
catodo
e
-
H
2
e
-
e
-
e
-
e
-
e
-
H
+
b. Durante quantos minutos este sistema produzir hidrognio gasoso a esta velocidade?
c. A energia de ativao de Arrhenius para esta hidrlise cataltica de boro-hidreto de sdio de
E
a
= 42,0 kJmol
-1
. Calcule a temperatura necessria para obter a mesma velocidade de
produo de hidrognio, usando a metade da quantidade de catalisador de rutnio usada
anteriormente a 25,0 C.
d. A clula combustvel constituda de trs
segmentos juntos (na forma de sanduiche): o
anodo, o eletrlito, e o catodo. O hidrognio
usado como combustvel e oxignio como
oxidante. Nas interfaces dos trs diferentes
segmentos ocorrem duas reaes qumicas.
O
2
(g) + 2H
2
O(l) + 4e
-
4OH
-
(aq)
H
2
(g) + 2OH
-
(aq) 2H
2
O(l) + 2e
-
O resultado liquido das duas rees :
2 H
2
(g) + O
2
(g) 2 H
2
O(l)
O hidrognio para a clula combustvel produzido a partir da hidrlise do boro-hidreto de
sdio.
Calcule o potencial padro para a semi-reao do catodo se o potencial padro de reduo
para a semi-reao do anodo de 0,83 V e
f
G (H
2
O(l)) de -237 kJmol
-1
.
e. Calcule o volume de ar a 25 C e 1,0 atm necessrio para gerar uma corrente constante de
2,5 A por 3,0 h nesta clula combustvel. Considere que o ar contm 20% de O
2
(g) por
volume.
f. A eficincia de uma clula combustvel dada pela razo entre o trabalho produzido e o calor
dissipado pela reao da clula. Assim, a eficincia mxima para a clula combustvel dada
por:

clula combustvel
=
trabalho
calor
Calcule a eficincia mxima para a clula combustvel usando os dados fornecidos abaixo, a
25 C e presso padro.
S (Jmol
-1
K
-1
)
H
2
(g) 130,7
O
2
(g) 205,2
H
2
O(l) 70,0
g. A segunda lei da termodinmica diz que impossvel converter todo o calor, q
H
, de um
reservatrio a alta temperatura, T
H
, em trabalho. Pelo menos, parte da energia, q
C
, deve ser
transferida para um reservatrio de baixa temperatura, T
C
. Assim, uma mquina trmica
(motor) com eficincia de 100% termodinamicamente imposssvel. Quando a mquina
f. Calcule a efcincia mxima para a clula combustvel usando os dados
fornecidosabaixo,a25Cepressopadro.
electrlito
O
2 H
2
O
anodo
catodo
e
-
H
2
e
-
e
-
e
-
e
-
e
-
H
+
b. Durante quantos minutos este sistema produzir hidrognio gasoso a esta velocidade?
c. A energia de ativao de Arrhenius para esta hidrlise cataltica de boro-hidreto de sdio de
E
a
= 42,0 kJmol
-1
. Calcule a temperatura necessria para obter a mesma velocidade de
produo de hidrognio, usando a metade da quantidade de catalisador de rutnio usada
anteriormente a 25,0 C.
d. A clula combustvel constituda de trs
segmentos juntos (na forma de sanduiche): o
anodo, o eletrlito, e o catodo. O hidrognio
usado como combustvel e oxignio como
oxidante. Nas interfaces dos trs diferentes
segmentos ocorrem duas reaes qumicas.
O
2
(g) + 2H
2
O(l) + 4e
-
4OH
-
(aq)
H
2
(g) + 2OH
-
(aq) 2H
2
O(l) + 2e
-
O resultado liquido das duas rees :
2 H
2
(g) + O
2
(g) 2 H
2
O(l)
O hidrognio para a clula combustvel produzido a partir da hidrlise do boro-hidreto de
sdio.
Calcule o potencial padro para a semi-reao do catodo se o potencial padro de reduo
para a semi-reao do anodo de 0,83 V e
f
G (H
2
O(l)) de -237 kJmol
-1
.
e. Calcule o volume de ar a 25 C e 1,0 atm necessrio para gerar uma corrente constante de
2,5 A por 3,0 h nesta clula combustvel. Considere que o ar contm 20% de O
2
(g) por
volume.
f. A eficincia de uma clula combustvel dada pela razo entre o trabalho produzido e o calor
dissipado pela reao da clula. Assim, a eficincia mxima para a clula combustvel dada
por:

clula combustvel
=
trabalho
calor
Calcule a eficincia mxima para a clula combustvel usando os dados fornecidos abaixo, a
25 C e presso padro.
S (Jmol
-1
K
-1
)
H
2
(g) 130,7
O
2
(g) 205,2
H
2
O(l) 70,0
g. A segunda lei da termodinmica diz que impossvel converter todo o calor, q
H
, de um
reservatrio a alta temperatura, T
H
, em trabalho. Pelo menos, parte da energia, q
C
, deve ser
transferida para um reservatrio de baixa temperatura, T
C
. Assim, uma mquina trmica
(motor) com eficincia de 100% termodinamicamente imposssvel. Quando a mquina
43
rd
IChO
| 125
Olimpada Brasileira
de Qumica 2011
Ano Internacional de Qumica
Exame Terico
g. A segunda lei da termodinmica diz que impossvel converter todo o
calor, qH, de um reservatrio a alta temperatura, T
H
, em trabalho. Pelo
menos,partedaenergia,qC,devesertransferidaparaumreservatriode
baixatemperatura,T
C
.Assim,umamquinatrmica(motor)comefcincia
de100%termodinamicamenteimposssvel.Quandoamquinatrmica
(motor)esttrabalhandoreversivelmente,comoemumciclodeCarnot,a
efcinciasermxima.
Para o motor trabalhar reversivelmente entre dois re-
servatrios,aplica-seaseguinterelao:

h.Qualdeveseratemperaturadoreservatrioquente,T
H
,deumamquina
trmicadeCarnotparamanteraefcinciadaclulacombustvelcalculada
noitem(f),seatemperaturadoreservatriofrioT
C
de40C?(Sevoc
notemovalorparaaefcincia,entouse0,80).
| 127 126 |
Programa Nacional
Olimpadas de Qumica
Ano Internacional de Qumica
PROBLEMA5
7.0%dototal
a b c d e f g Problema5 x%
i ii 1 6 5 2 2 6 30 7.0
5 3

CompostosdePolinitrogniotmsidopotencialmenteusadocomomateriais
dealtadensidadedeenergia.Elessotermodinamicamenteinstveis.Uma
grandequantidadedeenergialiberadadesuareaodedecomposiole-
vandoprodutosmaisestveis.Asespciesdepolinitrognioconhecidasso
somenteN
2
,N
3
-
andN
5
+
,isoladasem1772,1890e1999,respectivamente,e
recentementeonioncclico,N
5
-
.
a.(i)EscrevaaestruturadeLewisparaN
5
+
comastresformasderessonncia
energeticamentefavorveis.Indiqueosparesisoladoseascargasformais.
DesenheageometriamoleculardoN
5
+
cclico.
(ii) Escreva a estrutura de Lewis para o N
5
+
cclico com as cinco formas
deressonnciaenergeticamentefavorveis.Indiqueosparesisoladoseas
cargasformais.DesenheageometriamoleculardoN
5
-
cclico.
b.Asntesedo[N
5
+
][AsF
6
-
],umslidoinicobranco,foiobtidapelareao
do [N
2
F
+
][AsF
6
-
] com o cido hidrazico, HN
3
, no lquido HF a -78 C.
Escrevaaequaoqumicabalanceadaparaestareao.
Apreparaodo[N
2
F
+
][AsF
6
-
]requerareaodoN
2
F
2
comofortecidode
Lewis,AsF
5
,comosegue:
xC(grafte)+AsF
5
C
x
-AsF
5
(intercaladocomgraftex=10
-12
)
2C
x
-AsF
5
+N
2
F
4
2[Cx
+
][AsF
6
-
]+trans-N
2
F
2
trans-N
2
F
2
+AsF
5
[N
2
F
+
][AsF
6
-
]
NasntesedoN
2
F
2
,formadooismerotrans,quetermodinamicamente
menosestveldoqueoismerocis-N
2
F
2
.Contudo,aconversodotrans-
N
2
F
2
paraocis-N
2
F2passaporumaaltabarreiraenergticade251kJ/mol,
43
rd
IChO
| 127
Olimpada Brasileira
de Qumica 2011
Ano Internacional de Qumica
Exame Terico
deformaque,oequilbrioentreosismeroscisetransnoacontecesignif-
cativamentesemousodeumcatalizadorapropriado.
Quandootrans-N
2
F
2
mantidoemumrecipientefechadopor6dias,tem-
peratura ambiente, em presena de pequenas quantidades de SbF
5
como
catalizador,oequilbriotrmicocis-transestabelecido.

A entalpia padro de formao do trans- e cis-N


2
F
2
so, respectivamente,
67,31e62,03kJ/mol,esuasentropiaspadroa25Csorespectivamente,
262,10e266,50JK
-1
mol
-1
.
c.Encontrearazodonmerodemolculasdocis-N
2
F
2
sobreonmerode
molculasdotrans-N
2
F
2
namisturaemequilbrio,a25C.
d.EscrevaasestruturasdeLewismostrandoageometriadoonN
2
F
+
edos
ismeroscis-transdoN
2
F
2
.Incluatodososparesisoladoseascargasfor-
mais.SugiraumahibridizaoparacadatomodenitrognionoN
2
F
2
e
noN
2
F
+
.
Oslido[N
5
+
][AsF
6
-
]umpoucoestveltemperaturaambiente,masreage
explosivamentecomguaparaproduzirpentafuoretodearsnico,fuoreto
dehidrognio,nitrogniomoleculareoxigniomolecular.
e.Escrevaaequaobalanceadaparaareaoentre[N
5
+
][AsF
6
-
]egua.
Aconversodo[N
5
+
][SbF
6
-
]emoutrossaisdeN
5
+
podeserfeitapelareao
demettese:
[N
5
+
][SbF
6
-
]+[M
+
][X
-
] [N
5
+
][X
-
]+[M
+
][SbF
6
-
]
M
+
=Na
+
,K
+
,Cs
+
;X
-
=umniongrandetalcomoSnF
6
2-
andB(CF
3
)
4
-
.
Desdequeo[Cs
+
][SbF
6
-
]temumabaixasolubilidadeemHFanidroe[K
+
]
[SbF
6
-
]temumabaixasolubilidadeemSO
2
,estesdoissolventesforamusados
extensivamenteparaproduzir,respectivamente,areaodemettesea-78
Cand-64C.
f. Escrevaaequaobalanceadaparaapreparaodo[N
5
+
]
2
[SnF
6
2-
]e[N
5
+
]
[B(CF
3
)
4
-
]emsoluo,iniciando,respectivamente,com[Cs
+
]
2
[SnF
6
2-
]e[K
+
]
[B(CF
3
)
4
-
].Indiqueosolventeapropriado.
| 129 128 |
Programa Nacional
Olimpadas de Qumica
Ano Internacional de Qumica
Quandoo[N
5
+
]2[SnF
6
2-
]sedecompesobcondiescuidadosamentecon-
troladas,a25-30C,[N
5
+
][SnF
5
-
]eN
5
Fsoformados.Osal[N
5
+
][SnF
5
-
]um
slidobrancoetemumaestabilidadetrmicacomparveldo[N
5
+
][SbF
6
-
]
(5060C).OespectrodeRMNdasoluodo
119
Sntemmostradoqueo
nionSnF
5
-
nestecomposto,defato,umamisturadepolinionsdimricos
etetramricos.Emambosospolinions,onmerodecoordenaodoto-
modeSn6eexistempontesdetomosdefor.
g.Desenheasestruturasdospolinionsdimricosetetramtricos.
PROBLEMA6
7,0%dototal
a b c d e f g Problema6 x%
5 3 4 2 5 3 1 23 7,0
A extrao do ouro usando cianeto de sdio, um reagente muito txico,
causaproblemasambientaiseprovocasriaspreocupaesnaspopulaes
devidoaousodestemtodo,usualmentedesignadoporprocessocianeto.
A extrao do ouro com tiossulfato tem sido considerada uma alternativa.
Nesteprocessooreagenteprincipalotiossulfatodeamnio,(NH
4
)2S
2
O
3
,
quenotxico.Apesardesteprocessoseramigodoambiente,aqumica
envolvidacomplexaenecessitaserbemestudada.Asoluousadaparaa
extraodoourocontmS
2
O
3
2-
,Cu
2+
,NH
3
,eO2dissolvido.Asoluodeve
terumpHsuperiora8,5eamnialivre.
Deacordocomomecanismoproposto,forma-seumamicro-clulavoltaica
na superfcie das partculas de ouro durante o processo de extrao que
podesertraduzidadaseguinteforma:
Anodo:
complexa e necessita ser bem estudada. A soluo usada para a extrao do ouro contm
S
2
O
3
2-
, Cu
2+
, NH
3
, e O
2
dissolvido. A soluo deve ter um pH superior a 8,5 e amnia livre.
De acordo com o mecanismo proposto, forma-se uma micro-clula voltaica na superfcie das
partculas de ouro durante o processo de extrao que pode ser traduzida da seguinte forma:
Anodo
Au(s) + 2 NH
3
(aq) [Au(NH
3
)
2
]
+
(aq) + e
-
:
[Au(NH
3
)
2
]
+
(aq) + 2 S
2
O
3
2-
(aq) [Au(S
2
O
3
)
2
]
3-
(aq) + 2 NH
3
(aq)
[Cu(NH
3
)
4
]
2+
(aq) + e
-
[Cu(NH
3
)
2
]
+
(aq) + 2 NH
3
(aq)
Catodo:
[Cu(NH
3
)
2
]
+
(aq) + 3 S
2
O
3
2-
(aq) [Cu(S
2
O
3
)
3
]
5-
(aq) + 2 NH
3
(aq)
a. Escreva a equao que traduz a reao global da clula voltaica.
b. Na presena de amnia, o O
2
oxida o [Cu(S
2
O
3
)
3
]
5-
a [Cu(NH
3
)
4
]
2+
. Escreva a equao qumica
devidamente balanceada que traduz esta reao de oxidorreduo em meio bsico.
c. Neste processo de extrao, o on complexo [Cu(NH
3
)
4
]
2+
funciona como catalisador,
acelerando a dissoluo do ouro. Escreva a equao qumica global que traduz a reao de
oxidorreduo da dissoluo do ouro metlico, quando catalisada pelo on complexo
[Cu(NH
3
)
4
]
2+
d.
Escreva a geometria de coordenao do metal nos ons complexos [Au(NH
3
)
2
]
+
e
[Au(S
2
O
3
)
2
]
3-
, e indique os tomos de coordenao.



e. As constantes de formao, K
f
, dos ons complexos [Au(NH
3
)
2
]
+
e [Au(S
2
O
3
)
2
]
3-
so 1,0010
26
e 1,0010
28
, respectivamente. Considere a soluo de extrao, na qual as concentraes
das espcies em equilbrio so as seguintes:
[S
2
O
3
2-
] = 0,100 M; [NH
3
] = 0,100 M; concentrao total de espcies de ouro(I) = 5,5010
-5
M.
Calcule a porcentagem do on ouro(I) que existe na forma de complexo com tiossulfato.
Quando a concentrao de O
2
no suficiente e o pH superior a 10, a espcie S
2
O
3
2-
reduz o
on [Cu(NH
3
)
4
]
2+
a [Cu(S
2
O
3
)
3
]
5-
com a formao do on tetrationato, S
4
O
6
2-
:
2 [Cu(NH
3
)
4
]
2+
(aq) + 8 S
2
O
3
2-
(aq) 2 [Cu(S
2
O
3
)
3
]
5-
(aq) + S
4
O
6
2-
(aq) + 8 NH
3
(aq)
Numa soluo alcalina o on tetrationato desproporciona a on tritionato, S
3
O
6
2-
, e a
tiossulfato.
Escreva a equao qumica devidamente balanceada que representa esta reao de
desproporcionamento
43
rd
IChO
| 129
Olimpada Brasileira
de Qumica 2011
Ano Internacional de Qumica
Exame Terico
Catodo:
complexa e necessita ser bem estudada. A soluo usada para a extrao do ouro contm
S
2
O
3
2-
, Cu
2+
, NH
3
, e O
2
dissolvido. A soluo deve ter um pH superior a 8,5 e amnia livre.
De acordo com o mecanismo proposto, forma-se uma micro-clula voltaica na superfcie das
partculas de ouro durante o processo de extrao que pode ser traduzida da seguinte forma:
Anodo
Au(s) + 2 NH
3
(aq) [Au(NH
3
)
2
]
+
(aq) + e
-
:
[Au(NH
3
)
2
]
+
(aq) + 2 S
2
O
3
2-
(aq) [Au(S
2
O
3
)
2
]
3-
(aq) + 2 NH
3
(aq)
[Cu(NH
3
)
4
]
2+
(aq) + e
-
[Cu(NH
3
)
2
]
+
(aq) + 2 NH
3
(aq)
Catodo:
[Cu(NH
3
)
2
]
+
(aq) + 3 S
2
O
3
2-
(aq) [Cu(S
2
O
3
)
3
]
5-
(aq) + 2 NH
3
(aq)
a. Escreva a equao que traduz a reao global da clula voltaica.
b. Na presena de amnia, o O
2
oxida o [Cu(S
2
O
3
)
3
]
5-
a [Cu(NH
3
)
4
]
2+
. Escreva a equao qumica
devidamente balanceada que traduz esta reao de oxidorreduo em meio bsico.
c. Neste processo de extrao, o on complexo [Cu(NH
3
)
4
]
2+
funciona como catalisador,
acelerando a dissoluo do ouro. Escreva a equao qumica global que traduz a reao de
oxidorreduo da dissoluo do ouro metlico, quando catalisada pelo on complexo
[Cu(NH
3
)
4
]
2+
d.
Escreva a geometria de coordenao do metal nos ons complexos [Au(NH
3
)
2
]
+
e
[Au(S
2
O
3
)
2
]
3-
, e indique os tomos de coordenao.



e. As constantes de formao, K
f
, dos ons complexos [Au(NH
3
)
2
]
+
e [Au(S
2
O
3
)
2
]
3-
so 1,0010
26
e 1,0010
28
, respectivamente. Considere a soluo de extrao, na qual as concentraes
das espcies em equilbrio so as seguintes:
[S
2
O
3
2-
] = 0,100 M; [NH
3
] = 0,100 M; concentrao total de espcies de ouro(I) = 5,5010
-5
M.
Calcule a porcentagem do on ouro(I) que existe na forma de complexo com tiossulfato.
Quando a concentrao de O
2
no suficiente e o pH superior a 10, a espcie S
2
O
3
2-
reduz o
on [Cu(NH
3
)
4
]
2+
a [Cu(S
2
O
3
)
3
]
5-
com a formao do on tetrationato, S
4
O
6
2-
:
2 [Cu(NH
3
)
4
]
2+
(aq) + 8 S
2
O
3
2-
(aq) 2 [Cu(S
2
O
3
)
3
]
5-
(aq) + S
4
O
6
2-
(aq) + 8 NH
3
(aq)
Numa soluo alcalina o on tetrationato desproporciona a on tritionato, S
3
O
6
2-
, e a
tiossulfato.
Escreva a equao qumica devidamente balanceada que representa esta reao de
desproporcionamento
a.Escrevaaequaoquetraduzareaoglobaldaclulavoltaica.
b.Napresenadeamnia,oO
2
oxidao[Cu(S
2
O
3
)
3
]
5-
a[Cu(NH
3
)
4
]
2+
.Escreva
a equao qumica devidamente balanceada que traduz esta reao de
oxidorreduoemmeiobsico.
c.Nesteprocessodeextrao,ooncomplexo[Cu(NH
3
)
4
]
2+
funcionacomo
catalisador,acelerandoadissoluodoouro.Escrevaaequaoqumica
globalquetraduzareaodeoxidorreduodadissoluodoouromet-
lico,quandocatalisadapelooncomplexo[Cu(NH
3
)
4
]
2+

d.Escreva a geometria de coordenao do metal nos ons complexos


[Au(NH
3
)
2
]
+
e[Au(S
2
O
3
)
2
]
3-
,eindiqueostomosdecoordenao.
e.As constantes de formao, Kf, dos ons complexos [Au(NH
3
)
2
]
+
e
[Au(S
2
O
3
)
2
]
3-
so1,0010
26
e1,0010
28
,respectivamente.Considereaso-
luodeextrao,naqualasconcentraesdasespciesemequilbrioso
asseguintes:
[S
2
O
3
2-
] = 0,100 M; [NH
3
] = 0,100 M; concentrao total de espcies de
ouro(I)=5,5010
-5
M.
Calcule a porcentagem do on ouro(I) que existe na forma de complexo
comtiossulfato.
f. QuandoaconcentraodeO
2
nosufcienteeopHsuperiora10,a
espcieS
2
O
3
2-
reduzoon[Cu(NH
3
)
4
]
2+
a[Cu(S
2
O
3
)
3
]
5-
comaformaodo
ontetrationato,S
4
O
6
2-
:
2[Cu(NH
3
)
4
]
2+
(aq) + 8 S
2
O
3
2-
(aq) 2 [Cu(S
2
O
3
)
3
]
5-
(aq) + S
4
O
6
2-
(aq) + 8
NH
3
(aq)
Numasoluoalcalinaoontetrationatodesproporcionaaontritionato,
S
3
O
6
2-
,eatiossulfato.
Escrevaaequaoqumicadevidamentebalanceadaquerepresentaesta
reaodedesproporcionamento
| 131 130 |
Programa Nacional
Olimpadas de Qumica
Ano Internacional de Qumica
g.QuandoaconcentraodeO
2
muitoelevada,oonS
2
O
3
2-
oxida-seaos
onstritionatoesulfato.Escrevaaequaoqumicadevidamentebalance-
adaquetraduzestareao.
PROBLEMA7
8,5%dototal
A S B C D E F G
1
G
2
H I J K L M 1a 1b Problema7 x%
2 2 2 2 2 2 1 1 1 2 2 2 1 1 1 1 1 26 8,5
Sntesedeumcarbasugar
Oshidratosdecarbonosocomponentesessenciaisparaasclulasefontede
energiaparaosanimais.Nestaclasseestoincluidosdesdeacaressimples
compequenasmolculasatestruturasmacromoleculares.Nomonossaca-
rdeo,quandoooxigniodoanel(oxignioendocclico)substituidopelo
grupo metilnico, formam-se compostos designados por pseudosugars
oucarbasugars.
Umavezqueoscarbasugarsnosohidrolisadosnapresenadecidose
enzimas,elestmsidoaplicadoscomoinibidoresdeglicosidases.
A sntese total de duas formas isomricas do carbasugars 1 descrita a
seguir.
43
rd
IChO
| 131
Olimpada Brasileira
de Qumica 2011
Ano Internacional de Qumica

1
Asntesede1inicia-secomareduodobenzenocomsdionapresenade
amnialquidaobtendo-seocompostoA.OespectrodeRMNdeC-13deA
apresentadoissinaisem124,0e26,0ppm.
OcloretodetricloroacetilaempresenadeZnoriginaaespciereativaS.Um
equivalentedeSpromoveumaciclo-adio[2+2]comAeformaumprodu-
toracmicoB.AreaodeBcomZnemcidoacticooriginaocomposto
C.OcompostoCcontmapenastomosdecarbono,hidrognioeoxignio.
O espectro de RMN de C-13 de C apresenta trs sinais correspondentes a
carbonossp2,em210,0;126,5e125,3ppm.

AreaodeCcomumequivalentedecidom-cloroperbenzico(m-CPBA)
em diclorometano origina majoritariamente o composto D. O espectro de
RMNdeC-13deDapresenta,tambm,trssinaisnaregiodecarbonossp
2
em177,0;125,8e124,0ppm.
EscrevaasestruturasdoscompostosA,B,C,D,edointermedirioS.
AreduodocompostoDcomLiAlH
4
originaE,quereagecomumexcesso
decloretodeacetilaempiridinaparadarocompostoF.Escrevaasestruturas
(useapenasumenantimero)deEeF,usandoasnotaescunhatracejada
ecunhacheia.Indiqueainda,paraocompostoEasconfguraes(RouS)
doscarbonosassimticos.
OcompostoF(utilizeoenantimeroanteriormenterepresentado)reagecom
Exame Terico
| 133 132 |
Programa Nacional
Olimpadas de Qumica
Ano Internacional de Qumica
bromooriginandoosestereoismerosG
1
eG
2
.Escrevaasestruturasdestes
compostosusandonovamenteanotaotracejada/cheia.
AmisturadeG
1
eG
2
reagecomdoisequivalentesde1,8-diazabiciclo[5.4.0]
undec-7-eno (DBU), que uma amina e simultaneamente uma base forte,
originandoocompostoH.EscrevaaestruturadocompostoHusandonova-
menteanotaotracejada/cheia.

Escreva as estruturas dos compostos E, F, G


1
, G
2
e H usando a notao de
cunhatracejada/cheia.
AreaodocompostoHcomoxigniosingleto(originadonomeioreacio-
nal)originaocompostoI.Emboranestareaosejapossvelteoricamentea
formaodedoisismeros,forma-seapenasocompostoIdevidoaoimpe-
dimentoestereoqumicoerepulsoeletrnica.
DareaodocompostoIcomexcessodeLiAlH
4
resultaaformaodocom-
postoJ.OespectrodeRMNdeC-13deJapresenta8sinais,doisdelesna
regiodecarbonossp
2
.
AreaodocompostoJcomexcessodecloretodeacetilanapresenade
piridinaoriginaocompostoK.AreaosubsequentedeKcomOsO
4
,napre-
senade4-metilmorfolina4-xido(NMO)originaosesterioismeroLeM.
PorreduodeexcessodeLiAlH
4
,oscompostosLeMoriginamosesterioi-
smeros1ae1b,respectivamente.
43
rd
IChO
| 133
Olimpada Brasileira
de Qumica 2011
Ano Internacional de Qumica
Exame Terico
EscrevaasestruturasdoscompostosI,J,K,L,M,1a,e1busandoanotao
decunhatracejada/cheia.
PROBLEMA8
6,5%dototal
B C D E F G H I J K L M Problema8
x%
2 2 2 2 1 1 1 1 1,5 1,5 1,5 1,5 18 6,5

ClickchemistryumconceitoqumicointroduzidoporK.B.Sharpless,em
2001,quedescreveumconjuntodereaesqumicasquepossibilitamaob-
tenorpida,efcienteequantitativa,deumconjuntodenovassubstncias,
por juno de pequenas unidades estruturais moleculares e em condies
suaves.Estametodologiafoirecentementeaplicadacomoumpassochave
nasntesedecompostosbicclicos.
Ocidomandlicoumcompostonaturalverstilemuitousadocomofon-
te de quiralidade em sntese orgnica. A reduo do cido (R)-mandlico
comLiBH
4
originaocompostoA.
| 135 134 |
Programa Nacional
Olimpadas de Qumica
Ano Internacional de Qumica

AreaodocompostoAcom1equivalentedecloretodep-toluenosulfonila
originaocompostoB.OaquecimentodeBempiridinaproduzocomposto
C.Duranteestatransformao,oscompostosBeCmantmassuasconfgu-
raesabsolutas.
EscrevaasestruturasdoscompostosBeCrepresentandocomaestereoqu-
micacorreta.Utilizeanotaocunhatracejada/cheianodesenhodasestru-
turasdesteproblema.
A reao do composto C com azida de sdio em soluo aquosa de ace-
tonitrila origina uma mistura de regioismeros enantiomericamente puros,
DeE,narazo3:1.Poroutrolado,ocompostoB,nasmesmascondies
reacionais,originasomenteocompostoE.

EscrevaasestruturasdoscompostosDeErepresentandocomaestereoqu-
micacorreta.
ParteI:OscompostosDeEsoseparadosatravsdareaocom3-bromo-
prop-1-inonapresenadeNaH,originandooscompostosGeG,respecti-
vamente. Os compostos G e G quando aquecidos em tolueno originam os
compostosbicclicosHeI,respectivamente.
43
rd
IChO
| 135
Olimpada Brasileira
de Qumica 2011
Ano Internacional de Qumica
Exame Terico
EscrevaasestruturasdoscompostosF,G,HeIrepresentandocomaestereo-
qumicacorreta.
ParteII:AreaodecadaumdoscompostosDeEcomacetilenodicarboxi-
latodedimetilaemguaa70Corigina,respectivamente,osregioismeros
monocclicosJeK,opticamenteativos.OscompostosJeK,quandotratados
com NaH, originam, respectivamente, os produtos bicclicos L e M, ambos
comafrmulamolecularC
13
H
11
N
3
O
4
.

EscrevaasestruturasdoscompostosJ,K,LeMrepresentandocomaestereo-
qumicacorreta.
Sejsabesoquetensafazerenoofazes,
entoestaispiorqueantes.
(Confucio).
| 137 136 |
Programa Nacional
Olimpadas de Qumica
Ano Internacional de Qumica
XVIOlimpadaIberoamericanadeQumica
ProblemasTericos
20desetembrode2011
Teresina,Piau,Brasil

PROBLEMAN1:QUMICAINORGNICA
EstequiometriaeGeometriaMolecular
6%dototal
Questes I II III IV V VI total
Pontuao 1,0 1,0 1,0 1,0 1,0 1,0 6,0
Asplantasnecessitamdediversoselementosqumicosparasuasobrevivn-
cia,soeles:carbono,hidrognio,oxignio,nitrognio,fsforo,enxofre,cl-
cio,magnsioepotssio,boro,cobalto,cobre,ferro,mangans,molibdnio
e zinco. Para suprir a defcincia desses elementos qumicos so aplicados
aosoloe/ouplantasoschamadosfertilizantesouadubossintticos,como
intuitodemelhoraraproduo.Aindstriadefertilizanteumadasmaiores
do mundo, visto que a populao precisa de alimentos mais que qualquer
outrobemdeconsumo.Snoanode2009cercade200milhesdetonela-
dasdefertilizante,daclassedepotssio,nitrognioefsforo,foramconsu-
midasnomundo,sendoaChinaomaiorconsumidor.
AindstriaBOSSproduzofertilizantechamadodesuperfosfato.Aprodu-
orealizadatratando-seofosfatodeclciocom92%depurezapelocido
sulfricoconcentrado,deacordocomaseguintereao:
Ca
3
(PO
4
)
2
(s)+2H
2
SO
4
(l)

2CaSO
4
(s)+Ca(H
2
PO
4
)
2
(s)
16
a
OIAQ
| 137
Olimpada Brasileira
de Qumica 2011
Ano Internacional de Qumica
Em uma batelada de superfosfato produzida pela BOSS, foram misturados
0,50Mg(megagrama)defosfatodeclciocom0,26Mg(megagrama)deci-
dosulfrico,obtendo-se0,28Mg(megagrama)desuperfosfatoCa(H
2
PO
4
)
2
.
Respondaositensabaixo.
I)Qualoreagentelimitante?
II)Quala%doreagenteemexcesso?
III)Qualorendimentopercentualdofosfatoemsuperfosfato?
IV)DesenhetodasaspossveisestruturasdeLewisparaoonPO
4
-3
.
V)Qualacargaformalaoredordostomosdefsforoedeoxigniono
onPO
4
-3
?
VI)QualageometriamolecularparaoonPO
4
-3
?
M(g.mol
-1
):Ca
3
(PO
4
)
2
=310,10;H
2
SO
4
=98,04;CaSO
4
=136,10;
Ca(H
2
PO
4
)
2
=233,98

PROBLEMAN2:QUMICAINORGNICA
EstruturaCristalinadexidosMetlicos
9%dototal
Questes I II III IV V total
Pontuao 2,0 2,0 1,5 1,5 2,0 9,0
Osxidosmetlicosnanoestruturados,queapresentamsuasdimensesme-
noresque100nanmetros,constituemumapromissoraclassedemateriais
avanados, que exibem propriedades fsicas e qumicas melhoradas com
aplicaesnasreasdananocinciaenanotecnologia.Dentreestesxidos,
asestruturascristalinasdotrixidodernio(ReO
3
)eoxidodetitnio(IV)
- TiO
2
apresentam clulas unitrias distintas. O TiO
2
ocorre em trs formas
cristalinas diferentes: rutilo, anatase e brookita, porm o rutilo a forma
maiscomumencontrada,sendoutilizadocomopigmentobrancoemtintas,
plsticosepapel.Aestruturacristalinadorutilo(Figura1)umexemplode
uma rede de nions de estrutura hexagonal densa com ctions ocupando
somentemetadedosburacosoctadricos.
Exame Terico
| 139 138 |
Programa Nacional
Olimpadas de Qumica
Ano Internacional de Qumica

Figura1.ClulaunitriadoTiO
2
.
OReO
3
,decoloraovermelha,oxidomaisestveldogrupo7epodeser
obtidoatravsdareaodoxidodernio(VII)commonxidodecarbono
deacordocomareao:
Re
2
O
7
+CO

2ReO
3
+CO
2
AclulaunitriadoReO3cbicacomReemcadavrticedaclulaunitria
eumtomodeOemcadaarestadaclulaunitriaameiocaminhoentreos
tomosdeRe.
I)EsboceaclulaunitriadoReO
3
.
II)CalculeonmerototaldetomospresentenaclulaunitriadoReO
3
.
III)Determineonmerodecoordenaodoctionedonionnaestrutura
doReO
3
.
IV)Determineonmerodecoordenao(N.C.)doctionedonionnaes-
truturadoTiO
2
.
V)CalculeonmerototaldetomospresentenaclulaunitriadoTiO
2
.
PROBLEMAN3:FSICOQUMICA
TermodinmicaeEquilbriodeFases
8%dototal
Questes
I II III IV total
Pontuao 2,0 2,0 2,0 2,0 8,0
16
a
OIAQ
| 139
Olimpada Brasileira
de Qumica 2011
Ano Internacional de Qumica
Osxidosmet
Porserumpasdegrandeextensoterritorialecortadopelalinhadoequador
e o trpico de capricrnio, o Brasil apresenta uma considervel diversidade
climtica.Emdecorrnciadisso,possvelobservar,emumnicodia,grandes
diferenasdetemperaturasentreduascidadesbrasileiras.Talfatopoderepre-
sentarumproblemaparaalgumasatividades,comootransportedemateriais
volteis, j que pode haver um considervel aumento da presso de vapor
destesmateriaiscomoaumentodatemperatura,oquepodeprovocarexplo-
ses.Paraevitartalconsequncia,osreservatriosdoscaminhesquetrans-
portammateriaisvolteissoprovidosdeumdispositivodesegurana,que
ativadoparaaliviarapressointernaquandoamesmaatingecertovalor.
Tendoconhecimentodasinformaesmencionadasacima,umcaminhonei-
rosaiudeCaxiasdoSul(RioGrandedoSul,Brasil),transportando1,0tde
teretlicoemumreservatriocilndricode2,0m
3
,emumdiacujatempera-
turaerade5,0C,tendocomodestinofnal,TeresinaPiau.Oreservatrio
decargadocaminhoeraprovidododispositivodeseguranamencionado
anteriormente,queeraativadoautomaticamentequandoapressointerna
doreservatrioatingisse5atm.
I) Admitindo que todo ter etlico (1,0 tonelada) tenha sido colocado no
reservatrio no estado lquido, e que o caminho tenha fcado parado
tempo sufciente em Caxias do Sul (T = -5,0 C) para que a carga tenha
entrado em equilbrio trmico com o ambiente, determine a frao, em
massa,deterqueseencontravanoestadodevapormomentosantesdo
inciodaviagemparaTeresina.
II)Considerandoasinformaesanterioresesabendoque,aochegaraTe-
resina,quatrodiasdepoisdasada,ocaminhoneirodeparou-secomuma
temperaturaambientede40C,demonstre,comosclculosnecessrios,
seavlvuladeseguranadoreservatriofoiativada,ouno,quandooca-
minhoneirochegouaTeresina.Considerequedurantetodootrajetoentre
CaxiasdoSuleTeresinaatemperaturanotenhaultrapassadoos25C.
III)Imaginequeatemperaturadocaminhotenhaatingidoumatempera-
turatal,emdeterminadasituao,queavlvuladeseguranatenhasido
ativada.Considerandoestainformaoequeovapordoteretlicocom-
porta-secomoumgsideal,determineatemperaturamnimanecessria
paraqueavlvulatenhasidoativada.
Exame Terico
| 141 140 |
Programa Nacional
Olimpadas de Qumica
Ano Internacional de Qumica
IV)Considerandoquetodaacargadeteretlicoestivessenoestadolquido
nointervalodetemperaturaentre5,0Ce10C,sobpressoconstante,
determine as variaes de entalpia (H) e de entropia (S) associadas a
estavariaodetemperatura.
Dadossobreoteretlico:
Pressodevapor=58,9kPaa20C;
p=p*e
-x
,ondepapressodevaporatemperaturaT,p*apressode
vaporatemperaturaT*e
5

ou no, quando o caminhoneiro chegou a Teresina. Considere que durante todo o trajeto entre
Caxias do Sul e Teresina a temperatura no tenha ultrapassado os 25 C.
III) Imagine que a temperatura do caminho tenha atingido uma temperatura tal, em
determinada situao, que a vlvula de segurana tenha sido ativada. Considerando esta
informao e que o vapor do ter etlico comporta-se como um gs ideal, determine a
temperatura mnima necessria para que a vlvula tenha sido ativada.
IV) Considerando que toda a carga de ter etlico estivesse no estado lquido no intervalo
de temperatura entre 5,0 C e 10 C, sob presso constante, determine as variaes de
entalpia (H) e de entropia (S) associadas a esta variao de temperatura.
Dados sobre o ter etlico:
Presso de vapor = 58,9 kPa a 20 C;
p = p*e
-x
, onde p a presso de vapor a temperatura T, p* a presso de vapor a
temperatura T* e =

H
vap
= 29,0 kJ/mol; densidade = 0,750 g/mL (-5,0 C);
T
crtica
= 193 C; P
crtica
= 3,64 MPa; R = 8,314 J mol
-1
K
-1
, 1 atm =101325 Pa
C
p,m
= 107,1 J mol
-1
K
-1
;
Para todas as respostas, considere que o vapor de ter etlico tenha comportamento
ideal e que o reservatrio do caminho tenha somente ter etlico.
PROBLEMA N 4: FSICO-QUMICA
Equilbrio Qumico
7% do total
Questes A.I. A.II A.III B.I B.II B.III B.IV total
Pontuao 0,6 0,9 1,0 1,0 0,5 1,5 1,5 7,0

Para um sistema qualquer, o equilbrio termodinmico se estabelece quando os equilbrios
mecnico, trmico e qumico so alcanados, ou seja, nenhuma mudana de propriedade
fsica ou qumica varia com o tempo. Podemos ter uma viso do equilbrio qumico tanto
H
vap
=29,0kJ/mol;densidade=0,750g/mL(-5,0C);
T
crtica
=193C;P
crtica
=3,64MPa;R=8,314Jmol-1K-1,1atm=101325
Pa
C
p,m
=107,1Jmol
-1
K
-1
;
Para todas as respostas, considere que o vapor de ter etlico tenha
comportamentoidealequeoreservatriodocaminhotenhasomente
teretlico.
PROBLEMAN4:FSICO-QUMICA
EquilbrioQumico
7%dototal
Questes A.I. A.II A.III B.I B.II B.III B.IV
total
Pontuao 0,6 0,9 1,0 1,0 0,5 1,5
1,5 7,0
Paraumsistemaqualquer,oequilbriotermodinmicoseestabelecequando
osequilbriosmecnico,trmicoequmicosoalcanados,ouseja,nenhu-
mamudanadepropriedadefsicaouqumicavariacomotempo.Podemos
terumavisodoequilbrioqumicotantonasreaessemtransfernciade
eltronscomoparaaquelascomtransfernciadeeltrons.Nestasltimas,a
transformaodeenergiamaisevidente.
PARTE A) As mudanas no ponto de equilbrio podem ser avaliadas pela
aplicaodoPrincpiodeLeChatelier.Nossistemasdoquadro,queseen-
16
a
OIAQ
| 141
Olimpada Brasileira
de Qumica 2011
Ano Internacional de Qumica
contramemequilbrio,serealizamasseguintesaes:
6

nas reaes sem transferncia de eltrons como para aquelas com transferncia de eltrons.
Nestas ltimas, a transformao de energia mais evidente.
PARTE A) As mudanas no ponto de equilbrio podem ser avaliadas pela aplicao do
Princpio de Le Chatelier. Nos sistemas do quadro, que se encontram em equilbrio, se
realizam as seguintes aes:
Sistema Ao
1. CO
2 (g)
+ H
2 (g)
H
2
O
(g)
+ CO
(g)
Um agente secante adicionado.
2. H
2 (g)
+ I
2 (g)
2HI
(g)
Gs nitrognio adicionado.
3. 2 NaCl
(s)
+ H
2
SO
4(l)
Na
2
SO
4(s)
+ 2 HCl
(g)
A reao realizada em recipiente
aberto.
A.I) Para cada ao anterior, indique na folha de respostas se as afirmaes a, b e c so
verdadeiras ou falsas.
a - A reao 1 no se altera, pois o secante mantm o mesmo volume de gua;
b - Na reao 2, o gs adicionado provoca uma mudana para a direita;
c - A reao 3 permanece sem alterao.
AII) Assinale na tabela da folha de respostas o resultado esperado para cada ao indicada
no quadro acima.
A.III) Quantitativamente possvel avaliar qual reao ocorreu completamente? Ou seja,
em qual delas um dos reagentes foi consumido 99,9%? Em caso afirmativo, indique na
folha de respostas o nmero do sistema.
PARTE B) A obteno de energia limpa pode ser conseguida atravs das reaes de oxi-
reduo, como no caso do acumulador de Edson:
Fe(s)|FeO(s)|KOH(aq, c)|Ni
2
O
3
(s)|NiO(s)|Ni(s)
B.I) Escreva as reaes de meia pilha;
B.II) Qual a reao da pilha?
B.III) Indique na folha de respostas qual a relao entre o potencial da pilha e a
concentrao, c, do KOH?
A.I)Paracadaaoanterior,indiquenafolhaderespostasseasafrmaesa,
becsoverdadeirasoufalsas.
a - A reao 1 no se altera, pois o secante mantm o mesmo volume de
gua;
b-Nareao2,ogsadicionadoprovocaumamudanaparaadireita;
c-Areao3permanecesemalterao.
AII)Assinalenatabeladafolhaderespostasoresultadoesperadoparacada
aoindicadanoquadroacima.
A.III)Quantitativamentepossvelavaliarqualreaoocorreucompletamen-
te?Ouseja,emqualdelasumdosreagentesfoiconsumido99,9%?Emcaso
afrmativo,indiquenafolhaderespostasonmerodosistema.
PARTE B) A obteno de energia limpa pode ser conseguida atravs das
reaesdeoxi-reduo,comonocasodoacumuladordeEdson:
Fe(s)|FeO(s)|KOH(aq,c)|Ni
2
O
3
(s)|NiO(s)|Ni(s)
B.I)Escrevaasreaesdemeiapilha;
B.II)Qualareaodapilha?
B.III)Indiquenafolhaderespostasqualarelaoentreopotencialdapilha
eaconcentrao,c,doKOH?
B.IV)Quantodeenergia(G)seobterporquilogramadetodososreagen-
tesdapilha?
Exame Terico
| 143 142 |
Programa Nacional
Olimpadas de Qumica
Ano Internacional de Qumica
Dados:
MAemg/mol: H=1,00 O=15,99 K=39,10 Fe=55,85; Ni=58,71
7

B.IV) Quanto de energia (G) se obter por quilograma de todos os reagentes da pilha?
Dados:
MA em g/mol: H = 1,00 O = 15,99 K = 39,10 Fe = 55,85; Ni = 58,71
K
+
+ e

K = 2,925 V
Fe
2+
+ 2e

Fe = 0,440 V
FeO + H
2
O+ 2e

Fe +2OH

= 0,877 V
Ni
2+
+ 2e

Ni = 0,250 V
Ni
2
O
3
+ H
2
O + 2e

2NiO + 2OH

= +0,40 V
Constante de Faraday = 9,648 .10
4
C mol
-1
PROBLEMA N 5: QUMICA ANALTICA
Equilbrio Qumico Kps do Cd(OH)
2
7% do total
Questes I II III IV V total
Pontuao 1,0 1,0 2,0 1,0 2,0 7,0
O elemento cdmio (n atmico 48, massa atmica 112,41) um metal pesado altamente
txico que ocorre naturalmente como on Cd
2+
, quase sempre associado com os minrios
de zinco, cobre, chumbo e mercrio. Apesar da baixa ocorrncia natural, sua presena no
ambiente preocupante, pois a toxicidade alta e a eliminao lenta (efeito cumulativo).
Algumas plantas (inclusive espcies comestveis) so bastante tolerantes sua presena no
solo e o acumulam s vezes em quantidades apreciveis. Arroz, trigo e batata so alguns
exemplos bem conhecidos dessa propenso acumulao de cdmio. Peixes pescados em
guas prximas a garimpos ou indstrias metalrgicas ou eletrnicas (baterias
recarregveis do tipo nicad) tambm representam riscos segurana alimentar. No Japo,
uma doena conhecida localmente por itai-itai atribuda ao alto consumo de peixe e arroz
ConstantedeFaraday=9,648.10
4
Cmol
-1
PROBLEMAN5:QUMICAANALTICA
EquilbrioQumicoKpsdoCd(OH)
2
7%dototal
Questes I II III IV V Total
Pontuao 1,0 1,0 2,0 1,0 2,0 7,0
Oelementocdmio(natmico48,massaatmica112,41)ummetalpesado
altamente txico que ocorre naturalmente como on Cd
2+
, quase sempre as-
sociadocomosminriosdezinco,cobre,chumboemercrio.Apesardabaixa
ocorrncianatural,suapresenanoambientepreocupante,poisatoxicidade
altaeaeliminaolenta(efeitocumulativo).Algumasplantas(inclusiveesp-
ciescomestveis)sobastantetolerantessuapresenanosoloeoacumulam
svezesemquantidadesapreciveis.Arroz,trigoebatatasoalgunsexemplos
bemconhecidosdessapropensoacumulaodecdmio.Peixespescadosem
guasprximasagarimposouindstriasmetalrgicasoueletrnicas(baterias
recarregveisdotiponicad)tambmrepresentamriscosseguranaalimentar.
NoJapo,umadoenaconhecidalocalmenteporitai-itai atribudaaoalto
consumodepeixeearrozpelapopulaoexpostaaessesdoistiposderisco.
Outrofatomundialmenteconhecidoodainsanidadementaldofamosopin-
16
a
OIAQ
| 143
Olimpada Brasileira
de Qumica 2011
Ano Internacional de Qumica
torholandsVincentvanGogh,aopintarOsGirassis,umadesuasprincipais
obras,cujostonsamareloseramobtidoscompigmentosbasedecdmio.
Uma poro de hidrxido de cdmio (Kps = 4,5 x 10-
15
) foi colocada em
gua pura a 25 C e agitada energicamente at a saturao da soluo. A
soluosaturadafoiseparadadoexcessodeslidoeumaalquotade100,0
mLfoiretiradaereservadaparausoposterior.
I)Escrevaaequaobalanceadaparaadissociaodohidrxidodecdmio
emgua;
II)EscrevaaexpressodoKpsparaasoluosaturadadehidrxidodecd-
mio;
III)CalculeasconcentraesdosonsOH
-
,H
+
eCd
2+
nasoluo;
IV)CalculeopHdasoluo,apartirda[H
+
];
V)alquotade100,0mLacrescentou-se0,10mLdesoluodeNaOH1,00
M;quaisasnovasconcentraesdosonsOH
-
,H
+
eCd
2+
?Ignorarava-
riaodevolume.
PROBLEMAN6:QUMICAANALTICA
Condutimetria
7%dototal
Questes I II III IV Total
Pontuao 4,0 1,0 1,0 1,0 7,0
Acondutimetriaumatcnicaanalticabastantetil,tantonadeterminao
de parmetros fsico-qumicos de solues eletrolticas, quanto na anlise
quantitativadeespcies inicasemsoluo.Noprimeirocaso,atcnica,
svezeschamadadecondutimetriadiretaeseaplica,porexemplo,nade-
terminaodepKdebasesecidosfracosedeKpsdesaispoucosolveis;
nosegundocaso,fala-sedetitulaocondutimtrica,cujasaplicaesso
basicamente as mesmas das outras titulaes feitas com auxlio de instru-
mentos.Nestecaso,mede-seacondutnciadasoluo(inversodaresistn-
ciapassagemdecorrente)emfunodovolumedetitulante.Adiferena
fundamentalqueseobtmretasbemdefnidas,cujainterseo(ge-
Exame Terico
| 145 144 |
Programa Nacional
Olimpadas de Qumica
Ano Internacional de Qumica
ralmenteobtidaporextrapolao)correspondeaumpontofnal.Expe-
rimentalmente,considera-seovolumegastonesteponto.
Emumaauladelaboratrioumestudantedesatentomisturourestosdeuma
soluodeHClcomsoluodecidoactico(HAc,pKa=4,74).Comocastigo
peladesateno,foi-lhedadaatarefadedeterminaraporcentagemdecada
cidonamistura.Oaluno,muitoesperto,pipetou10,0mLdamistura,diluiu
para 100 mL e titulou condutimetricamente com soluo de NH
4
OH 0,50
M padronizada recentemente. Os valores de volume de titulante (V) e das
respectivasleiturasdecondutncia(G),emunidadearbitrria(u.a),estono
quadroabaixo.
V
(mL)
0,0 1,0 2,0 2,5 3,0 3,1 3,2 3,5 4,0 4,5 5,0 6,0 7,0
G
(u.a)
2,85 2,50 2,10 1,90 1,70 1,66 1,70 1,75 2,00 2,15 2,15 2,14 2,16
I)TraceogrfcodeGversusVeencontreospontosdeequivalnciacorres-
pondentesneutralizaodecadacidonasoluodaamostra;
II)Calculeamassadecadacidonaamostra;
III)Determineacomposiodamistura(%m/v);
IV) A diluio da amostra na proporo de 1:10 e o uso de uma soluo
concentrada(0,50M)comotitulante,foiintencionaleteveporobjetivo:
Marqueaalternativacorretanafolhaderespostas.
16
a
OIAQ
| 145
Olimpada Brasileira
de Qumica 2011
Ano Internacional de Qumica
PROBLEMAN7:QUMICAORGNICA
TransformaesdoLquidodaCastanhadeCaju
8%dototal
Questes AI AII AIII AIV BI BII BIIII BIV total
Pontuao 1,0 1,0 1,0 1,0 1,0 1,0 1,0 1,0 8,0
Olquidodacascadacastanhadocaju(LCC)oucashew nut shell liquid (CNSL)
uma das fontes mais ricas de lipdeos fenlicos no-isoprenides de ori-
gemnatural,cujacomposioqumicaprincipalcidoanacrdico,cardol,
2-metilcardol e cardanol. Uma vez separado, o cardanol, pode ser empre-
gado no setor da qumica fna, onde os preos dos produtos fnais: aditi-
vos, surfactantes, frmacos, pesticidas, dentre outros, so elevados. Por ser
umsubprodutodaindstriadecastanha,qualquermelhoria:concentrao,
separaoepreparaodederivadossecaracterizaverdadeiramentecomo
umainovaotecnolgica.
PARTEA)Considerandoarelevnciadassntesesorgnicasenvolvendoes-
tescompostos,completeasequnciadereaesdocardanol,indicandoas
estruturasdoscompostosI,II,IIIeIV:
10

OH
C
15
H
31
CH
3
SO
2
Cl
HNO
3
10% Pd/C
10% Pd/C, Mg, CH
3
COONH
4
Hidrogenlise
H
2
SO
4
(cat)
H
2
, EtOH
C
21
H
34
N
2
O
4
PARTE B) As xantonas tm sido relatadas por possurem diversos efeitos biolgicos tais
como: propriedades antitumorais, anti-inflamatrios, antitrombtico, antimicrobial e
neurofarmacolgicas. Algumas destas xantonas podem ser sintetizadas utilizando-se
transformaes estruturais de lipdios fenlicos isoladas do LCC, os quais so obtidos em
abundncia e com baixo custo. Dentre eles podemos destacar o cido anacrdico (1), o
cardol (2) e anlogos sintticos no intuito de se obter xantonas alquiladas de interesse
farmacolgico.
Complete a seguinte sequncia de sntese de xantonas (3,5,7 e 9) a partir de derivados do
LCC, indicando as estruturas dos compostos 3, 4, 6 e 8:
Composto I
Composto II
Composto III Composto IV
Exame Terico
| 147 146 |
Programa Nacional
Olimpadas de Qumica
Ano Internacional de Qumica
PARTE B) As xantonas tm sido relatadas por possurem diversos efeitos
biolgicos tais como: propriedades antitumorais, anti-infamatrios, anti-
trombtico, antimicrobial e neurofarmacolgicas. Algumasdestas xantonas
podem ser sintetizadas utilizando-se transformaes estruturais de lipdios
fenlicosisoladasdoLCC,osquaissoobtidosemabundnciaecombaixo
custo. Dentre eles podemos destacar o cido anacrdico (1), o cardol (2) e
anlogos sintticos no intuito de se obter xantonas alquiladas de interesse
farmacolgico.
Completeaseguintesequnciadesntesedexantonas(3,5,7e9)apartirde
derivadosdoLCC,indicandoasestruturasdoscompostos3,4,6e8:
11

OH
COOH
C
15
H
31
+
O H
OH
C
15
H
31
1
2
OH
COOH
C
15
H
31
1
+
70%
O
O
H
31
C
15
OH
OH
4
5
+
O H
OH
C
15
H
31
2
6
65%
O
O
C
15
H
31
OH
7
+
O H
OH
C
15
H
31
2
65%
8
O
O
C
15
H
31
O
OH
9
3
ZnCl
2
, POCl
3
, 60 C, 2 h
75%
ZnCl
2
, POCl
3
, 60 C, 2 h
ZnCl
2
, POCl
3
, 60 C, 2 h
ZnCl
2
, POCl
3
, 60 C, 2 h
PROBLEMA N 8: QUMICA ORGNICA
Sntese Assimtrica
8% do total
Questes AI AII AIII AIV AV AVI BI BII BIII BIV total
Pontuao 1,0 1,0 1,0 1,0 1,0 1,0 0,8 0,4 0,4 0,4 8,0
As snteses assimtricas so de grande importncia para indstria farmacutica na
preparao de frmacos quirais, opticamente puros. Essas snteses so realizadas, muitas
vezes, partindo-se de um produto natural opticamente ativo. Na sequncia de reaes
apresentada abaixo, tem-se um exemplo de sntese assimtrica que leva formao da
olefina VII a partir do cido L(+)tartrico, comercialmente disponvel, que por sua vez
pode ser extrado do suco da uva. Essa sntese foi realizada como parte de um projeto de
16
a
OIAQ
| 147
Olimpada Brasileira
de Qumica 2011
Ano Internacional de Qumica
PROBLEMAN8:QUMICAORGNICA
SnteseAssimtrica
8%dototal
Questes AI AII AIII AIV AV AVI BI BII BIII BIV total
Pontuao 1,0 1,0 1,0 1,0 1,0 1,0 0,8 0,4 0,4 0,4 8,0
Assntesesassimtricassodegrandeimportnciaparaindstriafarmacu-
tica na preparao de frmacos quirais, opticamente puros. Essas snteses
sorealizadas,muitasvezes,partindo-sedeumprodutonaturalopticamente
ativo.Nasequnciadereaesapresentadaabaixo,tem-seumexemplode
snteseassimtricaquelevaformaodaolefnaVIIapartirdocidoL(+)
tartrico,comercialmentedisponvel,queporsuavezpodeserextradodo
sucodauva.Essasntesefoirealizadacomopartedeumprojetodetesede
doutorado que tinha como objetivo fnal a olefna VII que pode ser usada
como substrato para a obteno de uma srie de compostos de interesse
farmacolgico.
12

tese de doutorado que tinha como objetivo final a olefina VII que pode ser usada como
substrato para a obteno de uma srie de compostos de interesse farmacolgico.
OH
O H COOH
HOOC H
H
Composto I
MeOH, H
2
SO
4
refluxo 18 h
Composto II
Etapa 2
O
O
COOMe
COOMe
Composto III
LiAlH
4,
Et
2
O
refluxo, 3 h
Composto IV
Etapa 4
O
O
CH
2
OCH
2
C
6
H
5
CH
2
OH
Composto V
1. DMSO, (COCl)
2
, -60 C, 1,5 h
2. TEA, -60 C, 15 min
Oxidao de Swern (oxidao branda)
Composto VI
O
O
CH
2
OCH
2
C
6
H
5
Etapa 6
Composto VII
Onde TEA= trietilamina
A) Complete a sequncia de reaes acima, indicando os reagentes apropriados para as
etapas 2, 4 e 6 e os produtos que se formam nas etapas 1, 3 e 5 (compostos II, IV e VI,
respectivamente).
B) Escreva os nomes do composto I incluindo a estereoqumica e indique a configurao
dos carbonos assimtricos nos compostos III, V e VII, incluindo as configuraes dos
carbonos assimtricos.

Exame Terico
| 149 148 |
Programa Nacional
Olimpadas de Qumica
Ano Internacional de Qumica
A)Completeasequnciadereaesacima,indicandoosreagentesapropria-
dosparaasetapas2,4e6eosprodutosqueseformamnasetapas1,3e
5(compostosII,IVeVI,respectivamente).
B)EscrevaosnomesdocompostoIincluindoaestereoqumicaeindiquea
confguraodoscarbonosassimtricosnoscompostosIII,VeVII,incluin-
doasconfguraesdoscarbonosassimtricos.
Paraseteroquenuncaseteve
precisofazeroquenuncasefez
16
a
OIAQ
| 149
Olimpada Brasileira
de Qumica 2011
Ano Internacional de Qumica
Exame Prtico
XVIOlimpadaIberoamericanadeQumica
ProblemasPrticos

PROBLEMAEXPERIMENTALN1
Determinaodapurezadeumaamostradesulfatodecobre
Perguntas 1 2 3 4 5 Total
Pontuao 1 2 13 5 4 25
Introduo
O sulfato de cobre comercial ou de uso tcnico apresenta-se geralmente
comocristaisazuisdeCuSO
4
.5H
2
O,svezesacompanhadodematerialamor-
fo de tonalidade acinzentada. largamente usado em agropecuria como
micronutrientedeplantaseanimaisetambmcomofungicida.Aindstria
tambm o utiliza com graus de pureza e hidratao variveis, em diversas
aplicaes tecnolgicas, como: petroqumica, qumica fna, corantes e pig-
mentos,curtume,detergente,galvanoplastia,txtil,papelecelulose,etc.
Osaldealtapureza(reagenteP.A.)extremamentetilempesquisaeem
trabalhos prticos de ensino, especialmente no campo da eletroqumica e
qumica eletroanaltica. Os potenciais de reduo do cobre envolvendo os
estados de oxidao (0), (I) e (II) so tais que a interconverso entre eles e
tambm as reaes destes com outras espcies eletroativas, permitem sua
utilizaoemcondiesrelativamentebrandas,dispensando,emmuitosca-
sos,anecessidadedeseusareletrodose/ousolventesespeciais.
| 151 150 |
Programa Nacional
Olimpadas de Qumica
Ano Internacional de Qumica
NesteexperimentoserfeitaaanlisedeumaamostradeCuSO
4
.5H
2
O,cujos
sinaisdecontaminaoe/oudeterioraoerambemvisveisnosalslidoe
nasoluorecm-preparada:noslidoeravisvelapredominnciadeump
amorfo cinzento-esbranquiado com raros cristais pequenos e mal forma-
dos;nasoluorecm-preparada(asamostrascomoestoagora),observa-
-seslidodepositadonofundodorecipienteeaausnciadacorazulbri-
lhantecaracterstica.
Em casos assim, recomenda-se que seja feito algum tratamento, visando a
purifcao do composto. Recomenda-se tambm uma anlise prvia do
compostoimpuro,afmdequeaefcciadotratamentotambmsejaava-
liada, mediante anlise posterior do composto tratado e comparao dos
resultados.Paraconseguiresseobjetivo,omtodoiodomtrico,quefazuso
detiossulfatodesdiocomoredutordeiodo,egomadeamidocomoindi-
cador,mostra-sebastanteefcaz,rpidoeseguro(desdequeentreasimpu-
rezasnohajaqualquerinterferente).
ProcedimentoExperimental
Realizeoexperimentoemtriplicata.
1.Transfra25,0mLdesoluodaamostra(sulfatodecobreimpuro)usando
umapipetavolumtricade25mL,paraumbquerde150mLeacrescente
3,0gdeiodetodepotssioslidoporexperimentoeagitebemcombas-
todevidroedeixeamisturarepousarporcercade5minutos.Aproveite
esse tempo para encher a bureta com a soluo de tiossulfato de sdio
0,05M,tendoocuidadoprviodelavaraburetacomasoluodetiossul-
fatodesdio.
2.SepareoprecipitadodeCuIslidodasoluo-me,usandopapeldefltro
quantitativo e recolha o fltrado em erlenmeyer de 250 mL. Lave bem o
precipitadoeopapel,usandoumasoluomista*deKIeNa
2
S
2
O
3
(3a
5pequenaspores,ouatodesaparecimentodasmanchasescurasdo
papel,usecontagota,senecessrio)e,fnalmente,comsoluodiludade
KIsemtiossulfato.
3.*SOLUOMISTA:retire2,0mLdasoluodetiossulfatodabureta,re-
cebendo-aemumbquerde25mL;acrescentecercade10mLdasolu-
16
a
OIAQ
| 151
Olimpada Brasileira
de Qumica 2011
Ano Internacional de Qumica
odiludadeKIefaaumaligeiraagitaocircularoucombastopara
homogeneizar.Estasoluodeveserusadaintegralmente,mesmoqueo
precipitadoeopapelfquemlimposantesdeus-latoda.(casonecess-
rio,prepararmaissoluomista).
4.Coloqueoerlenmeyercomocontedosobaburetaetitulecomasolu-
odetiossulfatodesdio0,05M(agitaomanual),atocontedodo
erlenmeyertornar-seamarelobemclaro.Acrescente,ento,3,0mLdoin-
dicador(gomadeamido)econtinuetitulandoataviragemdoindicador
deazulparaincolor.
5.Reaesqumicasnobalanceadas:
I) Cu
2+
(aq)+I
-
(aq)CuI(s)+I
3
-(aq)
II) I
3
-(aq)+S
2
O
3
2-
(aq)I
-
(aq)+S
4
O
6
2-
(aq)
Questes
1. A soluo mista usada na lavagem do papel de fltro com precipitado
composta de KI e Na
2
S
2
O
3
, ambos diludos. A funo de cada reagente,
nestaordem:
a)InibiradissoluodoCuIereduziroiodo(impregnadonopapel)aon
iodeto( ).
b)Inibir a dissoluo do CuI e solubilizar o iodo (impregnado no papel),
semqueestesejareduzidoaoniodeto( ).
c) Solubilizaroiodetodecobre(I)ereduziroonCu
+
acobremetlico().
d)Solubilizaroiodetodecobre(I)eoxidaroonCu
+
aonCu
2+
().
2.OvolumedesoluodeNa
2
S
2
O
3
quefoiretiradodaburetaparapreparar
asoluomistadeveserconsideradonovolumetotaldetitulantegastona
titulao?
3.A)BalancearasequaesIeII.
B)Registrenatabelaosvolumestotaisutilizadosdetiosulfatodesdioem
cadatitulao.
C)Qualovolumedetiosulfatodesdioqueserusadonosclculos?
Exame Prtico
| 153 152 |
Programa Nacional
Olimpadas de Qumica
Ano Internacional de Qumica
4.Seosulfatodecobreusadonapreparaodasoluoanalisadafossepuro
(MM=249,68g.mol
-1
),amassadeCuSO
4
.5H
2
Oexistentenaalquotaana-
lisadaseria_________g;amassaencontradafoi__________g.Combasenes-
tesvalores,oteordeimpurezasdosalanalisadode__________%(m/m).
5.OvalordeKpsdoCuI1x10
-12
.Combasenestevalor,responda:
a)AmassadeKIusadanoexperimentogarantesufcienteexcessodeon
iodeto,paraqueareaosejaseguramentequantitativa?
b)Admitindo,maisumavez,queosulfatodecobreusadofossepuro,qual
seriaovolumeesperadodetitulantequevocusounesteexperimento?
c)AbaixasolubilidadedoCuIemguacontribuiparaaespontaneidade
dareao?
PROBLEMAEXPERIMENTALN2
Identifcaodegruposfuncionaisportestesqumicos
Perguntas
1 2 3 Total
Pontuao 6 6 3 15
Atualmente os compostos orgnicos so identifcados pelo uso de mto-
dos fsicos, tais como: a espectroscopia de ressonncia magntica nuclear
de hidrognio e carbono-13 (RMN
1
H e RMN
13
C), espectroscopia no infra-
vermelho (IV) e no ultravioleta (UV) e espectrometria de massas (EM). No
entanto,algunstestesqumicossimples,defcilexecuo,continuamsendo
teis para arpida caracterizao dapresena de certos grupos funcionais
emumamolculaorgnica,umavezqueumgrupofuncionalaparteda
molculaondeassuasreaesqumicasocorrem;apartequeefetivamente
determina as propriedades qumicas do composto (e muitas das suas pro-
priedadesfsicastambm).
DentreestestestespodemosdestacarostestesdeLucas,deJones,deTollens,
doIodofrmio,da2,4-Dinitrofenil-hidrazinaedoCloretofrrico.
Neste experimento so fornecidas 6 (seis) amostras desconhecidas e uma
lista de 12 (doze) compostos, entre os quais se encontram os compostos
16
a
OIAQ
| 153
Olimpada Brasileira
de Qumica 2011
Ano Internacional de Qumica
Exame Prtico
correspondentes a cada uma dessas amostras. Voc dever, com base nos
resultadosobtidoscomarealizaodostestesacimacitados,identifcarcada
umadessasamostras.
Os procedimentos utilizados em cada um dos testes citados acima so os
seguintes:
TestedeLucas:Misturar,emumtubodeensaioseco,1mLdoreagentede
Lucascom4ou5gotasdaamostraaseranalisada.Aturvaodasoluoou
oaparecimentodeduascamadasindicamTESTEPOSITIVO.
TestedeJones:Colocaremumtubodeensaio,4ou5gotasdaamostraa
seranalisadaejuntar4ou5gotasdasoluodecidocrmico.Oapareci-
mentoimediatodeumprecipitadoverdeindicaTESTEPOSITIVO.Noreali-
zaressetestecomaamostraA:reaoaltamenteexotrmica.
TestedeTollens:Colocaremumtubodeensaio,cercade0,5mLdorea-
gente Tollens e 0,5 mL da amostra. A formao de um precipitado escuro
depratae/ouaformaodeespelhodepratasoresultadosindicativosde
TESTEPOSITIVO.
Testecom2,4-dinitrofenil-hidrazina(2,4-DNF):Colocaremumtubode
ensaio,4ou5gotasdaamostraaseranalisadaemcercade2mLdasoluo
de2,4-dinitrofenil-hidrazina.Agitaredeixaremrepousopor15minutos.Um
precipitadoamarelo-avermelhadoconsideradoTESTEPOSITIVO.
Testedoiodofrmio(CHI
3
):Colocaremumtubodeensaio,4ou5gotas
daamostraaserexaminadaejuntar1mLdesoluodeNaOHa10%.Em
seguida adicionar soluo de iodo, gota a gota, agitando sempre, at um
leve excesso, evidenciado pela colorao tpica do iodo, persistente por 5
minutos.Oaparecimentodeumprecipitadoamarelodeiodofrmioconsi-
deradoTESTEPOSITIVO.
Testecomcloretofrrico(FeCl
3
):Colocaremumtubodeensaio,cercade
1mLdasoluodecloretofrricoa3%,adicionar4ou5gotasdaamostrae
observarodesenvolvimentodecor.AcoloraovioletaoupretaindicaTESTE
POSITIVO.

| 155 154 |
Programa Nacional
Olimpadas de Qumica
Ano Internacional de Qumica
PROCEDIMENOEXPERIMENTAL
Utilizandoostubosdeensaiodisponveis,realize,dentreostestesacimaci-
tados, aqueles que forem necessrio para a identifcao de cada uma das
amostras (A a F) que lhe forem fornecidas, dentre os compostos citados a
seguir:
01cidoglico(cido3,4,5-tri-hidroxibenzico)
02lcoolterc-butlico
03Benzaldedo
04Cardanol(3-pentadecilfenol)
05n-Butanol
06Ciclo-hexeno
07D-Glicose
08.D-Manitol
09.cidoL-tartrico
10Hexano
11Pentan-3-ona
12Propanona
1.Preenchaatabelaabaixo,com(+)seotesteforconsideradopositivo,(-)
seotesteforconsideradonegativoe(NR)senofoinecessriorealizaro
testeparaaidentifcaodaamostra:
Amostra
RESULTADODOSTESTES
Composto
Lucas Jones Tollens 2,4-DNF CHI
3
FeCl
3
A
B
C
D
E
F

16
a
OIAQ
| 155
Olimpada Brasileira
de Qumica 2011
Ano Internacional de Qumica
Observaes:
1.NOREALIZAROTESTEDEJONESCOMOCOMPOSTOA.
2.Oscompostosslidosestofornecidosnaformadesoluoaquosa.
3.OcompostoAapresentaaatividadeptica.
4.OcompostoBreagecombicarbonatodesdio.
5.OcompostoCdtestedeBayer(reaocomKMnO4)negativo.
6.OcompostoFnopticamenteativo.

2.Dadososreagentesutilizadosnostestes
Lucas:Cloretodezinco(ZnCl
2
)emHClconcentrado.
Jones:Trixidodecromo(CrO
3
)emH
2
SO
4
diludo.
Cloreto frrico: Cloreto de ferro III (FeCl
3
.6H
2
O) em gua, acidifcada com
HCl.
2,4-Dinitrofenil-hidrazina:2,4-dinitrofenil-hidrazinaemetanol95%acidi-
fcadocomHCl.
Iodofrmio:SoluodeNaOHeSoluodeiodoemiodetodepotssio.
Tollens:Soluoamoniacaldenitratodeprata,[Ag(NH
3
)
2
]OH.
Escrevaasreaesqumicasgeraisquetraduzamreaesdecompostosor-
gnicosquedotestepositivocomosreagentesindicados.
3.Escrevaareaoqumicaquepoderiaserutilizadaparaaidentifcaode
umalcenoeoutrareaoqumicaquepoderiaserutilizadanaidentifca-
odeumcidocarboxlico.
Exame Prtico
| 157 156 |
Programa Nacional
Olimpadas de Qumica
Ano Internacional de Qumica
Destaques Olmpicos - 2011
Ao iniciar o processo seletivo para a escolha da equipe que representa
oBrasilnasolimpadasinternacionaisdequmicaem2011tnhamos35es-
tudantes dos quais quinze participaram do 10 Curso de Aprofundamento
eExcelnciaemQumicaministradonoInstitutodeQumicadaUNICAMP.
Durante15diasecomaparticipaodeprofessoresdoProgramadePs-
-graduaodoIQ-UNICAMPesse cursotranscorreucomaparticipaode
estudantesdasseguintesunidadesfederativas:Cear:BiancaRohsnerBezer-
ra,BrenoSaldanhaSousa,BrunoLimaverdeVillarLbo,DaviRodriguesCha-
ves,DavidsonAnthonyAragoFreire,EmersonHolandaMarinho,LaraMula-
toLima,NatliaAragoDias,RaulBrunoMachadodaSilva,SergioPereirade
OliveiraJnior,TaynaraCarvalhoSilvaeYuriJernimoMoreira;Piau:Pedro
Victor Barbosa Nolto; So Paulo: Daniel Arjona de Andrade Hara, Tbata
CludiaAmaraldePontes.Encerradoocurso,osestudantesseprepararam
para o ltimo exame, FASE VI da OBQ-2010 que defniu aqueles que re-
presentaramoBrasilnasolimpadasinternacionaisrealizadasem2011:43rd
InternationalChemistryOlympiad(Ankara)e16OlimpadaIberoamericana
deQumica,realizadaemTeresina(osassinaladosemnegritointegraramas
delegaes).
ParaAnkara,localda43rdInternationalChemistryOlympiad,adelega-
obrasileiraseguiucomquatroestudantes,Davi,Davidson,RauleTbata,
estadoestadodeSoPauloeostrsprimeirosdoestadodoCear.Aequipe
brasileiraconquistouduasmedalhasdebronzeeumamedalhadeprata,esta
conquistadapeloestudantecearenseDaviChaves.
Na16OlimpadaIberoamericanadeQumica,realizadaemTeresina,o
Brasilesteverepresentadopelosestudantescearenses,Davi,RauleTbatae
pelopaulistaDanielArjona.
Aequipeconquistou3medalhasdeouroe1debronze,primeirolugar
geralindividualemelhordesempenhoporequipe.
Destaques
| 157
Olimpada Brasileira
de Qumica 2011
Ano Internacional de Qumica
Destaques
EDaviChaves,medalhadepratana43
rd
IChOe
medalhadeourona14
a
OIAQ.
Raul Bruno, medalha de bronze na 43
rd
IChO e
medalhadeourona14
a
OIAQ.
TbataCludia,medalhadebronze
na43
rd
IChOemedalhadeourona14
a
OIAQ.
DavidsonAnthony,representouoBrasil
na43
rd
IChO.
| 159 158 |
Programa Nacional
Olimpadas de Qumica
Ano Internacional de Qumica
Destaques
DanielArjona,
medalhadebronze
na14
a
OIAQ.
Estudantes que representaram o Brasil na
16a Olimpada Iberoamericana de Qumica
realizadaemTeresina-Brasil.
Da esquerda para a direita: Tbata, Davi e
RauleDaniel.
| 159
Olimpada Brasileira
de Qumica 2011
Ano Internacional de Qumica
Destaques
Estudantes que representaram o Brasil na 43
rd
International Chemistry Olympiad realizada em
Ancara-Turquia.Daesquerdaparaadireita:Davidson,Tbata,DavieRaul.
| 161 160 |
Programa Nacional
Olimpadas de Qumica
Ano Internacional de Qumica
Depoimentos
Daolimpadaparaadocncia
M
inha participao no Programa Nacional de Olimpadas de
Qumicarepresentaummarcoquecontribuiuparavriasescolhas
eoportunidades.Malsabiaqueaoassistirsaulasintrodutrias
Qumicanooitavoano,essadisciplinarepresentariafuturamente
minhavidapessoal,socialeprofssional.Apsdezanos,aindahlourospor
contadoprograma.
O incio se deu em 2002, quando aos doze anos comecei a entender o
queeraQumica,ondepoderiaus-laecomous-laemnossofavor.Apartir
da, ocorreu a premiao da Maratona Cearense de Qumica, o primeiro
estmulo a continuar na carreira. medida que o ano se passava, havia um
maior entendimento entre mim e essa disciplina; foi-se estabelecendo uma
relaontima,auxiliadaportodososexcelentesprofessoresquetive.Nooutro
ano,maisumaMaratonaeumaOlimpadaCearense.Jhaviasidocativado.
EstudarQumicavirouprazer;curiosidadequemepareceinfnitadeentender
eentendermaisemais.EmoutraOlimpadaCearensedeQumica,conquistei
a oportunidade de participar da Fase III nacional da Olimpada Brasileira de
Qumica(OBQ)e,emseguida,aclassifcaoparaaFaseIV.Jhaviaumameta
acumprirnessemomento:comporadelegaobrasileiraparaasOlimpadas
Internacionais.Ametafoicumpridaeparticipeida38OlimpadaInternacional
deQumica(38
th
IChO)nacidadedeGyeongsan,CoriadoSul,almda11
OlimpadaIbero-AmericanadeQumica(11OIAQ),conquistandomedalhas
debronzeeouro,respectivamente.
No entanto, notei que usava a Qumica para crescer, quando participei
da Fase V da OBQ. Junto a um grupo de dezesseis estudantes, criava meus
primeiroslaossociaisintermediadosporessacincia.Crieilaosdeamizade
e at pequenos laos profssionais nesse momento, que me seriam bem
importantes para o futuro. Outros foram criados ao participar da IChO e
fortifcadosduranteaOIAQ.Nessesentido,houveatumprmioinusitadoque
ganheinaCoria,odeparticipantemaissocivel.Jhaviaentoumgrande
grupodecontatosespalhadospelomundo,oquemepermitiuestabelecerum
imensointercmbiocultural.Visiteiefuianftriodeamigosestrangeirosfeitos
ali;aolimpadamehaviamostradoqueomundonotograndeassim.
| 161
Olimpada Brasileira
de Qumica 2011
Ano Internacional de Qumica
Depoimentos
Se no to grande, nada melhor que poder escolher onde gostaria de
cursarminhagraduaoemQumica,claro.Almdesociais,haviamasrelaes
profssionais criadas, que me conduziram ao maior centro de pesquisa em
QumicaOrgnicadoBrasil,quidaAmricaLatina.Curseiminhagraduaona
UniversidadeEstadualdeCampinas(UNICAMP),fuirecebidocomentusiasmoem
seulaboratriopeloProf.Dr.RonaldoPilli,umdosmaioresqumicosorgnicos
sintticosdopas.Nosomente,asolimpadastambmmelevaramaoLeibniz
InstitutfrPfanzenbiochemie(IPBInstitutoLeibnizdeBioqumicaBotnica,
traduolivre)emHalle,Alemanha,ondedesenvolviumtrabalhodepesquisa
porseismeses.Acredite:nuncapenseiqueduasmedalhaspudessemmelevar
tolonge.
Ocrescimentonofoiapenassocialouprofssional,foitambmpessoal.
Serumalunodeolimpadasimplicadisciplinar-se,criarmtodoedesenvolver
orespeito.Fundamentalfoiaintensidadedoapoiofamiliaredoincentivodos
meus eternos mestres, os professores que me mostraram exatamente como
exerceranobreprofssocomdedicaoeamor.
Minha participao efetiva em olimpadas se encerrou h seis anos, mas
soueternamentegratoaoqueoprogramameproporcionou.Amaneiraque
encontreidepoderretribuirasoportunidadesquetivefoiescolherserprofessor.
Com22anos,desenvolvohojeumtrabalhodemestradoemQumicaOrgnica
comoProf.Pilli,lecionoemumaescoladeCampinasesouumentusiastado
ProgramaNacionaldeOlimpadasdeQumica.
LucasM.Lira
Bel.emQumica-UNICAMP
| 163 162 |
Programa Nacional
Olimpadas de Qumica
Ano Internacional de Qumica
Depoimentos
Penetrandonomundo
microscpicodaqumica
Comeceiaparticipardasolimpadasdequmicaquandofaziaa8srie,em
2000. Decidi participar da competio, pois seria uma forma de me preparar
paraaprovaespecfcadeQumicadovestibularparamedicina.Antesdissomal
entendiaoqueessacinciaestudava,nemoqueeratomo!
O que me estimulava a estudar e me aprofundar na qumica e participar
das olimpadas era o prprio cerne da Qumica (o mundo submicroscpico,
as misturas, o uso de muito raciocnio, matemtica, fsica), mas o ambiente
agradveldasaulaspreparatriasparaaolimpadaaliadoformaempolgante
queosprofessoresensinavamforammuitoimportantesparaessadedicao.No
decorrer do Ensino Mdio, vi que no tinha vocao pra ser mdico, e pensei
emfazerEngenhariaQumica,depoisFarmcia,atqueescolhimeformarcomo
bacharelemQumica.EuqueriacontinuaraestudarQumica.Paramimeramuito
bomleroslivrosdeQumicadoEnsinoSuperior,pensarnosseusdesafosenas
questesreferentesaessacincia.
ParticipardasolimpadasdeQumicametrouxemuitosbenefcios.Napoca
doensinomdioeuconseguiaverasboasamizadesquefz,aalegriadasvitrias,o
prazerdoestudo,eodirecionamentonaescolhadaprofsso.Hojevejoogrande
benefciodoconhecimentobemfundamentadodeQumicaadquiridoduranteas
olimpadas,poisissomeajudouduranteagraduaoetemmeajudadonoexerccio
domagistriosuperior.ApoioocrescimentodoprojetodaOlimpadaBrasileira
deQumica,poisacreditoqueesseprojetocontribuiparaumanovageraode
profssionaisbrasileirosdealtaqualidadequetraroumprogressoaoBrasil,alm
dedespertarointeressepeloconhecimentocientfconosadolescentesdonosso
pas.Acreditoquedevemosserestudantes/profssionaisquealiamconhecimento
e trabalho a valores importantes como respeito, humildade, honestidade e
excelncia. Gastar tempo e recursos em avanos cientfcos e tecnolgicos que
podemtrazerprogressoeconmiconovalerapenasenocontribuirmospara
umasociedadecommaispazejustia,menosopressoemisria.Nossaprofsso
podeserpartedeumgrandepropsitonavida.
IgorMarquesCavalcante
Bacharel,MestreeDoutorandoemQumicaUFC
ProfessorsubstitutoSetordeQumicaGeraleInorgnicaDQOIUFC
MedalhadeouronaOBQ2002(Mod.A)enaOBQ2003(Mod.B)
MedalhadepratanaVIIIONNeQ-2002

| 163
Olimpada Brasileira
de Qumica 2011
Ano Internacional de Qumica
Consideraes Finais
Umanoparafcarnahistria
Qumicaachavequeabreoshorizontesdoconhecimentoquepermitem
vivermos com melhor qualidade de vida em sintonia com a conservao da
natureza.Aspesquisasqueresultaramemavanosnamedicina,naproduoem
larga escala de alimentos, nas fontes sustentveis de energia ou na criao de
novosmateriaistiveramsubstancialparticipaodaqumica.Sodescobertas,
procedimentos e produtos essenciais para o desenvolvimento sustentvel do
nossomundoderivadasdopotencialcriativodaqumica.
A UNESCO entendeu que essas importantes contribuies deveriam ser
amplamente divulgadas e mereciam dar amplo conhecimento populao,
assimencaminhoupropostaparaaONUdestacar2011comoAnoInternacional
daQumica-AIQ.Aescolhadadatadeveu-seaocentenriodoPrmioNobel
de Qumica recebido por Mme Curie, agraciada em 1911 por suas pesquisas
comradioatividadeeadescobertadoselementosrdioepolnio.Dessaforma
destacouopapeldamulhernacinciaeaparticipaofemininaparaoavano
da qumica e para o bem-estar da sociedade. Apesar da reduzida quantidade
demulhereslaureadascomapremiaoNobel(5,2%)eapenasquatrodentre
162 laureados na rea de qumica, no Brasil, estima-se que a participao da
mulhernareadequmicaultrapassa1/3dototaldepesquisadores,umnmero
bastanteexpressivo.AsjustashomenagensprestadasMmeCurienocorrente
anosomotivodeestmuloaumamaiorparticipaodosjovens,especialmente
asmulheres,nomundocientfcoeincentivoparaseguiremcarreiranaqumica.
A contribuio feminina se destaca em praticamente todas as coordenadorias
estaduaisdoProgramaNacionalOlimpadasdeQumica,estoelasquercomo
coordenadorasouparticipandodaequipegestoranosprojetos.
SobosloganChemistry - our life, our futurequmicosdetodasaspartesdo
mundomovimentaramasociedadecompalestras,exposies,cursos,encontros
cientfcos e uma infnidade de experimentos postados na internet para
demonstrarcomoavanosdaqumicaimpactaramnaqualidadedevidadoser
humano.Sociedadesqumicas,escolas,universidadeseinstituiesbrasileirasque
ocupamdosfenmenosqumicosuniram-seUNESCOeIUPACparacelebraras
contribuiesvitaisdaqumica.Apreocupaocomapotabilidadedaguaque
consumimosfoiumdostemasdegranderepercusso,umexperimentoGlobal
sobreaQualidadedaguanoplanetamovimentoumilhesdeestudantesno
mundotodo,seusresultadosirointegrarumbancodedadossobreasituao
daguaderiosemananciaisdetodososcontinentes.
| 165 164 |
Programa Nacional
Olimpadas de Qumica
Ano Internacional de Qumica
Consideraes Finais
Importantes setores da indstria qumica brasileira associaram-se s
comemoraes do Ano Internacional da Qumica para divulgar a atuao
responsveleinovadoradaindstrianacional,aoitavadomundoeemfranco
crescimento. Esse trabalho ter continuidade, em 2012, com a colaborao do
ex-olmpico de qumica, Levindo Garcia Quarto, embaixador da qumica no
Brasil.Eleumdosquinzejovensque,porindicaodaIUPAC,foramnomeados
embaixadores em seus pases. Promover a aproximao da indstria com a
academia ser uma de suas misses durante 2012. Para receber orientaes,
juntamentecomosoutrosembaixadoresparticipoude1a5deoutubropassado,
em Berlim, da reunio anual da European Petrochemical Association (EPCA) -
umaredeglobalcomsedeemBruxelasquemovimentaumvolumedenegcios
de3,2trilhesdeeuroscom3,5milhesdeempregadosemtodoomundo.
EmbaladonascomemoraesdoAIQ,oProgramaNacionalOlimpadasde
Qumicaestimulouascoordenadoriasestaduaisaintensifcaremasatividadesde
promoodaqumicaespecifcamentenasescolasdeensinomdioefundamental
deformaaproporcionarointeressedosalunosporessacinciae,paraogrande
pblico, demonstrar os benefcios que a qumica oferece humanidade. Essas
iniciativastiverammaioralcancenosperodoscomemorativosaoDiadoQumico
e durante a Semana Nacional de Cincia e Tecnologia. Alm das olimpadas
estaduaiseregionaisapoiadaspeloProgramaNacionalOlimpadasdeQumica
os eventos de abrangncia nacional Olimpada Brasileira de Qumica (ensino
Mdio)eaOlimpadaBrasileiradeQumicajniortiveramexpressivaparticipao
dosjovensestudantes.
Neste ano, com incentivo da CAPES, iniciamos ousada ao de apoio ao
ensino e ao estudo da qumica nas escolas pblicas com a participao de
estudantes de licenciatura em qumica, ex-olmpicos do Programa Nacional
OlimpadasdeQumica.Trata-sedeprojetoquefamiliarizaoslicenciandoscomas
difculdadesdoensinopblico,contribuiparaaformaodosfuturosprofessores
dequmicaefacilitaoacessodealunosdasescolaspblicassuniversidades.A
prtica docente acompanhada por professores das prprias escolas pblicas
sobaorientaodedocentesdoscursosuniversitriosdequmica.Quinhentos
e setenta turmas de alunos foram formadas em treze estados brasileiros, um
investimento que se estender durante 2012 proporcionando slida formao
naquelesqueoptarampelacarreiradocenteemqumicaeestmulosaoestudo
daqumicanosestudantesdasescolaspblicas.
OanoseencerracomoannciodeumnovoparceironoProgramaNacional
Olimpadas de Qumica. Vem a Fundao Estudar trazer sua experincia de 20
anosdeatuaonareaeducacionaleconoscocolimaresforosnosentidode
| 165
Olimpada Brasileira
de Qumica 2011
Ano Internacional de Qumica
Consideraes Finais
despertar vocaes cientfcas, apoiar a formao profssional desses jovens e
incentivartalentoscompotencialparaliderana.Apartirdesteanoosparticipantes
das olimpadas de qumica que atingem a excelncia do rendimento de seus
estudosconcorremaoPrmioEstudarCincia,oferecidopelaFundaoEstudar.
Sejavocumdosselecionados.
Ao concluir mais um ciclo do Programa Nacional Olimpadas de Qumica
sentimo-nos recompensados pelo sucesso alcanado nas diferentes etapas,
traduzido pelo incremento no nmero de participantes, na destacada atuao
nasolimpadasinternacionais,nacrescentecapilarizaodoprojetoenaexitosa
captao de talentos para a qumica. Um esforo nacional desenvolvido pelos
coordenadoresdoprojetoparavencerodesafodetornaropascompetitivonas
reasdemdiaealtatecnologia.
A qumica uma cincia belssima cujo objeto de estudo
est sempre ao nosso redor: as transformaes da matria.
No pude deixar de me apaixonar por esta fascinante
matria a qual, sabia eu desde o princpio, no poderia
larg-la jamais.
Que o gosto pela Qumica seja algo que atravesse
geraes e seja sempre o propulsor para descobertas dos
detalhes do Universo que o homem desconhece. Essa
propulso que me foi dada pela Olimpada de Qumica
no se trata de algo efmero, mas de algo que perdurar
enquanto existir um esprito desbravador dentro de mim.
Raul Bruno Machado da Silva
Estudante olmpico
| 167 166 |
Programa Nacional
Olimpadas de Qumica
Ano Internacional de Qumica
Endereos
NCLEO COORDENADOR ENDEREOPROFISSIONAL
COORDENADORIAGERAL Prof.SrgioMaiaMelo
melo@ufc.br
sergio@funcap.ce.gov.br
FUNCAP-FundaoCearensedeApoioao
DesenvolvimentoCientfcoeTecnolgico
Av.OliveiraPaiva,941
60.822-130Fortaleza-CE
(85)3101.2170Ramal1203275.7862(Fax)
COORDENADORIA
REGIESSUL/SUDESTE
Prof.lvaroChrispino
chrispino@infolink.com.br
CEFET-RJ
(21)642.6644(Telefax)
Vice-COORDENADORIA Prof.JosArimatiaDantasLopes
arilopes@gmail.com
arilopes@ufpi.br
UniversidadeFederaldoPiau
CentrodeCinciasdaNatureza
64.049-550Teresina-PI
(86)3215.58403215.5692(Telefax)
ESTADO COORDENADOR ENDEREOPROFISSIONAL
ACRE DlcioDiasMarques
delciomarques@globo.com
Prof.RogrioA.Sartori
rogerio_sartori@yahoo.com.br
UniversidadeFederaldoAcre-CCN
Depto.deCienciasdaNaturezaBR364Km4
63.915-900RioBranco-AC
(68)3901.2591
ALAGOAS Prof.JoacyVicenteFerreira
joacyferreira@ifal.edu.br
IFAL-CampusMacei
InstitutoFederaldeAlagoas
RuaMizaelDomingues,75Poo
57.020-600Maceio-AL
Fone:(82)2126-7000/70242126.7050(fax)
AMAZONAS
www.oaq.ufam.edu.br
oaq@ufam.edu.br
Prof.PauloRogriodaCostaCouceiro
couceiro@ufam.edu.br
UniversidadeFederaldoAmazonas
Bloco10-DepartamentodeQumica/ICE/UFAM
SetorNortedoCampusUniversitrioSen.Arthur
VirglioFilho
Av.Gal.RodrigoOtvioJordoRamos,6.200
69.077-000Manaus-AMCoroado
(92)3305.2874(telefax)
BAHIA
www.obaq.ufba.br
Prof.LafaieteAlmeidaCardoso
lafaiete@ufba.br

Profa.SonildaMariaTeixeiradaSilva
sonilda@laquam.qui.ufba.br
UniversidadeFederaldaBahia
InstitutodeQumica-Depto.Qui.Orgnica
RuaBarodeGeremoabo,s/n(Ondina)
40.170-115Salvador-BA
(71)3283.6813-3237.4117(Fax)
ColgioEstadualdaBahia-Central
Av.JoanaAnglica,PraaCarneiroRibeiro
Salvador-BA
Fone:(71)3237.4124(LAQUAM)
CEAR
http://www.necim.ufc.br
Prof.LeonildeMariaCmaraJatahy
necim@bol.com.br
leojatahy@ig.com.br
Prof.CludiaChristinaB.S.Carneiro
UniversidadeFederaldoCear
NECIM-NcleodeEnsinodeCinciase
Matemtica
Av.daUniversidade,2470
60020-180Fortaleza-Cear(85)3366.7796
| 167
Olimpada Brasileira
de Qumica 2011
Ano Internacional de Qumica
Endereos
DISTRITOFEDERAL
www.unb.br/iq/pet
Profa.ElaineRoseMaia
emaia@unb.br
elainerm@terra.com.br
petqui@unb.br
UniversidadedeBraslia-InstitutodeQumica
CampusUniversitrioDarcyRibeiro-
70910-970ICCSul-AsaNorteCx.Postal:04478
(61)3107-3895/2150/21473273.4149(FAX)
ESPRITOSANTO
www.cce.ufes.br/dqui/ocq-es
Prof.CarlosVitalPaixodeMelo
cvpaixao@npd.ufes.br
UniversidadeFederaldoEspritoSanto
DepartamentodeQumica-CCE
Av.FernandoFerrari,514Goiabeiras
29.075-910VitriaES
(27)3335.2486-3335.2826
GOIS
www.obqgoias.com.br
Prof.RenatoCndidodaSilva
obqgoias@yahoo.com.br

Prof.CarlosCzardaSilva
cefetjatai@yahoo.com.br

Prof.HernanedeToledoBarcelos
UniversidadeFederaldeGois,InstitutodeQumica
CamposIISamambaiaBlocoI
74001-970Goiania,GO
Telefone:623521.1167
CentroFederaldeEducaoTecnolgicadeGois
UNEDJATA
RuaRiachuelo,2090SetorSamuelGraham
75.800-000Jata-GO
(64)3632.8600
CentroFederaldeEducaoTecnolgicadeGois
CampusGoinia
Rua75,n46,Centro.
74055-110.Goinia-GO
(62)3227-2700
MARANHO Prof.JeanCarloAntunesCatapreta
jcac889@hotmail.com
UniversidadeFederaldoMaranho
DepartamentodeQumica
Av.dosPortugueses,s/nCampusdaBacanga
65.080-040SoLuis-MA
Fone:(98)3301.8280
MATOGROSSO Prof.LuizBoth
bothluiz@ibest.com.brouluiz_both@
hotmail.com
IFMT
Rua28,Quadra38,Casa14,JardimUniversitrio
78.075-592Cuiab-MT
Fone:653653.9206(IFMT)3663.1374
MATOGROSSODOSUL Prof.OnofreSalgadoSiqueira
onofre.s.siqueira@gmail.com
UniversidadeFederaldoMatoGrossodoSul
CentrodeCinciasExataseTecnologia-
DepartamentodeQumica(Cidadeuniversitria)
79.070-900CampoGrande-MS
(67)345.3556-345.3552(FAX)
MINASGERAIS
http://www.qui.ufmg.br/
omq/
Profa.AnaLuizadeQuadros
omq.ufmg@gmail.com
UniversidadeFederaldeMinasGerais
DepartamentodeQumica
InstitutodeCinciasExatas(ICEx)
Av.Pres.AntnioCarlos,6627-Pampulha
31.270-901BeloHorizonte-MG
(31)3409.7558Fax:(31)3499.5700
| 169 168 |
Programa Nacional
Olimpadas de Qumica
Ano Internacional de Qumica
PAR Prof.MrciodeSouzaFarias
toraqk@yahoo.com.br

Profa.PatrciadaLuz
pdaluz@yahoo.com
AssociaoBrasileiradeQumica
RuaOdeAlmeida,490,Ed.Rotary,Sala404
66017-050Belm-Par.
Fone/Fax:(091)3222.0870
918112.5382TIM9187435407(OI)
Fone:918215.4852(TIM)
PARABA Prof.VimarioSimesSilva
vimario@deq.ufcg.edu.br

Prof.FranciscoDantasFilho

Prof.JosEstreladosSantos
santosje@gmail.com
UniversidadeFederaldeCampinaGrande-Unidade
AcadmicadeEngenhariaQumica
58.109-900CampinaGrande(83)2101.1115

UniversidadeEstadualdaParaba-UEPB
CentrodeCinciasExataseSociaisAplicadas
CampusVII
58.500-000Patos-PB833421.1475
UniversidadeFederaldeCampinaGrande
RuaSrgioMoreiradeFigueiredo,S/N
Bairro:CasasPopulares
58.900-000Cajazeiras-PB
(83)3532-20403532.20633531.3940(FAX)
PARAN
http://www.oprq.daqbi.
ct.utfpr.edu.br/
Prof.JosCarlosColombo
colombo@utfpr.edu.br
UniversidadeTecnolgicaFederaldoParan
Av.7desetembro,3165
80.230-010Curitiba-PR
(41)3310.46663310.4787(fax)
PERNAMBUCO
olimpiadasdequimica.pe@
gmail.com
www.espacociencia.pe.gov.br/
index.php/atividade/quimica/
AntnioCarlosPavo
pavao@ufpe.br
Vice-coordenadora:
Profa.KarinaMaiaBatistadeOliveira
kmboliveira@gmail.com
EspaoCincia:813383.55253185.5528
MemorialArcoverde,Parque2,S/N,Complexode
SalgadinhoOlinda-PE
UniversidadeFederaldePernambuco-CCEN
Depto.deQumicaFundamental-Cidade
Universitria
50.740-521Recife-PE
Fone:(81)2126.74152126.4646(fax)
PIAU
http://www.ufpi.br/quimica/
opq/
Prof.JosMiltonEliasdeMatos
jmematos@gmail.com
UniversidadeFederaldoPiau
DeptodeQumica-SG2CCN-CampusdaIninga
64.049-480Teresina-PI
Fone:(86)3215.5620telefax
RIODEJANEIRO
http://sites.google.com/site/
olimpiadadequimicarj/
oqrj2006@gmail.com
Prof.PauloChagas
paulocha@gmail.comou
luisquimica@globo.com
IFRJ-InstitutoFederaldeCincia
eTecnologiadoRiodeJaneiro
RuaSenadorFurtado,121PraadaBandeira
20.270-021RiodeJaneiro-RJ
(21)3978.59183567.0283(Fax)
Endereos
| 169
Olimpada Brasileira
de Qumica 2011
Ano Internacional de Qumica
Endereos
RIOGRANDEDONORTE Prof.KassioMichelGomesdeLima
kassiolima@gmail.com
Profa.MdeFtimaVitriade
Moura
mfvmoura@quimica.ufrn.br
UniversidadeFederaldoRioGrandedoNorte
CentrodeCinciasExatasedaTerra
DeptodeQumica
Av.SenadorSalgadoFilho,3000LagoaNova
CampusUniversitrio
59.072-970Natal-RN
Fone:(84)3215.3828R:2223215.9224(Fax)
RIOGRANDEDOSUL
http://gaia.liberato.com.br/
olimpiada/index.php
Twitter:http://twitter.com/
oqdors
CoordenaoColegiada:
Prof.DanielJacobus
Prof.FvioRobertoBeckerDillio
Prof.NairCristinaMuller
oqdors@gmail.com
quimica@liberato.com.br
FundaoEscolaTcnicaLiberatoSalzanoV.da
Cunha
RuaInconfdentes,395Primavera
93.340-140-NovoHamburgo-RS
(51)3584.2027
RONDNIA Profa.AdaianeSpinelli
obqrondonia@gmail.com
Prof.JandiCosta
jand_costa@hotmail.com
FundaoUniversidadeFederaldeRondnia-UNIR
NvleodeCinciasExatasedaTerra-NCET
DepartamentodeQumicaBR364km9
78.000-000PortoVelho-RO
Fone(69)2182.2193
RORAIMA Profa.MariaLciaTaveira
taveiraml@ig.com.br
UniversidadeFederaldeRoraima
DepartamentodeQumica-CampusdoParicarana
69.301-270BoaVista-RR
Fone:(95)621.3140/621.3137/623.1581-224.7302
(Fax)
SANTACATARINA Prof.GilsonRochaReynaldo
gilson@unisul.br
JosMaximilianoMullerNetto
max@crq.org.br
UniversidadedoSuldeSantaCatarina-UNISUL
Av.JosAccioMoreira,787-Caixapostal370
88.704-900Tubaro-SCBairroDehonFone:(48)
621.3116Fax(48)6213021
SOPAULO
http://allchemy.iq.usp.br
Prof.IvanoG.R.Gutz
gutz@iq.usp.br
abqsp@iq.usp.br
InstitutodeQuimicadaUniversidadedeSoPaulo
Av.Prof.LineuPrestes,748sala1274
05.508-000SaoPaulo-SP
(11)3091.2150(Telefax)
SERGIPE
www.osequim.hd1.com.br
Prof.JuvenalCarolinodaSilvaFilho
jcarolino@hotmail.com
Profa.ElianaMidoriSussuchi
esmidori@gmail.com
UniversidadeFederaldeSergipe-Depto.de
Qumica
CampusProf.AlbertoCarvalho
Av.VereadorOlimpioGrande,s/nCentro
49.500.000Itabaiana-SE
Fone:(79)3431.82163432.8200
TOCANTINS Profa.VanessaViebrantzOster
vanessaoster@gmail.com
InstitutoFederaldoTocantins-Depto.deQumica
AE310Sul,Av.LO05s/n
PlanodiretorSulCampusUniversitrio
77.020-210Palmas-TO
(63)3233.1300e3233.1309(Fax)
| 171 170 |
Programa Nacional
Olimpadas de Qumica
Ano Internacional de Qumica
PATROCINADORES
ABICLOR-AssociaoBrasileiradaIndstriade
lcaliseCloroDerivados
www.abiclor.com.br
RuaChedidJafet,222BlocoC4andarVilaOlmpia
04.551-065SoPauloSP
(11)2148.4780FAX2148.4788
ABIQUIM-AssociaoBrasileiradaIndstria
Qumica
www.abiquim.org.br
RuaChedidJafet,222BlocoC4andarVilaOlmpia
04.551-065SoPauloSP
(11)2148.4700FAX2148.4760
BancodoNordestedoBrasilSA
www.bnb.gov.br
Av.Paranjana,5700Castelo
60.180-420Fortaleza-CE
(85)4005.3300
PROMOTORES
UniversidadeFederaldoCear
Pr-ReitoriadeExtenso
Av.daUniversidade,2932-CampusdoBenfca
60.020Fortaleza-CE
Fone:(85)3366.7300
UniversidadeEstadualdoCear
Pr-ReitoriadeExtenso
Av.DedeBrasil,1700Paranjana
60.740-000Fortaleza-CE
Fone:(85)3299.2555
UniversidadeFederaldoPiau
Pr-ReitoriadeExtenso
CampusdaIninga
64.049-550Teresina-PI
Fone:(86)3215.5692Fax:(86)215.5570
FundaoCearensedeApoioaoDesenvolvimento
CientfcoeTecnolgico
Av.OliveiraPaiva,941
60.822-130Fortaleza-CE
Fone:(85)3101.2170Fax:3275.7862
APOIO
CNPq-ConselhoNacionaldeDesenvolvimento
CientfcoeTecnolgico
SHISQI1ConjuntoB-BlocoD,2andar
EdifcioSantosDumont,LagoSul
71605-190Braslia-DF
Fone:(61)3211-9408
CAPES-CoordenaodeAperfeioamentode
PessoaldeNvelSuperior
MinistriodaEducao-MEC
SetorBancrioNorte,Quadra02,BlocoL,Lote6,4.Andar
70.040-020-Braslia/DF
AssociaoBrasileiradeEngenhariaQumica
abeq@abeq.org.br
RuaLberoBardar,152-11andar-Centro
01008-903SoPaulo-SP
Fone:(11)3107-8747ouTelefax:(11)3104-4649
AssociaoNorte-NordestedeQumica
www.annq.com
CaixaPostal167
69.301-970BoaVistaRR
Fone:(95)3224.7128
Endereos
| 171
Olimpada Brasileira
de Qumica 2011
Ano Internacional de Qumica
EspaoCincia
www.espacociencia.pe.gov.br
ComplexodeSalgadinhos/nParque2
53.111-970OlindaPE
Fone:(81)3301-6139
FundaoEstudar
http://www.estudar.org.br
http://twitter.com/fundacaoestudar
Website:http://www.estudar.org.br
Tel:+55117124-4187
REALIZADOR
AssociaoBrasileiradeQumica
www.abq.org.br
Av.PresidenteVargas,633sala2208
20071-004RiodeJaneiroRJ
Telefone:212224-4480Fax:212224-6881
E-mail:abqrj@alternex.com.br
Endereos

Anda mungkin juga menyukai